[obm-l] Re: [obm-l] Questão de probabilidade

2022-06-29 Por tôpico Rogerio Ponce
Ola' Vanderlei e pessoal da lista!
Pediram-me para resolver o problema por inteiro.
Ok, vamos la'!


Em um pet shop ha' 3 gatos e 5 caes. Sabemos que 3 desses animais sao
pretos, 4 sao brancos e 1 e' malhado. Alem disso, pelo menos 1
cachorro e' preto. Assinale o que for correto.
01) A probabilidade de haver exatamente 1 cachorro preto e' de 1/6.
02) A probabilidade de haver pelo menos 1 gato branco e pelo menos 2
cachorros brancos e' de 2/3.
04) A probabilidade de haver um cachorro malhado e' maior do que a
probabilidade de haver um gato malhado.
08) Se um animal for escolhido ao acaso, a probabilidade de ele ser um
cachorro preto e' de 1/8.
16) Se um animal for escolhido ao acaso, a probabilidade de ele ser um
gato malhado e' de 1/16.



Afirmacao 01) A probabilidade de haver exatamente 1 cachorro preto e' de 1/6.

-

O cachorro que ja' e' preto, deixaremos de lado, de modo que temos
3Gatos,4Caes para pintar com 2Preto,4Branco,1Malhado.
Bem, dizer que existe exatamente 1 cachorro preto, significa dizer que
as outras 2 pinturas pretas foram para os gatos, ou que existem
exatamente 2 gatos pretos.

Entao, vamos comecar a pintar com a tinta preta, e verificar de
quantas formas podemos fazer isso.

Repare que para ponderar corretamente as ocorrencias que interessam,
eu considero todas as permutacoes possiveis, ou das tintas, ou dos
animais. Assim, posso, por exemplo, fixar a ordem das tintas, e
aplicar isso em todas as permutacoes possiveis entre os animais.
A razao entre os casos que interessam e os casos possiveis e'
justamente a probabilidade de ocorrencia de um caso que interessa.

Como sao 2 pinturas pretas, temos um total de 7 escolhas de animal
para a primeira pintura, e 6 escolhas para a segunda, com um total de
42 escolhas para a primeira e segunda pinturas pretas.

Deste total, temos 3 gatos para a primeira escolha e 2 gatos para a
segunda, com um total de 6 escolhas para termos 2 gatos pretos.
Deste total, vamos escolher 1 entre 3 gatos para a primeira pintura, e
1 entre 2 gatos para a segunda pintura, com um total de 6 escolhas
para pintarmos 2 gatos de preto.

Assim, a probabilidade de termos exatamente 2 gatos pretos vale 6/42 = 1/7
Ou seja, a probabilidade de termos exatamente um cachorro preto vale
1/7, de modo que a afirmacao 01 esta' errada.

OBS: note que, entre os gatos g1, g2 e g3 (por exemplo), mesmo que
pintemos g1 e g2 de preto, a ordem e' importante, pois ordens
diferentes correspondem a permutacoes diferentes. Pintar de preto o
gato g1, e depois o gato g2, e' uma permutacao diferente daquela em
que pintamos de preto primeiramente o gato g2, e depois o gato g1,
embora o resultado final seja obtermos g1 e g2 pretos.



Afirmacao 02) A probabilidade de haver pelo menos 1 gato branco e pelo
menos 2 cachorros brancos e' de 2/3.

-

O cachorro que ja' e' preto pode ser deixado de lado.

Seguindo agora com as 4 pinturas brancas, temos um total de 7 escolhas
para o primeiro animal, 6 para o segundo, 5 para o terceiro, e 4 para
o quarto, de modo que existem 7*6*5*4 escolhas diferentes para as
pinturas brancas.

Entre os animais que receberao as pinturas brancas, existem os seguintes casos:

1o caso: 4 caes brancos
O numero de combinacoes de 4 caes entre 4 caes e' C(4,4)=1.
Assim, considerando-se as permutacoes entre estes 4 animais
brancos, obtemos um total de permutacoes diferentes igual a
C(4,4)*4! = 1*4! = 24

2o caso: 3 caes e 1 gato brancos
Como sao 3 caes de um total de 4 caes possiveis, e 1 gato de
um total de 3 gatos possiveis, o numero total de permutacoes vale
C(4,3) * C(3,1) * 4! = 4*3*4! = 288

3o caso: 2 caes e 2 gatos brancos
o numero total de permutacoes vale
C(4,2) * C(3,2) * 4! = 6*3*4! = 432

4o caso: 1 cao e 3 gatos brancos
o numero total de permutacoes vale
C(4,1) * C(3,3) * 4! = 4*1*4! = 96


Contabilizando-se o segundo e o terceiro casos (que sao os que nos
interessam), temos um total de (288+432)=720 permutacoes, de um
universo de 840 permutacoes, de modo que a probabilidade pedida vale
720/840 = 6/7

Portanto, a afirmacao 02 esta' errada.



Afirmacao 04) A probabilidade de haver um cachorro malhado e' maior do
que a probabilidade de haver um gato malhado.

-

Deixando de lado o cachorro que ja' e' preto, e seguindo agora com a
pintura malhada, temos 7 escolhas de animais, das quais 4 sao de
cachorros, e 3 sao de gatos.

Assim, a probabilidade de haver um cachorro malhado e' 4/7 , e a
probabilidade de haver um gato malhado e' 3/7.

Portanto, a afirmacao 04 esta' correta.



Afirmacao 08) Se um animal for escolhido ao acaso, a probabilidade de
ele ser um cachorro preto e' de 1/8.

-

Se houvesse apenas um cachorro preto - aquele que ja' seria mesmo
preto, por definicao - a probabilidade de escolhermos um 

[obm-l] Re: [obm-l] Re: [obm-l] Re: [obm-l] Re: [obm-l] Re: [obm-l] Questão de probabilidade

2022-06-24 Por tôpico Rogerio Ponce
Otima explicacao!
Obrigado, Ralph!

PS: e sim, a provocacao foi pra voce mesmo!
:)

[]'s
Rogerio Ponce


On Wed, Jun 22, 2022 at 1:00 PM Ralph Costa Teixeira  wrote:
>
> Ponce está provocando a gente... senti que esta flecha tinha um bocado a 
> minha direção...  :D :D :D
>
> Olha, tem duas "visões" sobre o que "probabilidade" significa.
>
> A primeira vai na linha de que só podemos falar de probabilidade sobre coisas 
> que ainda não aconteceram. Vai nessa linha: se os eventos estão no passado, 
> então já aconteceram, já estão definidos, e não faz sentido dizer que tinha x 
> de chance de ser assim ou y de ser assado. Se você já jogou a moeda justa, 
> não é mais 50/50 --  é 100% de ser cara, ou 100% de ser coroa, dependendo do 
> que ocorreu. Quem pensa assim vai dizer que dado um certo evento (sempre no 
> futuro), ele tem uma probabilidade dada; se duas pessoas diferentes derem 
> duas probabilidades diferentes para o mesmo evento, uma delas errou.
>
> Outra linha diz que podemos falar de probabilidade sempre que houver 
> incerteza; não interessa o que aconteceu ou o que vai acontecer, o que 
> interessa é o que você SABE sobre o acontecimento. Se você jogou a moeda 
> justa mas eu não sei nada mais sobre o lançamento, continua sendo 50/50 
> **PARA MIM**. Probabilidade passa a ser um conceito sobre INFORMAÇÃO, não 
> sobre os fatos em si (a probabilidade não está na moeda, está no que você 
> sabe sobre a moeda). Quem pensa assim vai dizer que a probabilidade do evento 
> depende não apenas do evento em si, mas da informação que se tem em mãos. 
> Quem pensa assim admite que duas pessoas diferentes podem dar probabilidades 
> diferentes ao mesmo evento SE SOUBEREM FATOS DIFERENTES a respeito do evento, 
> ou seja, probabilidade passa a ser bastante "subjetivo".
>
> Eu talvez tenha descrito mal a primeira interpretação, pois sou ferrenho 
> defensor da segunda. Ela simplesmente engloba a primeira, porque você pode 
> ter informação parcial sobre fatos que ocorrem no futuro. E falar de 
> probabilidade para descrever incerteza presente ou passada é MUITO útil! Eu 
> quero poder expressar incerteza sobre fatos passados com frases do tipo 
> "fulano tem x% de probabilidade de ter cometido tal crime", ou "tem y% de 
> probabilidade de ter petróleo nesse poço", ou "tem z% de chance de eu ter 
> COVID"... Se você tem uma reação negativa a essas frases, lembre o que elas 
> realmente significam (na segunda interpretação): claro que ou o cara cometeu 
> o crime ou não, não faz sentido dizer que ele cometeu o crime x% das vezes em 
> que fizermos um experimento de ele cometer o crime... mas o que aquilo 
> significa é "com a informação que eu tenho, numa escala de 0 a 1, eu tenho 
> x/100 de certeza que fulano cometeu o crime". E "certeza baseada em informaç�!
 �es" é sim quantificável -- e satisfaz exatamente as leis das probabilidades 
com as quais concordamos. "Subjetivo" não significa "posso falar qualquer 
coisa", significa apenas que a conta pode variar de pessoa para pessoa... mas, 
de novo, SE ESSAS PESSOAS TIVEREM INFORMAÇÕES DIFERENTES sobre o evento.
>
> Abraço, Ralph.
>
> On Wed, Jun 22, 2022 at 12:09 PM Rogerio Ponce  wrote:
>>
>> Olá Pedro e pessoal da lista!
>>
>> Segundo a opinião do Pedro, nao faz sentido perguntar qual a probabilidade 
>> de Jose ter conseguido um 6 ao jogar o dado ontem, pois isso ja' aconteceu, 
>> e, portanto, ja' esta' definido.
>>
>> Sera' que e' isso mesmo?
>>
>> []'s
>> Rogerio Ponce
>>
>>
>> On Mon, Jun 20, 2022 at 9:45 PM Pedro José  wrote:
>>>
>>> Eu na minha humilde opinião creio que a probabilidade exista quando pode 
>>> ser uma coisa ou outra. No caso já é definido o que os animais são. Então 
>>> já está tudo errado. A questão seria viável se dessem esses limitantes para 
>>> uma criança que pintaria os desenhos dos animais. Aí sim há probabilidade.
>>>
>>> Em sáb., 18 de jun. de 2022 03:33, Rogerio Ponce  
>>> escreveu:

 Ola' Vanderlei e pessoal da lista!

 Sem perda de generalidade, podemos imaginar que vamos fazer o seguinte:

 - uma pintura preta em um dos caes, escolhido aleatoriamente

 - uma pintura "malhada" em um dos animais, escolhido aleatoriamente entre 
 os 7 animais nao pintados

 - duas pintura pretas, em dois animais, escolhidos aleatoriamente entre os 
 6 animais restantes,

 - quatro pinturas brancas nos 4 animais restantes


 Analisando a afirmacao 04, por exemplo, verificamos que, no segundo passo 
 (pintura malhada) existem 4 opcoes de cachorro e 3 opcoes de gato.

 Assim, a probabilidade de haver um cachorro malhado (4/7) e' maior que a 
 probabilidade de haver um gato malhado (3/7).
 Portanto, a afirmacao 04 esta' correta.
 (e o gabarito esta' errado).

 []'s
 Rogerio Ponce



 On Wed, Mar 16, 2022 at 8:08 AM Professor Vanderlei Nemitz 
  wrote:
>
> Bom dia!
> Na questão a seguir, do vestibular da UEM, 

[obm-l] Re: [obm-l] Re: [obm-l] Re: [obm-l] Re: [obm-l] Questão de probabilidade

2022-06-22 Por tôpico Ralph Costa Teixeira
Ponce está provocando a gente... senti que esta flecha tinha um bocado a
minha direção...  :D :D :D

Olha, tem duas "visões" sobre o que "probabilidade" significa.

A primeira vai na linha de que só podemos falar de probabilidade sobre
coisas que ainda não aconteceram. Vai nessa linha: se os eventos estão no
passado, então já aconteceram, já estão definidos, e não faz sentido dizer
que tinha x de chance de ser assim ou y de ser assado. Se você já jogou a
moeda justa, não é mais 50/50 --  é 100% de ser cara, ou 100% de ser coroa,
dependendo do que ocorreu. Quem pensa assim vai dizer que dado um certo
evento (sempre no futuro), ele tem uma probabilidade dada; se duas pessoas
diferentes derem duas probabilidades diferentes para o mesmo evento, uma
delas errou.

Outra linha diz que podemos falar de probabilidade sempre que houver
incerteza; não interessa o que aconteceu ou o que vai acontecer, o que
interessa é o que você SABE sobre o acontecimento. Se você jogou a moeda
justa mas eu não sei nada mais sobre o lançamento, continua sendo 50/50
**PARA MIM**. Probabilidade passa a ser um conceito sobre INFORMAÇÃO, não
sobre os fatos em si (a probabilidade não está na moeda, está no que você
sabe sobre a moeda). Quem pensa assim vai dizer que a probabilidade do
evento depende não apenas do evento em si, mas da informação que se tem em
mãos. Quem pensa assim admite que duas pessoas diferentes podem dar
probabilidades diferentes ao mesmo evento SE SOUBEREM FATOS DIFERENTES a
respeito do evento, ou seja, probabilidade passa a ser bastante "subjetivo".

Eu talvez tenha descrito mal a primeira interpretação, pois sou ferrenho
defensor da segunda. Ela simplesmente engloba a primeira, porque você pode
ter informação parcial sobre fatos que ocorrem no futuro. E falar de
probabilidade para descrever incerteza presente ou passada é MUITO útil! Eu
quero poder expressar incerteza sobre fatos passados com frases do tipo
"fulano tem x% de probabilidade de ter cometido tal crime", ou "tem y% de
probabilidade de ter petróleo nesse poço", ou "tem z% de chance de eu ter
COVID"... Se você tem uma reação negativa a essas frases, lembre o que elas
realmente significam (na segunda interpretação): claro que ou o cara
cometeu o crime ou não, não faz sentido dizer que ele cometeu o crime x%
das vezes em que fizermos um experimento de ele cometer o crime... mas o
que aquilo significa é "com a informação que eu tenho, numa escala de 0 a
1, eu tenho x/100 de certeza que fulano cometeu o crime". E "certeza
baseada em informações" é sim quantificável -- e satisfaz exatamente as
leis das probabilidades com as quais concordamos. "Subjetivo" não significa
"posso falar qualquer coisa", significa apenas que a conta pode variar de
pessoa para pessoa... mas, de novo, SE ESSAS PESSOAS TIVEREM INFORMAÇÕES
DIFERENTES sobre o evento.

Abraço, Ralph.

On Wed, Jun 22, 2022 at 12:09 PM Rogerio Ponce  wrote:

> Olá Pedro e pessoal da lista!
>
> Segundo a opinião do Pedro, nao faz sentido perguntar qual a probabilidade
> de Jose ter conseguido um 6 ao jogar o dado ontem, pois isso ja' aconteceu,
> e, portanto, ja' esta' definido.
>
> Sera' que e' isso mesmo?
>
> []'s
> Rogerio Ponce
>
>
> On Mon, Jun 20, 2022 at 9:45 PM Pedro José  wrote:
>
>> Eu na minha humilde opinião creio que a probabilidade exista quando pode
>> ser uma coisa ou outra. No caso já é definido o que os animais são. Então
>> já está tudo errado. A questão seria viável se dessem esses limitantes para
>> uma criança que pintaria os desenhos dos animais. Aí sim há probabilidade.
>>
>> Em sáb., 18 de jun. de 2022 03:33, Rogerio Ponce da Silva <
>> abrlw...@gmail.com> escreveu:
>>
>>> Ola' Vanderlei e pessoal da lista!
>>>
>>> Sem perda de generalidade, podemos imaginar que vamos fazer o seguinte:
>>>
>>> - uma pintura preta em um dos caes, escolhido aleatoriamente
>>>
>>> - uma pintura "malhada" em um dos animais, escolhido aleatoriamente
>>> entre os 7 animais nao pintados
>>>
>>> - duas pintura pretas, em dois animais, escolhidos aleatoriamente entre
>>> os 6 animais restantes,
>>>
>>> - quatro pinturas brancas nos 4 animais restantes
>>>
>>>
>>> Analisando a afirmacao 04, por exemplo, verificamos que, no segundo
>>> passo (pintura malhada) existem 4 opcoes de cachorro e 3 opcoes de gato.
>>>
>>> Assim, a probabilidade de haver um cachorro malhado (4/7) e' maior que a
>>> probabilidade de haver um gato malhado (3/7).
>>> Portanto, a afirmacao 04 esta' correta.
>>> (e o gabarito esta' errado).
>>>
>>> []'s
>>> Rogerio Ponce
>>>
>>>
>>>
>>> On Wed, Mar 16, 2022 at 8:08 AM Professor Vanderlei Nemitz <
>>> vanderma...@gmail.com> wrote:
>>>
 Bom dia!
 Na questão a seguir, do vestibular da UEM, penso que o espaço amostral
 tem 105 elementos, pois um cachorro é preto (desconsideramos esse). Porém,
 com esse pensamento, não consigo obter o gabarito, que diz que 02 e 16 são
 corretas.
 Alguém poderia ajudar?
 Muito obrigado!

 *Em um pet shop há 3 gatos e 5 cães. 

[obm-l] Re: [obm-l] Re: [obm-l] Re: [obm-l] Questão de probabilidade

2022-06-22 Por tôpico Rogerio Ponce
Olá Pedro e pessoal da lista!

Segundo a opinião do Pedro, nao faz sentido perguntar qual a probabilidade
de Jose ter conseguido um 6 ao jogar o dado ontem, pois isso ja' aconteceu,
e, portanto, ja' esta' definido.

Sera' que e' isso mesmo?

[]'s
Rogerio Ponce


On Mon, Jun 20, 2022 at 9:45 PM Pedro José  wrote:

> Eu na minha humilde opinião creio que a probabilidade exista quando pode
> ser uma coisa ou outra. No caso já é definido o que os animais são. Então
> já está tudo errado. A questão seria viável se dessem esses limitantes para
> uma criança que pintaria os desenhos dos animais. Aí sim há probabilidade.
>
> Em sáb., 18 de jun. de 2022 03:33, Rogerio Ponce da Silva <
> abrlw...@gmail.com> escreveu:
>
>> Ola' Vanderlei e pessoal da lista!
>>
>> Sem perda de generalidade, podemos imaginar que vamos fazer o seguinte:
>>
>> - uma pintura preta em um dos caes, escolhido aleatoriamente
>>
>> - uma pintura "malhada" em um dos animais, escolhido aleatoriamente entre
>> os 7 animais nao pintados
>>
>> - duas pintura pretas, em dois animais, escolhidos aleatoriamente entre
>> os 6 animais restantes,
>>
>> - quatro pinturas brancas nos 4 animais restantes
>>
>>
>> Analisando a afirmacao 04, por exemplo, verificamos que, no segundo passo
>> (pintura malhada) existem 4 opcoes de cachorro e 3 opcoes de gato.
>>
>> Assim, a probabilidade de haver um cachorro malhado (4/7) e' maior que a
>> probabilidade de haver um gato malhado (3/7).
>> Portanto, a afirmacao 04 esta' correta.
>> (e o gabarito esta' errado).
>>
>> []'s
>> Rogerio Ponce
>>
>>
>>
>> On Wed, Mar 16, 2022 at 8:08 AM Professor Vanderlei Nemitz <
>> vanderma...@gmail.com> wrote:
>>
>>> Bom dia!
>>> Na questão a seguir, do vestibular da UEM, penso que o espaço amostral
>>> tem 105 elementos, pois um cachorro é preto (desconsideramos esse). Porém,
>>> com esse pensamento, não consigo obter o gabarito, que diz que 02 e 16 são
>>> corretas.
>>> Alguém poderia ajudar?
>>> Muito obrigado!
>>>
>>> *Em um pet shop há 3 gatos e 5 cães. Sabemos que 3 desses animais são
>>> pretos, 4 são brancos e 1 é malhado. Além disso, pelo menos 1 cachorro é
>>> preto. Assinale o que for correto. *
>>> *01) A probabilidade de haver exatamente 1 cachorro preto é de 1/6. *
>>> *02) A probabilidade de haver pelo menos 1 gato branco e pelo menos 2
>>> cachorros brancos é de 2/3.*
>>> *04) A probabilidade de haver um cachorro malhado é maior do que a
>>> probabilidade de haver um gato malhado. *
>>> *08) Se um animal for escolhido ao acaso, a probabilidade de ele ser um
>>> cachorro preto é de 1/8. *
>>> *16) Se um animal for escolhido ao acaso, a probabilidade de ele ser um
>>> gato malhado é de 1/16.   *
>>>
>>> --
>>> Esta mensagem foi verificada pelo sistema de antivírus e
>>> acredita-se estar livre de perigo.
>>
>>
>> --
>> Esta mensagem foi verificada pelo sistema de antivírus e
>> acredita-se estar livre de perigo.
>
>
> --
> Esta mensagem foi verificada pelo sistema de antivírus e
> acredita-se estar livre de perigo.

-- 
Esta mensagem foi verificada pelo sistema de antiv�rus e
 acredita-se estar livre de perigo.



[obm-l] Re: [obm-l] Re: [obm-l] Questão de probabilidade

2022-06-20 Por tôpico Pedro José
Eu na minha humilde opinião creio que a probabilidade exista quando pode
ser uma coisa ou outra. No caso já é definido o que os animais são. Então
já está tudo errado. A questão seria viável se dessem esses limitantes para
uma criança que pintaria os desenhos dos animais. Aí sim há probabilidade.

Em sáb., 18 de jun. de 2022 03:33, Rogerio Ponce da Silva <
abrlw...@gmail.com> escreveu:

> Ola' Vanderlei e pessoal da lista!
>
> Sem perda de generalidade, podemos imaginar que vamos fazer o seguinte:
>
> - uma pintura preta em um dos caes, escolhido aleatoriamente
>
> - uma pintura "malhada" em um dos animais, escolhido aleatoriamente entre
> os 7 animais nao pintados
>
> - duas pintura pretas, em dois animais, escolhidos aleatoriamente entre os
> 6 animais restantes,
>
> - quatro pinturas brancas nos 4 animais restantes
>
>
> Analisando a afirmacao 04, por exemplo, verificamos que, no segundo passo
> (pintura malhada) existem 4 opcoes de cachorro e 3 opcoes de gato.
>
> Assim, a probabilidade de haver um cachorro malhado (4/7) e' maior que a
> probabilidade de haver um gato malhado (3/7).
> Portanto, a afirmacao 04 esta' correta.
> (e o gabarito esta' errado).
>
> []'s
> Rogerio Ponce
>
>
>
> On Wed, Mar 16, 2022 at 8:08 AM Professor Vanderlei Nemitz <
> vanderma...@gmail.com> wrote:
>
>> Bom dia!
>> Na questão a seguir, do vestibular da UEM, penso que o espaço amostral
>> tem 105 elementos, pois um cachorro é preto (desconsideramos esse). Porém,
>> com esse pensamento, não consigo obter o gabarito, que diz que 02 e 16 são
>> corretas.
>> Alguém poderia ajudar?
>> Muito obrigado!
>>
>> *Em um pet shop há 3 gatos e 5 cães. Sabemos que 3 desses animais são
>> pretos, 4 são brancos e 1 é malhado. Além disso, pelo menos 1 cachorro é
>> preto. Assinale o que for correto. *
>> *01) A probabilidade de haver exatamente 1 cachorro preto é de 1/6. *
>> *02) A probabilidade de haver pelo menos 1 gato branco e pelo menos 2
>> cachorros brancos é de 2/3.*
>> *04) A probabilidade de haver um cachorro malhado é maior do que a
>> probabilidade de haver um gato malhado. *
>> *08) Se um animal for escolhido ao acaso, a probabilidade de ele ser um
>> cachorro preto é de 1/8. *
>> *16) Se um animal for escolhido ao acaso, a probabilidade de ele ser um
>> gato malhado é de 1/16.   *
>>
>> --
>> Esta mensagem foi verificada pelo sistema de antivírus e
>> acredita-se estar livre de perigo.
>
>
> --
> Esta mensagem foi verificada pelo sistema de antivírus e
> acredita-se estar livre de perigo.

-- 
Esta mensagem foi verificada pelo sistema de antiv�rus e
 acredita-se estar livre de perigo.



[obm-l] Re: [obm-l] Questão de probabilidade

2022-06-18 Por tôpico Rogerio Ponce da Silva
Ola' Vanderlei e pessoal da lista!

Sem perda de generalidade, podemos imaginar que vamos fazer o seguinte:

- uma pintura preta em um dos caes, escolhido aleatoriamente

- uma pintura "malhada" em um dos animais, escolhido aleatoriamente entre
os 7 animais nao pintados

- duas pintura pretas, em dois animais, escolhidos aleatoriamente entre os
6 animais restantes,

- quatro pinturas brancas nos 4 animais restantes


Analisando a afirmacao 04, por exemplo, verificamos que, no segundo passo
(pintura malhada) existem 4 opcoes de cachorro e 3 opcoes de gato.

Assim, a probabilidade de haver um cachorro malhado (4/7) e' maior que a
probabilidade de haver um gato malhado (3/7).
Portanto, a afirmacao 04 esta' correta.
(e o gabarito esta' errado).

[]'s
Rogerio Ponce



On Wed, Mar 16, 2022 at 8:08 AM Professor Vanderlei Nemitz <
vanderma...@gmail.com> wrote:

> Bom dia!
> Na questão a seguir, do vestibular da UEM, penso que o espaço amostral tem
> 105 elementos, pois um cachorro é preto (desconsideramos esse). Porém, com
> esse pensamento, não consigo obter o gabarito, que diz que 02 e 16 são
> corretas.
> Alguém poderia ajudar?
> Muito obrigado!
>
> *Em um pet shop há 3 gatos e 5 cães. Sabemos que 3 desses animais são
> pretos, 4 são brancos e 1 é malhado. Além disso, pelo menos 1 cachorro é
> preto. Assinale o que for correto. *
> *01) A probabilidade de haver exatamente 1 cachorro preto é de 1/6. *
> *02) A probabilidade de haver pelo menos 1 gato branco e pelo menos 2
> cachorros brancos é de 2/3.*
> *04) A probabilidade de haver um cachorro malhado é maior do que a
> probabilidade de haver um gato malhado. *
> *08) Se um animal for escolhido ao acaso, a probabilidade de ele ser um
> cachorro preto é de 1/8. *
> *16) Se um animal for escolhido ao acaso, a probabilidade de ele ser um
> gato malhado é de 1/16.   *
>
> --
> Esta mensagem foi verificada pelo sistema de antivírus e
> acredita-se estar livre de perigo.

-- 
Esta mensagem foi verificada pelo sistema de antiv�rus e
 acredita-se estar livre de perigo.



[obm-l] Re: [obm-l] Re: [obm-l] Questão sobre desigualdades

2021-04-14 Por tôpico Carlos Monteiro
De onde saiu essa desigualdade?

Em qua., 14 de abr. de 2021 às 20:39, Anderson Torres <
torres.anderson...@gmail.com> escreveu:

> Em qua., 14 de abr. de 2021 às 15:54, Carlos Monteiro
>  escreveu:
> >
> > Encontre os valores máximo e mínimo da expressão:  x/(x^2+1) + y/(y^2+1)
> + z/(z^2+1) , onde x, y e z são números reais que satisfazem x+y+z = 1.
> >
> >
>
> Verifica-se que 3(12x+1)/50 >= x/(x^2+1), e assim o valor máximo é 3/10
>
> >
> >
> > --
> > Esta mensagem foi verificada pelo sistema de antivírus e
> > acredita-se estar livre de perigo.
>
> --
> Esta mensagem foi verificada pelo sistema de antivírus e
>  acredita-se estar livre de perigo.
>
>
> =
> Instru�ões para entrar na lista, sair da lista e usar a lista em
> http://www.mat.puc-rio.br/~obmlistas/obm-l.html
> =
>

-- 
Esta mensagem foi verificada pelo sistema de antiv�rus e
 acredita-se estar livre de perigo.



[obm-l] Re: [obm-l] Questão sobre desigualdades

2021-04-14 Por tôpico Anderson Torres
Em qua., 14 de abr. de 2021 às 15:54, Carlos Monteiro
 escreveu:
>
> Encontre os valores máximo e mínimo da expressão:  x/(x^2+1) + y/(y^2+1) + 
> z/(z^2+1) , onde x, y e z são números reais que satisfazem x+y+z = 1.
>
>

Verifica-se que 3(12x+1)/50 >= x/(x^2+1), e assim o valor máximo é 3/10

>
>
> --
> Esta mensagem foi verificada pelo sistema de antivírus e
> acredita-se estar livre de perigo.

-- 
Esta mensagem foi verificada pelo sistema de antiv�rus e
 acredita-se estar livre de perigo.


=
Instru��es para entrar na lista, sair da lista e usar a lista em
http://www.mat.puc-rio.br/~obmlistas/obm-l.html
=


[obm-l] Re: [obm-l] Re: [obm-l] Re: [obm-l] Questão OBM - U

2020-01-23 Por tôpico Ralph Teixeira
Seja ABCD o quadrilatero convexo, e seja P o encontro das diagonais.

No triangulo APB, temos AP+PB>AB. Escreva as desigualdades analogas para os
triangulos BPC, CPD e DPA. Somando-as, voce vai obter que

2(AC+BD)>perimetro=8

Ou seja, o infimo tem que ser pelo menos 4.

Agora, para chegar no infimo, voce vai ter que "degenerar" os triangulos...
Entao considere um quadrilatero do tipo ABCB (ou seja, tome D=B), com,
digamos, AC=BC=2. Note que o perimetro eh 8, enquando AC=4 e BB=0, ou seja,
a soma das diagonais eh 4.

Mas alguns diriam que isso nao eh um quadrilatero convexo (bom, depende da
sua definicao de quadrilatero!)... Entao se "quadrilateros" nao incluem
casos degenrados, para fazer isso ficar rigoroso, voce teria que tomar um
quadrilatero convexo QUASE degenerado de perimetro 8 (um losango serve, a
conta fica facil), e mostrar que a soma das diagonais fica tao perto de 4
quanto voce queira.

Abraco, Ralph.

On Thu, Jan 23, 2020 at 7:24 AM gilberto azevedo 
wrote:

> Tentei com o retângulo e o quadrado, porém não obtive a resposta...  O
> gabarito é 4.
>
> Em sáb, 11 de jan de 2020 12:03, Bernardo Freitas Paulo da Costa <
> bernardo...@gmail.com> escreveu:
>
>> On Sat, Jan 11, 2020 at 11:24 AM gilberto azevedo 
>> wrote:
>> >
>> > Qual o ínfimo sobre todos os quadriláteros convexos com perímetro 8
>> da soma dos comprimentos de suas diagonais ?
>>
>> Quais são os quadriláteros que você tentaria?
>> --
>> Bernardo Freitas Paulo da Costa
>>
>> --
>> Esta mensagem foi verificada pelo sistema de antivírus e
>>  acredita-se estar livre de perigo.
>>
>>
>> =
>> Instruções para entrar na lista, sair da lista e usar a lista em
>> http://www.mat.puc-rio.br/~obmlistas/obm-l.html
>> =
>>
>
> --
> Esta mensagem foi verificada pelo sistema de antivírus e
> acredita-se estar livre de perigo.

-- 
Esta mensagem foi verificada pelo sistema de antiv�rus e
 acredita-se estar livre de perigo.



[obm-l] Re: [obm-l] Re: [obm-l] Re: [obm-l] Re: [obm-l] Re: [obm-l] Re: [obm-l] Questão OBM - U

2020-01-23 Por tôpico Esdras Muniz
É fácil ver que esse ínfimo tem que ser no mínimo 4, basta fazer
desigualdade triângulos com os triângulos que têm dois vértices comuns com
o quadrilátero e o terceiro sendo a interseção das diagonais. E por esse
argumento do Caio, vemos que é 4 mesmo.

Em qui, 23 de jan de 2020 08:59, Caio Costa  escreveu:

> Minimiza-se a soma das diagonais ao tomar-se um losango degenerado, com
> uma diagonal valendo 4 e outra valendo 0.
>
> Em qui, 23 de jan de 2020 08:34, gilberto azevedo 
> escreveu:
>
>> Pensei em minimizar √(a² + (4-a)²)
>> 4 - a, devido ao fato do perímetro ser 8.
>> No caso obtenho o mínimo sendo 2√2, quando o retângulo é um quadrado de
>> lado 2.
>> A soma das diagonais seria no caso 4√2, e não bate com o gabarito.
>>
>> Em qui, 23 de jan de 2020 08:20, Bernardo Freitas Paulo da Costa <
>> bernardo...@gmail.com> escreveu:
>>
>>> On Thu, Jan 23, 2020 at 7:24 AM gilberto azevedo 
>>> wrote:
>>> >> On Sat, Jan 11, 2020 at 11:24 AM gilberto azevedo <
>>> gil159...@gmail.com> wrote:
>>> >> >
>>> >> > Qual o ínfimo sobre todos os quadriláteros convexos com
>>> perímetro 8 da soma dos comprimentos de suas diagonais ?
>>> >
>>> > Tentei com o retângulo e o quadrado, porém não obtive a resposta...  O
>>> gabarito é 4.
>>>
>>> Qual (ou quais?) retângulo(s) você testou??  Que resposta você obteve?
>>> --
>>> Bernardo Freitas Paulo da Costa
>>>
>>> --
>>> Esta mensagem foi verificada pelo sistema de antivírus e
>>>  acredita-se estar livre de perigo.
>>>
>>>
>>> =
>>> Instru�ões para entrar na lista, sair da lista e usar a lista em
>>> http://www.mat.puc-rio.br/~obmlistas/obm-l.html
>>> =
>>>
>>
>> --
>> Esta mensagem foi verificada pelo sistema de antivírus e
>> acredita-se estar livre de perigo.
>
>
> --
> Esta mensagem foi verificada pelo sistema de antivírus e
> acredita-se estar livre de perigo.

-- 
Esta mensagem foi verificada pelo sistema de antiv�rus e
 acredita-se estar livre de perigo.



[obm-l] Re: [obm-l] Re: [obm-l] Re: [obm-l] Re: [obm-l] Re: [obm-l] Questão OBM - U

2020-01-23 Por tôpico Caio Costa
Minimiza-se a soma das diagonais ao tomar-se um losango degenerado, com uma
diagonal valendo 4 e outra valendo 0.

Em qui, 23 de jan de 2020 08:34, gilberto azevedo 
escreveu:

> Pensei em minimizar √(a² + (4-a)²)
> 4 - a, devido ao fato do perímetro ser 8.
> No caso obtenho o mínimo sendo 2√2, quando o retângulo é um quadrado de
> lado 2.
> A soma das diagonais seria no caso 4√2, e não bate com o gabarito.
>
> Em qui, 23 de jan de 2020 08:20, Bernardo Freitas Paulo da Costa <
> bernardo...@gmail.com> escreveu:
>
>> On Thu, Jan 23, 2020 at 7:24 AM gilberto azevedo 
>> wrote:
>> >> On Sat, Jan 11, 2020 at 11:24 AM gilberto azevedo 
>> wrote:
>> >> >
>> >> > Qual o ínfimo sobre todos os quadriláteros convexos com perímetro
>> 8 da soma dos comprimentos de suas diagonais ?
>> >
>> > Tentei com o retângulo e o quadrado, porém não obtive a resposta...  O
>> gabarito é 4.
>>
>> Qual (ou quais?) retângulo(s) você testou??  Que resposta você obteve?
>> --
>> Bernardo Freitas Paulo da Costa
>>
>> --
>> Esta mensagem foi verificada pelo sistema de antivírus e
>>  acredita-se estar livre de perigo.
>>
>>
>> =
>> Instru�ões para entrar na lista, sair da lista e usar a lista em
>> http://www.mat.puc-rio.br/~obmlistas/obm-l.html
>> =
>>
>
> --
> Esta mensagem foi verificada pelo sistema de antivírus e
> acredita-se estar livre de perigo.

-- 
Esta mensagem foi verificada pelo sistema de antiv�rus e
 acredita-se estar livre de perigo.



[obm-l] Re: [obm-l] Re: [obm-l] Re: [obm-l] Re: [obm-l] Questão OBM - U

2020-01-23 Por tôpico gilberto azevedo
Pensei em minimizar √(a² + (4-a)²)
4 - a, devido ao fato do perímetro ser 8.
No caso obtenho o mínimo sendo 2√2, quando o retângulo é um quadrado de
lado 2.
A soma das diagonais seria no caso 4√2, e não bate com o gabarito.

Em qui, 23 de jan de 2020 08:20, Bernardo Freitas Paulo da Costa <
bernardo...@gmail.com> escreveu:

> On Thu, Jan 23, 2020 at 7:24 AM gilberto azevedo 
> wrote:
> >> On Sat, Jan 11, 2020 at 11:24 AM gilberto azevedo 
> wrote:
> >> >
> >> > Qual o ínfimo sobre todos os quadriláteros convexos com perímetro
> 8 da soma dos comprimentos de suas diagonais ?
> >
> > Tentei com o retângulo e o quadrado, porém não obtive a resposta...  O
> gabarito é 4.
>
> Qual (ou quais?) retângulo(s) você testou??  Que resposta você obteve?
> --
> Bernardo Freitas Paulo da Costa
>
> --
> Esta mensagem foi verificada pelo sistema de antivírus e
>  acredita-se estar livre de perigo.
>
>
> =
> Instru�ões para entrar na lista, sair da lista e usar a lista em
> http://www.mat.puc-rio.br/~obmlistas/obm-l.html
> =
>

-- 
Esta mensagem foi verificada pelo sistema de antiv�rus e
 acredita-se estar livre de perigo.



[obm-l] Re: [obm-l] Re: [obm-l] Re: [obm-l] Questão OBM - U

2020-01-23 Por tôpico Bernardo Freitas Paulo da Costa
On Thu, Jan 23, 2020 at 7:24 AM gilberto azevedo  wrote:
>> On Sat, Jan 11, 2020 at 11:24 AM gilberto azevedo  
>> wrote:
>> >
>> > Qual o ínfimo sobre todos os quadriláteros convexos com perímetro 8 da 
>> > soma dos comprimentos de suas diagonais ?
>
> Tentei com o retângulo e o quadrado, porém não obtive a resposta...  O 
> gabarito é 4.

Qual (ou quais?) retângulo(s) você testou??  Que resposta você obteve?
-- 
Bernardo Freitas Paulo da Costa

-- 
Esta mensagem foi verificada pelo sistema de antiv�rus e
 acredita-se estar livre de perigo.


=
Instru��es para entrar na lista, sair da lista e usar a lista em
http://www.mat.puc-rio.br/~obmlistas/obm-l.html
=


[obm-l] Re: [obm-l] Re: [obm-l] Questão OBM - U

2020-01-23 Por tôpico gilberto azevedo
Tentei com o retângulo e o quadrado, porém não obtive a resposta...  O
gabarito é 4.

Em sáb, 11 de jan de 2020 12:03, Bernardo Freitas Paulo da Costa <
bernardo...@gmail.com> escreveu:

> On Sat, Jan 11, 2020 at 11:24 AM gilberto azevedo 
> wrote:
> >
> > Qual o ínfimo sobre todos os quadriláteros convexos com perímetro 8
> da soma dos comprimentos de suas diagonais ?
>
> Quais são os quadriláteros que você tentaria?
> --
> Bernardo Freitas Paulo da Costa
>
> --
> Esta mensagem foi verificada pelo sistema de antivírus e
>  acredita-se estar livre de perigo.
>
>
> =
> Instruções para entrar na lista, sair da lista e usar a lista em
> http://www.mat.puc-rio.br/~obmlistas/obm-l.html
> =
>

-- 
Esta mensagem foi verificada pelo sistema de antiv�rus e
 acredita-se estar livre de perigo.



[obm-l] Re: [obm-l] Questão OBM - U

2020-01-11 Por tôpico Bernardo Freitas Paulo da Costa
On Sat, Jan 11, 2020 at 11:24 AM gilberto azevedo  wrote:
>
> Qual o ínfimo sobre todos os quadriláteros convexos com perímetro 8 da soma 
> dos comprimentos de suas diagonais ?

Quais são os quadriláteros que você tentaria?
-- 
Bernardo Freitas Paulo da Costa

-- 
Esta mensagem foi verificada pelo sistema de antiv�rus e
 acredita-se estar livre de perigo.


=
Instru��es para entrar na lista, sair da lista e usar a lista em
http://www.mat.puc-rio.br/~obmlistas/obm-l.html
=


[obm-l] Re: [obm-l] Questão sobre equações funcionais

2019-07-28 Por tôpico Esdras Muniz
Errei, satisfaz sim :)

Em dom, 28 de jul de 2019 14:21, Esdras Muniz 
escreveu:

> Mas essa função que VC achou não satisfaz a igualdade.
>
> Em dom, 28 de jul de 2019 01:05, Carlos Monteiro <
> cacacarlosalberto1...@gmail.com> escreveu:
>
>> (Questão) Encontre todas as funções f : R-> R tais que
>>  f(xy - f(x)) = x.f(y)
>>
>> Minha tentativa, não sei se está correta:
>> I) p(x, f(x)/(x-1)): f( f(x)/(x-1) ) = 0; x diferente de 1
>>
>> II) Seja c um número real tal que f(c)=0
>>   i) fazendo x=c na equação encontrada em I: *c diferente de 1*
>> f(0)=0
>>
>>  Se f(0)=0
>> p(x, f(x)/x): 0=x.f( f(x)/x ); x diferente de 0 .: f(x)/x=0
>>  f(x)=0, para todo x real.
>>
>>ii) p(c,1) na equação dada no problema:
>>   0=c.f(1) <=> c=0(caso anterior) ou f(1)=0
>>
>>   Se f(1)=0, 1 é raíz única
>>f( f(x)/(x-1) )=0 .: f(x)/(x-1) = 1 => f(x)=x-1, para todo x real.
>>
>>
>>
>>
>> --
>> Esta mensagem foi verificada pelo sistema de antivírus e
>> acredita-se estar livre de perigo.
>
>

-- 
Esta mensagem foi verificada pelo sistema de antiv�rus e
 acredita-se estar livre de perigo.



[obm-l] Re: [obm-l] Questão sobre equações funcionais

2019-07-28 Por tôpico Esdras Muniz
Mas essa função que VC achou não satisfaz a igualdade.

Em dom, 28 de jul de 2019 01:05, Carlos Monteiro <
cacacarlosalberto1...@gmail.com> escreveu:

> (Questão) Encontre todas as funções f : R-> R tais que
>  f(xy - f(x)) = x.f(y)
>
> Minha tentativa, não sei se está correta:
> I) p(x, f(x)/(x-1)): f( f(x)/(x-1) ) = 0; x diferente de 1
>
> II) Seja c um número real tal que f(c)=0
>   i) fazendo x=c na equação encontrada em I: *c diferente de 1*
> f(0)=0
>
>  Se f(0)=0
> p(x, f(x)/x): 0=x.f( f(x)/x ); x diferente de 0 .: f(x)/x=0
>  f(x)=0, para todo x real.
>
>ii) p(c,1) na equação dada no problema:
>   0=c.f(1) <=> c=0(caso anterior) ou f(1)=0
>
>   Se f(1)=0, 1 é raíz única
>f( f(x)/(x-1) )=0 .: f(x)/(x-1) = 1 => f(x)=x-1, para todo x real.
>
>
>
>
> --
> Esta mensagem foi verificada pelo sistema de antivírus e
> acredita-se estar livre de perigo.

-- 
Esta mensagem foi verificada pelo sistema de antiv�rus e
 acredita-se estar livre de perigo.



[obm-l] Re: [obm-l] Re: [obm-l] Questão de probabilidade

2018-11-06 Por tôpico Vanderlei Nemitz
Bela solução, Bruno!
Muito obrigado!

Em ter, 6 de nov de 2018 15:38, Bruno Visnadi  Seja Pa a probabilidade de ocorrência de a. Defina Pb e Pc analogamente.
> a = Pa(1-Pb)(1-Pc)
> b = Pb(1-Pa)(1-Pc)
> c = Pc(1-Pa)(1-Pb)
> p = (1-Pa)(1-Pb)(1-Pc)
> Queremos achar a razão Pa/Pc
> Da equação (a - 2b)p = ab, obtemos:
> (1-Pa)(1-Pb)(1-Pc)²(Pa(1-Pb) - 2Pb(1-Pa)) = PaPb(1-Pa)(1-Pb)(1-Pc)²
> Pa(1-Pb) - 2Pb(1-Pa)  = PaPb
> Pa - 2Pb + PaPb = PaPb
> Pa = 2Pb -> Pb = Pa/2
> Da equação  (b - 3c)p = 2bc, obtemos:
> (1-Pa)²(1-Pb)(1-Pc)(Pb(1-Pc) - 3Pc(1-Pb)) = 2PbPc(1-Pa)²(1-Pb)(1-Pc)
> Pb(1-Pc) - 3Pc(1-Pb) = 2PbPc
> Pb - 3Pc + 2PcPb = 2PbPc
> Pb = 3Pc
> Logo: Pa/2 = 3Pc
> Pa/Pc = 6
>
>
>
>
>
> Em ter, 6 de nov de 2018 às 12:43, Vanderlei Nemitz 
> escreveu:
>
>> Pessoal, alguém tem um ideia de como resolver a seguinte questão? Já
>> tentei muita coisa, sem sucesso.
>> Muito obrigado!
>>
>> Vanderlei
>>
>> Sejam três eventos independentes A, B e C. A probabilidade de que ocorra
>> apenas o evento A é a, apenas o evento B é b e apenas o evento C é c. Seja
>> p a probabilidade de que nenhum dos eventos A, B ou C ocorra. Sabendo que
>> todas as probabilidades citadas são números no intervalo ]0, 1[ e que p
>> satisfaz as equações (a - 2b).p = ab e (b - 3c).p = 2bc, a razão entre a
>> probabilidade de ocorrência de A e a probabilidade de ocorrência de C é:
>> a) 12
>> b) 3
>> c) 10
>> d) 5
>> e) 6
>>
>> --
>> Esta mensagem foi verificada pelo sistema de antivírus e
>> acredita-se estar livre de perigo.
>
>
> --
> Esta mensagem foi verificada pelo sistema de antivírus e
> acredita-se estar livre de perigo.

-- 
Esta mensagem foi verificada pelo sistema de antiv�rus e
 acredita-se estar livre de perigo.



[obm-l] Re: [obm-l] Questão de probabilidade

2018-11-06 Por tôpico Bruno Visnadi
Seja Pa a probabilidade de ocorrência de a. Defina Pb e Pc analogamente.
a = Pa(1-Pb)(1-Pc)
b = Pb(1-Pa)(1-Pc)
c = Pc(1-Pa)(1-Pb)
p = (1-Pa)(1-Pb)(1-Pc)
Queremos achar a razão Pa/Pc
Da equação (a - 2b)p = ab, obtemos:
(1-Pa)(1-Pb)(1-Pc)²(Pa(1-Pb) - 2Pb(1-Pa)) = PaPb(1-Pa)(1-Pb)(1-Pc)²
Pa(1-Pb) - 2Pb(1-Pa)  = PaPb
Pa - 2Pb + PaPb = PaPb
Pa = 2Pb -> Pb = Pa/2
Da equação  (b - 3c)p = 2bc, obtemos:
(1-Pa)²(1-Pb)(1-Pc)(Pb(1-Pc) - 3Pc(1-Pb)) = 2PbPc(1-Pa)²(1-Pb)(1-Pc)
Pb(1-Pc) - 3Pc(1-Pb) = 2PbPc
Pb - 3Pc + 2PcPb = 2PbPc
Pb = 3Pc
Logo: Pa/2 = 3Pc
Pa/Pc = 6





Em ter, 6 de nov de 2018 às 12:43, Vanderlei Nemitz 
escreveu:

> Pessoal, alguém tem um ideia de como resolver a seguinte questão? Já
> tentei muita coisa, sem sucesso.
> Muito obrigado!
>
> Vanderlei
>
> Sejam três eventos independentes A, B e C. A probabilidade de que ocorra
> apenas o evento A é a, apenas o evento B é b e apenas o evento C é c. Seja
> p a probabilidade de que nenhum dos eventos A, B ou C ocorra. Sabendo que
> todas as probabilidades citadas são números no intervalo ]0, 1[ e que p
> satisfaz as equações (a - 2b).p = ab e (b - 3c).p = 2bc, a razão entre a
> probabilidade de ocorrência de A e a probabilidade de ocorrência de C é:
> a) 12
> b) 3
> c) 10
> d) 5
> e) 6
>
> --
> Esta mensagem foi verificada pelo sistema de antivírus e
> acredita-se estar livre de perigo.

-- 
Esta mensagem foi verificada pelo sistema de antiv�rus e
 acredita-se estar livre de perigo.



[obm-l] Re: [obm-l] Re: [obm-l] Questão do ITA

2018-10-11 Por tôpico Vanderlei Nemitz
Valeu, Ralph!
Como sempre, uma explicação clara e simples!

Em qua, 10 de out de 2018 17:05, Ralph Teixeira 
escreveu:

> Note que x=5 é um possível valor que resolve aquela equação (mas,
> sinceramente, não interessa, eu faria o raciocínio abaixo com qualquer
> número).
>
> Então qualquer polinômio que satisfaça f(1)=5, f(-1)=10 e f(0)=20
> automaticamente satisfaz todas as condições do enunciado (note que
> a_0=f(0)). Em outras palavras, qualquer polinômio cujo gráfico passe pelos
> pontos (-1,10),(0,20),(1,5) serve.
>
> Agora escolha um ponto (z,0) qualquer como 4o ponto (onde z não é -1, 0
> nem 1). Como quaisquer 4 pontos (com "x"s diferentes) determinam um único
> polinômio de grau 3, haverá um polinômio de grau 3 que passa pelos pontos
> dados e que tem raiz z. Como z pode ser negativo, positivo, raiz(2), ou 42,
> nenhuma das respostas (A)-(D) pode valer (respectivamente!). Então tem que
> ser (E).
>
> Abraço, Ralph.
>
> On Wed, Oct 10, 2018 at 5:41 AM Vanderlei Nemitz 
> wrote:
>
>> Bom dia, pessoal!
>> Encontrei essa questão, que diz ser do ITA (eu particularmente não
>> encontrei na internet).
>> Como a resposta é E (nenhuma das anteriores), não sei se é possível
>> provar que as anteriores são falsas. Eu não consegui concluir coisa alguma.
>>
>> *Seja f(x) = am.x^m + am–1.x^(m–1) + ... + a1.x + a0, onde am, am–1, ...,
>> a1, a0 são reais, am diferente de 0 e a0 diferente de 0. Se f(1) é solução
>> real da equação 2^(x–3) + 2^(x–4) = 2^(x–2) – 2^(x–1) + 14, f(–1) = 2.f(1)
>> e a0 = 2.f(–1), então podemos afirmar:*
>>
>> *a) f(x) tem somente raízes reais positivas.*
>>
>> *b) f(x) tem somente raízes reais negativas.*
>>
>> *c) f(x) tem somente raízes reais inteiras.*
>>
>> *d) f(x) não tem raízes reais inteiras.*
>>
>> *e) nda*
>> Alguém tem alguma ideia?
>> Muito obrigado!
>>
>> --
>> Esta mensagem foi verificada pelo sistema de antivírus e
>> acredita-se estar livre de perigo.
>
>
> --
> Esta mensagem foi verificada pelo sistema de antivírus e
> acredita-se estar livre de perigo.

-- 
Esta mensagem foi verificada pelo sistema de antiv�rus e
 acredita-se estar livre de perigo.



[obm-l] Re: [obm-l] Questão do ITA

2018-10-10 Por tôpico Ralph Teixeira
Note que x=5 é um possível valor que resolve aquela equação (mas,
sinceramente, não interessa, eu faria o raciocínio abaixo com qualquer
número).

Então qualquer polinômio que satisfaça f(1)=5, f(-1)=10 e f(0)=20
automaticamente satisfaz todas as condições do enunciado (note que
a_0=f(0)). Em outras palavras, qualquer polinômio cujo gráfico passe pelos
pontos (-1,10),(0,20),(1,5) serve.

Agora escolha um ponto (z,0) qualquer como 4o ponto (onde z não é -1, 0 nem
1). Como quaisquer 4 pontos (com "x"s diferentes) determinam um único
polinômio de grau 3, haverá um polinômio de grau 3 que passa pelos pontos
dados e que tem raiz z. Como z pode ser negativo, positivo, raiz(2), ou 42,
nenhuma das respostas (A)-(D) pode valer (respectivamente!). Então tem que
ser (E).

Abraço, Ralph.

On Wed, Oct 10, 2018 at 5:41 AM Vanderlei Nemitz 
wrote:

> Bom dia, pessoal!
> Encontrei essa questão, que diz ser do ITA (eu particularmente não
> encontrei na internet).
> Como a resposta é E (nenhuma das anteriores), não sei se é possível provar
> que as anteriores são falsas. Eu não consegui concluir coisa alguma.
>
> *Seja f(x) = am.x^m + am–1.x^(m–1) + ... + a1.x + a0, onde am, am–1, ...,
> a1, a0 são reais, am diferente de 0 e a0 diferente de 0. Se f(1) é solução
> real da equação 2^(x–3) + 2^(x–4) = 2^(x–2) – 2^(x–1) + 14, f(–1) = 2.f(1)
> e a0 = 2.f(–1), então podemos afirmar:*
>
> *a) f(x) tem somente raízes reais positivas.*
>
> *b) f(x) tem somente raízes reais negativas.*
>
> *c) f(x) tem somente raízes reais inteiras.*
>
> *d) f(x) não tem raízes reais inteiras.*
>
> *e) nda*
> Alguém tem alguma ideia?
> Muito obrigado!
>
> --
> Esta mensagem foi verificada pelo sistema de antivírus e
> acredita-se estar livre de perigo.

-- 
Esta mensagem foi verificada pelo sistema de antiv�rus e
 acredita-se estar livre de perigo.



[obm-l] Re: [obm-l] Questão do Gandhi

2018-08-15 Por tôpico gilberto azevedo
Só uma ressalva, alí depois do "ou a+1 será par, e a ... "
Não tem esse "a" no final, erro de digitação.

Em Qua, 15 de ago de 2018 18:02, gilberto azevedo 
escreveu:

> Supondo que b>a, então b = a+1
> Logo :
> D = a² + (a+1)² + (a*(a+1))²
> D = a² + a² + 2a + 1 + (a²+a)²
> D = 2a² + 2a + 1 + (a²+a)²
> D = 2(a²+a) + 1 + (a²+a)²
> D = (a²+a)² + 2(a²+a) + 1 (só organizei)
> Agora a sacada é perceber que está na forma x²+2xy+y² sendo x = a²+a e y =
> 1
> Logo :
> D = (a²+a+1)²
> √D = a²+a+1
> √D = a(a+1) + 1
> Agora basta analisar que :
> a(a+1) é sempre par pois ou a ou a+1 será par, e a somando com 1 irá
> formar um número ímpar.
> Assim a raíz é inteira e é sempre ímpar !
> Espero ter ajudado. Abs.
>
>
> Em 15 de ago de 2018 17:30, "Daniel Quevedo" 
> escreveu:
>
> Seja D = a^2 + b^2 + c^2, onde a e b são inteiros consecutivos e c = a•b.
> Então sobre a raiz quadrada de D podemos afirmar que:
>
> A) é sempre inteiro par
> B) algumas vezes é inteiro par, outras vezes não.
> C) algumas vezes é racional, outras vezes não.
> D) é sempre inteiro ímpar.
> E) é sempre irracional.
>
> Gab: d
>
> PS: é fácil mostrar q D é inteiro ímpar, minha dificuldade está em mostrar
> q a raiz quadrada tbm é.
> --
> Fiscal: Daniel Quevedo
>
> --
> Esta mensagem foi verificada pelo sistema de antivírus e
> acredita-se estar livre de perigo.
>
>
>

-- 
Esta mensagem foi verificada pelo sistema de antiv�rus e
 acredita-se estar livre de perigo.



[obm-l] Re: [obm-l] Questão do Gandhi

2018-08-15 Por tôpico Claudio Buffara
D = a^2 + (a+1)^2 + a^2*(a+1)^2 = a^4 + 2a^3 + 3a^2 + 2a + 1.

Se D for um quadrado, então será da forma (a^2 + a + x)^2.

Expandindo isso e comparando coeficientes, obtemos x = 1 ==> D = (a^2 + a +
1)^2.

Como a^2 + a é par, raiz(D) = a^2 + a + 1 é ímpar.

[]s,
Claudio.


2018-08-15 17:22 GMT-03:00 Daniel Quevedo :

> Seja D = a^2 + b^2 + c^2, onde a e b são inteiros consecutivos e c = a•b.
> Então sobre a raiz quadrada de D podemos afirmar que:
>
> A) é sempre inteiro par
> B) algumas vezes é inteiro par, outras vezes não.
> C) algumas vezes é racional, outras vezes não.
> D) é sempre inteiro ímpar.
> E) é sempre irracional.
>
> Gab: d
>
> PS: é fácil mostrar q D é inteiro ímpar, minha dificuldade está em mostrar
> q a raiz quadrada tbm é.
> --
> Fiscal: Daniel Quevedo
>
> --
> Esta mensagem foi verificada pelo sistema de antivírus e
> acredita-se estar livre de perigo.

-- 
Esta mensagem foi verificada pelo sistema de antiv�rus e
 acredita-se estar livre de perigo.



[obm-l] Re: [obm-l] Questão do Gandhi

2018-08-15 Por tôpico gilberto azevedo
Supondo que b>a, então b = a+1
Logo :
D = a² + (a+1)² + (a*(a+1))²
D = a² + a² + 2a + 1 + (a²+a)²
D = 2a² + 2a + 1 + (a²+a)²
D = 2(a²+a) + 1 + (a²+a)²
D = (a²+a)² + 2(a²+a) + 1 (só organizei)
Agora a sacada é perceber que está na forma x²+2xy+y² sendo x = a²+a e y = 1
Logo :
D = (a²+a+1)²
√D = a²+a+1
√D = a(a+1) + 1
Agora basta analisar que :
a(a+1) é sempre par pois ou a ou a+1 será par, e a somando com 1 irá formar
um número ímpar.
Assim a raíz é inteira e é sempre ímpar !
Espero ter ajudado. Abs.


Em 15 de ago de 2018 17:30, "Daniel Quevedo"  escreveu:

Seja D = a^2 + b^2 + c^2, onde a e b são inteiros consecutivos e c = a•b.
Então sobre a raiz quadrada de D podemos afirmar que:

A) é sempre inteiro par
B) algumas vezes é inteiro par, outras vezes não.
C) algumas vezes é racional, outras vezes não.
D) é sempre inteiro ímpar.
E) é sempre irracional.

Gab: d

PS: é fácil mostrar q D é inteiro ímpar, minha dificuldade está em mostrar
q a raiz quadrada tbm é.
-- 
Fiscal: Daniel Quevedo

-- 
Esta mensagem foi verificada pelo sistema de antivírus e
acredita-se estar livre de perigo.

-- 
Esta mensagem foi verificada pelo sistema de antiv�rus e
 acredita-se estar livre de perigo.



[obm-l] Re: [obm-l] Re: [obm-l] Questão do IME

2018-07-14 Por tôpico Vanderlei Nemitz
Muito obrigado, Claudio!
Bela solução!

Em 13 de julho de 2018 13:35, Claudio Buffara 
escreveu:

> Os prolongamentos de DM e EN se intersectam num mesmo ponto P pertencente
> a AB.
> Pra ver isso, repare que os triângulos DCM e PAM são semelhantes (razão de
> semelhança = 2).
> Idem para os triângulos EFN e PNB.
> Como, no triângulo PDE (que é isósceles), vale PM/PD = PN/PE = 1/3,
> concluímos que MN é paralelo a DE.
>
> []s,
> Claudio.
>
>
> 2018-07-13 12:13 GMT-03:00 Vanderlei Nemitz :
>
>> Sejam dois quadrados ABCD e ABEF, tendo um lado comum AB, mas não
>> situados num mesmo plano. Sejam M e N pertencentes, respectivamente, às
>> diagonais AC e BF tais que AM/AC = BN/BF = 1/3. Mostre que MN é paralelo a
>> DE.
>>
>> Alguém poderia ajudar?
>> Obrigado,
>> Vanderlei
>>
>> --
>> Esta mensagem foi verificada pelo sistema de antivírus e
>> acredita-se estar livre de perigo.
>
>
>
> --
> Esta mensagem foi verificada pelo sistema de antivírus e
> acredita-se estar livre de perigo.

-- 
Esta mensagem foi verificada pelo sistema de antiv�rus e
 acredita-se estar livre de perigo.



Re: [obm-l] Re: [obm-l] Questão do IME

2018-07-14 Por tôpico wagner

Brilhante!


Quoting Claudio Buffara :


Os prolongamentos de DM e EN se intersectam num mesmo ponto P pertencente a
AB.
Pra ver isso, repare que os triângulos DCM e PAM são semelhantes (razão de
semelhança = 2).
Idem para os triângulos EFN e PNB.
Como, no triângulo PDE (que é isósceles), vale PM/PD = PN/PE = 1/3,
concluímos que MN é paralelo a DE.

[]s,
Claudio.


2018-07-13 12:13 GMT-03:00 Vanderlei Nemitz :


Sejam dois quadrados ABCD e ABEF, tendo um lado comum AB, mas não situados
num mesmo plano. Sejam M e N pertencentes, respectivamente, às diagonais AC
e BF tais que AM/AC = BN/BF = 1/3. Mostre que MN é paralelo a DE.

Alguém poderia ajudar?
Obrigado,
Vanderlei

--
Esta mensagem foi verificada pelo sistema de antivírus e
acredita-se estar livre de perigo.


--
Esta mensagem foi verificada pelo sistema de antiv?rus e
 acredita-se estar livre de perigo.





--
Esta mensagem foi verificada pelo sistema de antiv�rus e
acredita-se estar livre de perigo.

=
Instru��es para entrar na lista, sair da lista e usar a lista em
http://www.mat.puc-rio.br/~obmlistas/obm-l.html
=


[obm-l] Re: [obm-l] Questão do IME

2018-07-13 Por tôpico Claudio Buffara
Os prolongamentos de DM e EN se intersectam num mesmo ponto P pertencente a
AB.
Pra ver isso, repare que os triângulos DCM e PAM são semelhantes (razão de
semelhança = 2).
Idem para os triângulos EFN e PNB.
Como, no triângulo PDE (que é isósceles), vale PM/PD = PN/PE = 1/3,
concluímos que MN é paralelo a DE.

[]s,
Claudio.


2018-07-13 12:13 GMT-03:00 Vanderlei Nemitz :

> Sejam dois quadrados ABCD e ABEF, tendo um lado comum AB, mas não situados
> num mesmo plano. Sejam M e N pertencentes, respectivamente, às diagonais AC
> e BF tais que AM/AC = BN/BF = 1/3. Mostre que MN é paralelo a DE.
>
> Alguém poderia ajudar?
> Obrigado,
> Vanderlei
>
> --
> Esta mensagem foi verificada pelo sistema de antivírus e
> acredita-se estar livre de perigo.

-- 
Esta mensagem foi verificada pelo sistema de antiv�rus e
 acredita-se estar livre de perigo.



[obm-l] Re: [obm-l] Questão sobre divisor primo

2018-06-06 Por tôpico Otávio Araújo
Só uma curiosidade: de onde é essa questão?

Em qua, 6 de jun de 2018 11:38, Pedro Chaves 
escreveu:

> Caros Colegas,
>
> Não consegui resolver a questão abaixo. Peço auxílio.
>
> Questão:Para cada inteiro positivo n, mostrar que todo divisor primo
> de 12n^2 + 1 é da forma 6k +1, sendo k um inteiro positivo.
>
>
>
>
> 
>  Livre
> de vírus. www.avast.com
> .
> <#m_-594041030565945_DAB4FAD8-2DD7-40BB-A1B8-4E2AA1F9FDF2>
>
> --
> Esta mensagem foi verificada pelo sistema de antivírus e
> acredita-se estar livre de perigo.
>

-- 
Esta mensagem foi verificada pelo sistema de antiv�rus e
 acredita-se estar livre de perigo.



[obm-l] Re: [obm-l] Questão sobre divisor primo

2018-06-06 Por tôpico Otávio Araújo
Só um detalhe que errei na digitação:
1 = (1/p) = (x.12/p) = ((-x).(-12)/p)=
(-x/p)(-12/p)

Em qua, 6 de jun de 2018 15:55, Otávio Araújo 
escreveu:

> Tenho uma solução aqui:
> Seja p um primo que divide 12n^2 +1, teremos que 12n^2 = -1 mód p. Seja x
> o inverso multiplicativo de 12 módulo p em (Zp)*, então n^2= -x mód p,
> portanto
> -x é resíduo quadrático módulo p. Denote (/) o simbolo de Legendre,
> teremos (-x/p)=1, mas 1=(1/p)=(x.12/p)
> (-x/p)(-12/p) --> (-12/p)=1 -->
> (-1/p)(4/p)(3/p)=1 --> (-1/p)(3/p)=1, ou seja, (-1/p)=(3/p)=1 ou
> (-1/p)=(3/p)=-1.
> Em qualquer livro de teoria dos números que contenha o assunto de resíduos
> quadráticos podemos ver a demonstração de que
> (-1/p)=1 se p=1 mód 4 e (-1/p)=-1 se
>  p=3 mód 4.
> Do teorema da reciprocidade quadrática (p é diferente de 2 e 3), temos
> (p/3)(3/p)= (-1)^((p-1)/2) -->
> Mas p é congruente a 1,5 ,7 ou 11 módulo 12 ( pois é primo e não é 2 ou
> 3), testando cada caso, temos (p/3)=1 se p=1 ou 7 módulo 12 (pois devemos
> ter p=1 mód 3) e (p/3)=-1 se p=5 ou 11 módulo 12.  Observando que
> (-1)^((p-1)/2)=1 se p=1 ou 5 módulo 12 e -1 se p=7 ou 11 módulo 12,
> obteremos que (3/p)=1 se p =1 ou 11 módulo 12 e -1 se p =5 ou 7 módulo 12.
>
> Por último, se (-1/p)=(3/p)=1, teremos
> p=1 mód 4 e p= 1 ou 11 módulo 12
> --> p=1 mód 12.
> Se (-1/p)=(3/p)=-1, teremos
> p=3 mód 4 e p=5 ou 7 módulo 12
> --> p=7 mód 12.
> Daí p = 1 ou 7 módulo 12 --> p=1 mód 6, como queríamos.
>
> Em qua, 6 de jun de 2018 11:38, Pedro Chaves 
> escreveu:
>
>> Caros Colegas,
>>
>> Não consegui resolver a questão abaixo. Peço auxílio.
>>
>> Questão:Para cada inteiro positivo n, mostrar que todo divisor primo
>> de 12n^2 + 1 é da forma 6k +1, sendo k um inteiro positivo.
>>
>>
>>
>>
>> 
>>  Livre
>> de vírus. www.avast.com
>> .
>>
>> <#m_3212190965219518123_m_8119351896224020777_m_-594041030565945_DAB4FAD8-2DD7-40BB-A1B8-4E2AA1F9FDF2>
>>
>> --
>> Esta mensagem foi verificada pelo sistema de antivírus e
>> acredita-se estar livre de perigo.
>>
>

-- 
Esta mensagem foi verificada pelo sistema de antiv�rus e
 acredita-se estar livre de perigo.



[obm-l] Re: [obm-l] Questão sobre divisor primo

2018-06-06 Por tôpico Otávio Araújo
Tenho uma solução aqui:
Seja p um primo que divide 12n^2 +1, teremos que 12n^2 = -1 mód p. Seja x o
inverso multiplicativo de 12 módulo p em (Zp)*, então n^2= -x mód p,
portanto
-x é resíduo quadrático módulo p. Denote (/) o simbolo de Legendre, teremos
(-x/p)=1, mas 1=(1/p)=(x.12/p)
(-x/p)(-12/p) --> (-12/p)=1 -->
(-1/p)(4/p)(3/p)=1 --> (-1/p)(3/p)=1, ou seja, (-1/p)=(3/p)=1 ou
(-1/p)=(3/p)=-1.
Em qualquer livro de teoria dos números que contenha o assunto de resíduos
quadráticos podemos ver a demonstração de que
(-1/p)=1 se p=1 mód 4 e (-1/p)=-1 se
 p=3 mód 4.
Do teorema da reciprocidade quadrática (p é diferente de 2 e 3), temos
(p/3)(3/p)= (-1)^((p-1)/2) -->
Mas p é congruente a 1,5 ,7 ou 11 módulo 12 ( pois é primo e não é 2 ou 3),
testando cada caso, temos (p/3)=1 se p=1 ou 7 módulo 12 (pois devemos ter
p=1 mód 3) e (p/3)=-1 se p=5 ou 11 módulo 12.  Observando que
(-1)^((p-1)/2)=1 se p=1 ou 5 módulo 12 e -1 se p=7 ou 11 módulo 12,
obteremos que (3/p)=1 se p =1 ou 11 módulo 12 e -1 se p =5 ou 7 módulo 12.

Por último, se (-1/p)=(3/p)=1, teremos
p=1 mód 4 e p= 1 ou 11 módulo 12
--> p=1 mód 12.
Se (-1/p)=(3/p)=-1, teremos
p=3 mód 4 e p=5 ou 7 módulo 12
--> p=7 mód 12.
Daí p = 1 ou 7 módulo 12 --> p=1 mód 6, como queríamos.

Em qua, 6 de jun de 2018 11:38, Pedro Chaves 
escreveu:

> Caros Colegas,
>
> Não consegui resolver a questão abaixo. Peço auxílio.
>
> Questão:Para cada inteiro positivo n, mostrar que todo divisor primo
> de 12n^2 + 1 é da forma 6k +1, sendo k um inteiro positivo.
>
>
>
>
> 
>  Livre
> de vírus. www.avast.com
> .
> <#m_-594041030565945_DAB4FAD8-2DD7-40BB-A1B8-4E2AA1F9FDF2>
>
> --
> Esta mensagem foi verificada pelo sistema de antivírus e
> acredita-se estar livre de perigo.
>

-- 
Esta mensagem foi verificada pelo sistema de antiv�rus e
 acredita-se estar livre de perigo.



[obm-l] Re: [obm-l] Re: [obm-l] Questão sobre divisor primo

2018-06-06 Por tôpico Claudio Buffara
Na verdade é pra provar que se p é primo e divide 12n^2 + 1, então p é de
forma 6k+1.



2018-06-06 12:50 GMT-03:00 Daniel Quevedo :

> De uma maneira bem informal 6| 12n^2 , para qqr n inteiro. Logo 12n^2+1= 1
> (mod 6) ou seja é da forma 6k +1.
>
> Uma demonstração formal seria por indução finita, onde P(0)= 12+1 = 13 =
> 1(mod 6)
> Se P(n) é verdade, logo
> P (n +1) = 12n^2 + 24n +12 + 1 = 6(2n^2 + 4n + 3) + 13 = 1 (mod 6) é vdd
> Acho q é isso
>
> Em qua, 6 de jun de 2018 às 11:38, Pedro Chaves 
> escreveu:
>
>> Caros Colegas,
>>
>> Não consegui resolver a questão abaixo. Peço auxílio.
>>
>> Questão:Para cada inteiro positivo n, mostrar que todo divisor primo
>> de 12n^2 + 1 é da forma 6k +1, sendo k um inteiro positivo.
>>
>>
>>
>>
>> 
>>  Livre
>> de vírus. www.avast.com
>> .
>>
>> <#m_-1183774050225021055_m_7705839601006527736_DAB4FAD8-2DD7-40BB-A1B8-4E2AA1F9FDF2>
>>
>> --
>> Esta mensagem foi verificada pelo sistema de antivírus e
>> acredita-se estar livre de perigo.
>>
> --
> Fiscal: Daniel Quevedo
>
> --
> Esta mensagem foi verificada pelo sistema de antivírus e
> acredita-se estar livre de perigo.
>

-- 
Esta mensagem foi verificada pelo sistema de antiv�rus e
 acredita-se estar livre de perigo.



[obm-l] Re: [obm-l] Questão sobre divisor primo

2018-06-06 Por tôpico Daniel Quevedo
De uma maneira bem informal 6| 12n^2 , para qqr n inteiro. Logo 12n^2+1= 1
(mod 6) ou seja é da forma 6k +1.

Uma demonstração formal seria por indução finita, onde P(0)= 12+1 = 13 =
1(mod 6)
Se P(n) é verdade, logo
P (n +1) = 12n^2 + 24n +12 + 1 = 6(2n^2 + 4n + 3) + 13 = 1 (mod 6) é vdd
Acho q é isso

Em qua, 6 de jun de 2018 às 11:38, Pedro Chaves 
escreveu:

> Caros Colegas,
>
> Não consegui resolver a questão abaixo. Peço auxílio.
>
> Questão:Para cada inteiro positivo n, mostrar que todo divisor primo
> de 12n^2 + 1 é da forma 6k +1, sendo k um inteiro positivo.
>
>
>
>
> 
>  Livre
> de vírus. www.avast.com
> .
> <#m_7705839601006527736_DAB4FAD8-2DD7-40BB-A1B8-4E2AA1F9FDF2>
>
> --
> Esta mensagem foi verificada pelo sistema de antivírus e
> acredita-se estar livre de perigo.
>
-- 
Fiscal: Daniel Quevedo

-- 
Esta mensagem foi verificada pelo sistema de antiv�rus e
 acredita-se estar livre de perigo.



[obm-l] Re: [obm-l] Re: [obm-l] Re: [obm-l] Re: [obm-l] Questão de derivada

2018-04-23 Por tôpico Igor Caetano Diniz
na verdade eu não fiz rsrs.

Eu queria ver um modo claro de mostrar. Se não puder usar L'Hospital, acho
que tem que fazer uma sequência por baixo e uma por cima aplicando TVM em
cada intervalo. Aí usa o fato dessa sequencia ser limitada, e monotona,
portanto, convergente. Logo lim f'(xn) = L tanto por cima, quanto por
baixo. como existem esses limites laterais e de mesmo valor, a derivada
existe.

-- 
Esta mensagem foi verificada pelo sistema de antiv�rus e
 acredita-se estar livre de perigo.



[obm-l] Re: [obm-l] Re: [obm-l] Re: [obm-l] Questão de derivada

2018-04-23 Por tôpico Artur Steiner
Eu li errado, temos que lim x --> 0 f'
(x) = L. Assim,  a Regra de l' Hopital conforme mostrei demonstra que, de
fato, f'(c) = L.

Mas o que vc fez não mostra que f'(c) = L.

Artur Costa Steiner

Em Seg, 23 de abr de 2018 14:31, Igor Caetano Diniz 
escreveu:

> Se a questão tivesse um intervalo explícito [a,b] e diferenciável em todo
> ponto (a,b) exceto possivelmente num ponto c em (a,b) tal que lim f '(x) =
> L, x-> c, o que eu fiz estaria correto?
>
> 2018-04-23 14:11 GMT-03:00 Artur Steiner :
>
>> Como f é contínua em 0, então, pela regra de L'Hopital,
>>
>> lim x --> 0+  (f(x) - f0))/(x - 0) = lim x --> 0+ f'(x) = L
>>
>> Pela definição de derivada lateral, o limite do primeiro membro é a
>> derivada à direita de 0. É só o que podemos concluir do enunciado. Nada
>> garante que a derivada à esquerda de 0 sequer exista.
>>
>> Artur Costa Steiner
>>
>> Em Dom, 22 de abr de 2018 22:45, Igor Caetano Diniz <
>> icaetanodi...@gmail.com> escreveu:
>>
>>> Boa noite,
>>> Gostaria de uma ajuda numa questão. Primeiro saber se pensei
>>> corretamente na maneira (1) e se é possível resolver como pensei também na
>>> maneira (2).
>>> Aí vai:
>>> Questão 5.3.8 do livro do Stephen Abbot, Understanding Analysis:
>>>
>>> Assuma que f é contínua em um intervalo que contém o zero e
>>> diferenciável em todo ponto diferente de zero. Se lim f ' (x) = L, x->0,
>>> prove que f ' (0) existe e é igual a L.
>>>
>>> O que pensei em fazer:
>>>
>>> Pensei em duas maneiras.
>>> 1)Se o limite existe em 0, então existem os limites laterais, limite a
>>> esquerda e limite a direita: lim x->0- f ' (x) = L e lim x->0+ f ' (x) = L.
>>> Lema: f ' (c) = lim f(c+h)-f(c-h)/2h = lim [ f(c+h)-f(c) +f(c) - f(c-h)
>>> ]/2h = 1/2 lim x->c-[f(c+h)-f(c)/h] + 1/2 lim x->c+ [f(c+h)-f(c)/h]
>>>
>>> Logo como existem esses limites laterais, existe a derivada em 0, e
>>> portanto, é L
>>>
>>> 2) queria tentar fazer, usando uma sequência xn<0 com limxn = 0 e yn>0
>>> com lim yn = 0 e provar que lim(f(yn)-f(xn)/(yn-xn)) = f'(0) = L. Mas sinto
>>> que isso é verdade e não sei provar
>>>
>>> Abraços
>>>
>>> --
>>> Esta mensagem foi verificada pelo sistema de antivírus e
>>> acredita-se estar livre de perigo.
>>
>>
>> --
>> Esta mensagem foi verificada pelo sistema de antivírus e
>> acredita-se estar livre de perigo.
>>
>
>
> --
> Esta mensagem foi verificada pelo sistema de antivírus e
> acredita-se estar livre de perigo.

-- 
Esta mensagem foi verificada pelo sistema de antiv�rus e
 acredita-se estar livre de perigo.



[obm-l] Re: [obm-l] Re: [obm-l] Questão de derivada

2018-04-23 Por tôpico Igor Caetano Diniz
Se a questão tivesse um intervalo explícito [a,b] e diferenciável em todo
ponto (a,b) exceto possivelmente num ponto c em (a,b) tal que lim f '(x) =
L, x-> c, o que eu fiz estaria correto?

2018-04-23 14:11 GMT-03:00 Artur Steiner :

> Como f é contínua em 0, então, pela regra de L'Hopital,
>
> lim x --> 0+  (f(x) - f0))/(x - 0) = lim x --> 0+ f'(x) = L
>
> Pela definição de derivada lateral, o limite do primeiro membro é a
> derivada à direita de 0. É só o que podemos concluir do enunciado. Nada
> garante que a derivada à esquerda de 0 sequer exista.
>
> Artur Costa Steiner
>
> Em Dom, 22 de abr de 2018 22:45, Igor Caetano Diniz <
> icaetanodi...@gmail.com> escreveu:
>
>> Boa noite,
>> Gostaria de uma ajuda numa questão. Primeiro saber se pensei corretamente
>> na maneira (1) e se é possível resolver como pensei também na maneira (2).
>> Aí vai:
>> Questão 5.3.8 do livro do Stephen Abbot, Understanding Analysis:
>>
>> Assuma que f é contínua em um intervalo que contém o zero e diferenciável
>> em todo ponto diferente de zero. Se lim f ' (x) = L, x->0, prove que f '
>> (0) existe e é igual a L.
>>
>> O que pensei em fazer:
>>
>> Pensei em duas maneiras.
>> 1)Se o limite existe em 0, então existem os limites laterais, limite a
>> esquerda e limite a direita: lim x->0- f ' (x) = L e lim x->0+ f ' (x) = L.
>> Lema: f ' (c) = lim f(c+h)-f(c-h)/2h = lim [ f(c+h)-f(c) +f(c) - f(c-h)
>> ]/2h = 1/2 lim x->c-[f(c+h)-f(c)/h] + 1/2 lim x->c+ [f(c+h)-f(c)/h]
>>
>> Logo como existem esses limites laterais, existe a derivada em 0, e
>> portanto, é L
>>
>> 2) queria tentar fazer, usando uma sequência xn<0 com limxn = 0 e yn>0
>> com lim yn = 0 e provar que lim(f(yn)-f(xn)/(yn-xn)) = f'(0) = L. Mas sinto
>> que isso é verdade e não sei provar
>>
>> Abraços
>>
>> --
>> Esta mensagem foi verificada pelo sistema de antivírus e
>> acredita-se estar livre de perigo.
>
>
> --
> Esta mensagem foi verificada pelo sistema de antivírus e
> acredita-se estar livre de perigo.
>

-- 
Esta mensagem foi verificada pelo sistema de antiv�rus e
 acredita-se estar livre de perigo.



[obm-l] Re: [obm-l] Questão de derivada

2018-04-23 Por tôpico Artur Steiner
Como f é contínua em 0, então, pela regra de L'Hopital,

lim x --> 0+  (f(x) - f0))/(x - 0) = lim x --> 0+ f'(x) = L

Pela definição de derivada lateral, o limite do primeiro membro é a
derivada à direita de 0. É só o que podemos concluir do enunciado. Nada
garante que a derivada à esquerda de 0 sequer exista.

Artur Costa Steiner

Em Dom, 22 de abr de 2018 22:45, Igor Caetano Diniz 
escreveu:

> Boa noite,
> Gostaria de uma ajuda numa questão. Primeiro saber se pensei corretamente
> na maneira (1) e se é possível resolver como pensei também na maneira (2).
> Aí vai:
> Questão 5.3.8 do livro do Stephen Abbot, Understanding Analysis:
>
> Assuma que f é contínua em um intervalo que contém o zero e diferenciável
> em todo ponto diferente de zero. Se lim f ' (x) = L, x->0, prove que f '
> (0) existe e é igual a L.
>
> O que pensei em fazer:
>
> Pensei em duas maneiras.
> 1)Se o limite existe em 0, então existem os limites laterais, limite a
> esquerda e limite a direita: lim x->0- f ' (x) = L e lim x->0+ f ' (x) = L.
> Lema: f ' (c) = lim f(c+h)-f(c-h)/2h = lim [ f(c+h)-f(c) +f(c) - f(c-h)
> ]/2h = 1/2 lim x->c-[f(c+h)-f(c)/h] + 1/2 lim x->c+ [f(c+h)-f(c)/h]
>
> Logo como existem esses limites laterais, existe a derivada em 0, e
> portanto, é L
>
> 2) queria tentar fazer, usando uma sequência xn<0 com limxn = 0 e yn>0 com
> lim yn = 0 e provar que lim(f(yn)-f(xn)/(yn-xn)) = f'(0) = L. Mas sinto que
> isso é verdade e não sei provar
>
> Abraços
>
> --
> Esta mensagem foi verificada pelo sistema de antivírus e
> acredita-se estar livre de perigo.

-- 
Esta mensagem foi verificada pelo sistema de antiv�rus e
 acredita-se estar livre de perigo.



[obm-l] Re: [obm-l] Re: [obm-l] Questão de derivada

2018-04-23 Por tôpico Igor Caetano Diniz
Então,

Se existem os limites laterais, lim f ' (0-) = lim f ' (0+) então, defina
q(x) = [f(x) - f(0)]/x. Para todo x<0, existe y1 entre x e 0 tal que f '
(y) = q(x). Analogamente para x>0, existe z1 entre 0 e x tal que f ' (z) =
q(x).
Defina r(x,0) a distancia de x para 0
Então, seja yn = yn-1 + r(y_n-1,0)/2 e zn = zn-1 + r(z_n-1,0)/2. Tal
sequência converge para o 0 com lim yn = 0 = lim zn. Além disso, lim f '
(yn) = L = lim f ' (zn)

Não estou conseguindo concluir. Alguém poderia ajudar?

Abraços

2018-04-23 11:25 GMT-03:00 Bernardo Freitas Paulo da Costa <
bernardo...@gmail.com>:

> 2018-04-22 22:36 GMT-03:00 Igor Caetano Diniz :
> > Boa noite,
> > Gostaria de uma ajuda numa questão. Primeiro saber se pensei
> corretamente na
> > maneira (1) e se é possível resolver como pensei também na maneira (2).
> > Aí vai:
> > Questão 5.3.8 do livro do Stephen Abbot, Understanding Analysis:
> >
> > Assuma que f é contínua em um intervalo que contém o zero e
> diferenciável em
> > todo ponto diferente de zero. Se lim f ' (x) = L, x->0, prove que f ' (0)
> > existe e é igual a L.
> >
> > O que pensei em fazer:
> >
> > Pensei em duas maneiras.
> > 1)Se o limite existe em 0, então existem os limites laterais, limite a
> > esquerda e limite a direita: lim x->0- f ' (x) = L e lim x->0+ f ' (x) =
> L.
> > Lema: f ' (c) = lim f(c+h)-f(c-h)/2h = lim [ f(c+h)-f(c) +f(c) - f(c-h)
> ]/2h
> > = 1/2 lim x->c-[f(c+h)-f(c)/h] + 1/2 lim x->c+ [f(c+h)-f(c)/h]
> >
> > Logo como existem esses limites laterais, existe a derivada em 0, e
> > portanto, é L
>
> Cuidado, Igor: a existência do limite [f(c+h) - f(c-h)]/2h quando h ->
> 0 não implica que existe a derivada.  Por exemplo, se f(x) = |x|, o
> limite dá zero, mas a derivada não existe.
>
> > 2) queria tentar fazer, usando uma sequência xn<0 com limxn = 0 e yn>0
> com
> > lim yn = 0 e provar que lim(f(yn)-f(xn)/(yn-xn)) = f'(0) = L. Mas sinto
> que
> > isso é verdade e não sei provar
>
> De novo, pelo mesmo exemplo acima, não basta provar que os limites com
> um ponto de cada lado dão certo.  Acho (só acho) que se *todos* os
> limites possíveis derem iguais, então a derivada existe, o que
> justifica a sua abordagem, mas daí você teria que provar o enunciado
> geral
>
> "Se f é contínua, e para TODA sequência x_n < 0 < y_n, com x_n -> 0,
> y_n ->0, vale que [f(y_n) - f(x_n)]/(y_n - x_n) -> L, então f é
> derivável em zero, e f'(0) = L."
>
> Abraços,
> --
> Bernardo Freitas Paulo da Costa
>
> --
> Esta mensagem foi verificada pelo sistema de antivírus e
>  acredita-se estar livre de perigo.
>
>
> =
> Instru�ões para entrar na lista, sair da lista e usar a lista em
> http://www.mat.puc-rio.br/~obmlistas/obm-l.html
> =
>

-- 
Esta mensagem foi verificada pelo sistema de antiv�rus e
 acredita-se estar livre de perigo.



[obm-l] Re: [obm-l] Questão de derivada

2018-04-23 Por tôpico Bernardo Freitas Paulo da Costa
2018-04-22 22:36 GMT-03:00 Igor Caetano Diniz :
> Boa noite,
> Gostaria de uma ajuda numa questão. Primeiro saber se pensei corretamente na
> maneira (1) e se é possível resolver como pensei também na maneira (2).
> Aí vai:
> Questão 5.3.8 do livro do Stephen Abbot, Understanding Analysis:
>
> Assuma que f é contínua em um intervalo que contém o zero e diferenciável em
> todo ponto diferente de zero. Se lim f ' (x) = L, x->0, prove que f ' (0)
> existe e é igual a L.
>
> O que pensei em fazer:
>
> Pensei em duas maneiras.
> 1)Se o limite existe em 0, então existem os limites laterais, limite a
> esquerda e limite a direita: lim x->0- f ' (x) = L e lim x->0+ f ' (x) = L.
> Lema: f ' (c) = lim f(c+h)-f(c-h)/2h = lim [ f(c+h)-f(c) +f(c) - f(c-h) ]/2h
> = 1/2 lim x->c-[f(c+h)-f(c)/h] + 1/2 lim x->c+ [f(c+h)-f(c)/h]
>
> Logo como existem esses limites laterais, existe a derivada em 0, e
> portanto, é L

Cuidado, Igor: a existência do limite [f(c+h) - f(c-h)]/2h quando h ->
0 não implica que existe a derivada.  Por exemplo, se f(x) = |x|, o
limite dá zero, mas a derivada não existe.

> 2) queria tentar fazer, usando uma sequência xn<0 com limxn = 0 e yn>0 com
> lim yn = 0 e provar que lim(f(yn)-f(xn)/(yn-xn)) = f'(0) = L. Mas sinto que
> isso é verdade e não sei provar

De novo, pelo mesmo exemplo acima, não basta provar que os limites com
um ponto de cada lado dão certo.  Acho (só acho) que se *todos* os
limites possíveis derem iguais, então a derivada existe, o que
justifica a sua abordagem, mas daí você teria que provar o enunciado
geral

"Se f é contínua, e para TODA sequência x_n < 0 < y_n, com x_n -> 0,
y_n ->0, vale que [f(y_n) - f(x_n)]/(y_n - x_n) -> L, então f é
derivável em zero, e f'(0) = L."

Abraços,
-- 
Bernardo Freitas Paulo da Costa

-- 
Esta mensagem foi verificada pelo sistema de antiv�rus e
 acredita-se estar livre de perigo.


=
Instru��es para entrar na lista, sair da lista e usar a lista em
http://www.mat.puc-rio.br/~obmlistas/obm-l.html
=


[obm-l] Re: [obm-l] Re: [obm-l] Re: [obm-l] Re: [obm-l] Questão de Combinatória

2018-03-30 Por tôpico Anderson Torres
Em 29 de março de 2018 15:37, Igor Caetano Diniz
 escreveu:
> Vou mostrar a sua e a minha e aí se ele não aprender com as duas, tento
> fazer devagar em casos menores. hehe
>
> Abraços Cláudio e obrigado =)
>
> 2018-03-29 15:17 GMT-03:00 Claudio Buffara :
>>
>> Sim. Acho essa uma solução bem mais elegante.
>> Mas também é mais sofisticada, e você falou que o aluno é principiante.
>>
>> De todo jeito, acho que raciocinar recursivamente é uma habilidade que
>> todo estudante de matemática deveria desenvolver.
>>

Não seria mais interessante ir "montando" as possíveis sequências num
diagrama de árvore mesmo, já que é para fazer no bração?

Cada nível da árvore é obtido acrescentando 0 e 1 ao final do nível
anterior e aniquilando os que quebram o padrão (dois 1s consecutivos).

*

0 1

00 01 10 11X

000 001 010 011X 100 101

 0001 0010 0011X 0100 0101 1000 1001 1010 1011X

E assim por diante. Afinal, se é para contar na mão, tem que organizar.

>> []s,
>> Claudio.
>>
>>
>> 2018-03-29 14:45 GMT-03:00 Igor Caetano Diniz :
>>>
>>> Olá Claudio
>>> Pensei numa solução agora que acredito que eu possa explicar e a pessoa
>>> irá entender:
>>>
>>> Para 1 bit, 2 possibilidades
>>> Para 2 bits, 3
>>> Para 3 bits, basta separar em casos: Se for 0 _ _, cai no caso anterior.
>>> Se for 1 _ _ tem que ser  1 0 _ e, então, cai no caso anterior-1.
>>> Para 4 bits, separe de novo: 0 _ _ _, que cai no problema anterior, ou, 1
>>> 0 _ _, caindo no anterior -1.
>>> Ou seja, Para N bits: F(N) = F(N-1) + F(N-2). É um Fibonacci começando de
>>> F(1) = 2 e F(2) = 3
>>>
>>>
>>> Estaria correto assim?
>>>
>>> Abraços
>>>
>>> 2018-03-29 14:26 GMT-03:00 Claudio Buffara :

 Sugestão: separe em casos em função do número N de 1’s na sequência.

 N = 0: 1 sequência
 N = 1: 8 sequências
 N = 2: 8*7/2 - 7 = 21
 (No de sequências sem restrições menos o no de sequências com os dois
 1’s adjacentes)
 N = 4: 2
 N > 4: 0

 O caso N = 3 é o mais chatinho pois tem mais subcasos, mas não chega a
 ser difícil.

 Depois eu mando.

 Abs





 Enviado do meu iPhone

 Em 29 de mar de 2018, à(s) 13:31, Igor Caetano Diniz
  escreveu:

 > Olá pessoal,
 >
 > Estou com uma questão de Combinatória e gostaria de uma solução
 > didática para ela pq como eu fiz ficou complexo para um aluno que 
 > iniciou
 > combinatória agora.
 > segue a questão:
 >
 > Quantas sequências de 8 bits(com 0's e 1's) não têm dois 1
 > consecutivos?
 >
 > Como foi resolvida: usando variáveis para contar quantos 0 estão
 > entre 1's consecutivos, separada em casos de dois, três e quatro 1's
 > consecutivos. Mas assim fica difícil para quem começou a aprender 
 > agora.
 >
 > Abraços
 >
 > --
 > Esta mensagem foi verificada pelo sistema de antivírus e
 > acredita-se estar livre de perigo.

 --
 Esta mensagem foi verificada pelo sistema de antivírus e
  acredita-se estar livre de perigo.



 =
 Instru�ões para entrar na lista, sair da lista e usar a lista em
 http://www.mat.puc-rio.br/~obmlistas/obm-l.html

 =
>>>
>>>
>>>
>>> --
>>> Esta mensagem foi verificada pelo sistema de antivírus e
>>> acredita-se estar livre de perigo.
>>
>>
>>
>> --
>> Esta mensagem foi verificada pelo sistema de antivírus e
>> acredita-se estar livre de perigo.
>
>
>
> --
> Esta mensagem foi verificada pelo sistema de antivírus e
> acredita-se estar livre de perigo.

-- 
Esta mensagem foi verificada pelo sistema de antiv�rus e
 acredita-se estar livre de perigo.


=
Instru��es para entrar na lista, sair da lista e usar a lista em
http://www.mat.puc-rio.br/~obmlistas/obm-l.html
=


[obm-l] Re: [obm-l] Re: [obm-l] Re: [obm-l] Re: [obm-l] Questão de Combinatória

2018-03-29 Por tôpico Claudio Buffara
Outra sugestão: proponha o problema de contar de quantas maneiras é
possível arrumar N dominós 1x2 numa caixa 2xN.
Fibonacci também aparece neste aí.

A diferença é que, no dos bits, B(N) = F(N+2) enquanto que, no dos dominós,
D(N) = F(N+1)
(F é definida da forma usual, com F(1) = F(2) = 1)

Ou então: quantas sequências de 1's e 2's existem que têm soma N?
Aqui, X(N) = F(N+1) também.

Um problema complementar interessante é achar bijeções "naturais" entre as
sequências definidas por estes três problemas.
Entre D e X é fácil.  Entre estes e as suas sequências de bits nem tanto.

[]s,
Claudio.

-- 
Esta mensagem foi verificada pelo sistema de antiv�rus e
 acredita-se estar livre de perigo.



[obm-l] Re: [obm-l] Re: [obm-l] Questão de Combinatória

2018-03-29 Por tôpico Claudio Buffara
Sugestão de natureza didática: eu mostraria uma solução mais braçal, tal
como a minha, e depois mostraria a solução recursiva.
Moral: em geral vale a pena pensar no problema antes de sair escrevendo...

2018-03-29 15:17 GMT-03:00 Claudio Buffara :

> Sim. Acho essa uma solução bem mais elegante.
> Mas também é mais sofisticada, e você falou que o aluno é principiante.
>
> De todo jeito, acho que raciocinar recursivamente é uma habilidade que
> todo estudante de matemática deveria desenvolver.
>
> []s,
> Claudio.
>
>
> 2018-03-29 14:45 GMT-03:00 Igor Caetano Diniz :
>
>> Olá Claudio
>> Pensei numa solução agora que acredito que eu possa explicar e a pessoa
>> irá entender:
>>
>> Para 1 bit, 2 possibilidades
>> Para 2 bits, 3
>> Para 3 bits, basta separar em casos: Se for 0 _ _, cai no caso anterior.
>> Se for 1 _ _ tem que ser  1 0 _ e, então, cai no caso anterior-1.
>> Para 4 bits, separe de novo: 0 _ _ _, que cai no problema anterior, ou, 1
>> 0 _ _, caindo no anterior -1.
>> Ou seja, Para N bits: F(N) = F(N-1) + F(N-2). É um Fibonacci começando de
>> F(1) = 2 e F(2) = 3
>>
>>
>> Estaria correto assim?
>>
>> Abraços
>>
>> 2018-03-29 14:26 GMT-03:00 Claudio Buffara :
>>
>>> Sugestão: separe em casos em função do número N de 1’s na sequência.
>>>
>>> N = 0: 1 sequência
>>> N = 1: 8 sequências
>>> N = 2: 8*7/2 - 7 = 21
>>> (No de sequências sem restrições menos o no de sequências com os dois
>>> 1’s adjacentes)
>>> N = 4: 2
>>> N > 4: 0
>>>
>>> O caso N = 3 é o mais chatinho pois tem mais subcasos, mas não chega a
>>> ser difícil.
>>>
>>> Depois eu mando.
>>>
>>> Abs
>>>
>>>
>>>
>>>
>>>
>>> Enviado do meu iPhone
>>>
>>> Em 29 de mar de 2018, à(s) 13:31, Igor Caetano Diniz <
>>> icaetanodi...@gmail.com> escreveu:
>>>
>>> > Olá pessoal,
>>> >
>>> > Estou com uma questão de Combinatória e gostaria de uma solução
>>> didática para ela pq como eu fiz ficou complexo para um aluno que iniciou
>>> combinatória agora.
>>> > segue a questão:
>>> >
>>> > Quantas sequências de 8 bits(com 0's e 1's) não têm dois 1
>>> consecutivos?
>>> >
>>> > Como foi resolvida: usando variáveis para contar quantos 0 estão
>>> entre 1's consecutivos, separada em casos de dois, três e quatro 1's
>>> consecutivos. Mas assim fica difícil para quem começou a aprender agora.
>>> >
>>> > Abraços
>>> >
>>> > --
>>> > Esta mensagem foi verificada pelo sistema de antivírus e
>>> > acredita-se estar livre de perigo.
>>>
>>> --
>>> Esta mensagem foi verificada pelo sistema de antivírus e
>>>  acredita-se estar livre de perigo.
>>>
>>>
>>> 
>>> =
>>> Instru�ões para entrar na lista, sair da lista e usar a lista em
>>> http://www.mat.puc-rio.br/~obmlistas/obm-l.html
>>> 
>>> =
>>>
>>
>>
>> --
>> Esta mensagem foi verificada pelo sistema de antivírus e
>> acredita-se estar livre de perigo.
>>
>
>

-- 
Esta mensagem foi verificada pelo sistema de antiv�rus e
 acredita-se estar livre de perigo.



[obm-l] Re: [obm-l] Re: [obm-l] Re: [obm-l] Questão de Combinatória

2018-03-29 Por tôpico Igor Caetano Diniz
Vou mostrar a sua e a minha e aí se ele não aprender com as duas, tento
fazer devagar em casos menores. hehe

Abraços Cláudio e obrigado =)

2018-03-29 15:17 GMT-03:00 Claudio Buffara :

> Sim. Acho essa uma solução bem mais elegante.
> Mas também é mais sofisticada, e você falou que o aluno é principiante.
>
> De todo jeito, acho que raciocinar recursivamente é uma habilidade que
> todo estudante de matemática deveria desenvolver.
>
> []s,
> Claudio.
>
>
> 2018-03-29 14:45 GMT-03:00 Igor Caetano Diniz :
>
>> Olá Claudio
>> Pensei numa solução agora que acredito que eu possa explicar e a pessoa
>> irá entender:
>>
>> Para 1 bit, 2 possibilidades
>> Para 2 bits, 3
>> Para 3 bits, basta separar em casos: Se for 0 _ _, cai no caso anterior.
>> Se for 1 _ _ tem que ser  1 0 _ e, então, cai no caso anterior-1.
>> Para 4 bits, separe de novo: 0 _ _ _, que cai no problema anterior, ou, 1
>> 0 _ _, caindo no anterior -1.
>> Ou seja, Para N bits: F(N) = F(N-1) + F(N-2). É um Fibonacci começando de
>> F(1) = 2 e F(2) = 3
>>
>>
>> Estaria correto assim?
>>
>> Abraços
>>
>> 2018-03-29 14:26 GMT-03:00 Claudio Buffara :
>>
>>> Sugestão: separe em casos em função do número N de 1’s na sequência.
>>>
>>> N = 0: 1 sequência
>>> N = 1: 8 sequências
>>> N = 2: 8*7/2 - 7 = 21
>>> (No de sequências sem restrições menos o no de sequências com os dois
>>> 1’s adjacentes)
>>> N = 4: 2
>>> N > 4: 0
>>>
>>> O caso N = 3 é o mais chatinho pois tem mais subcasos, mas não chega a
>>> ser difícil.
>>>
>>> Depois eu mando.
>>>
>>> Abs
>>>
>>>
>>>
>>>
>>>
>>> Enviado do meu iPhone
>>>
>>> Em 29 de mar de 2018, à(s) 13:31, Igor Caetano Diniz <
>>> icaetanodi...@gmail.com> escreveu:
>>>
>>> > Olá pessoal,
>>> >
>>> > Estou com uma questão de Combinatória e gostaria de uma solução
>>> didática para ela pq como eu fiz ficou complexo para um aluno que iniciou
>>> combinatória agora.
>>> > segue a questão:
>>> >
>>> > Quantas sequências de 8 bits(com 0's e 1's) não têm dois 1
>>> consecutivos?
>>> >
>>> > Como foi resolvida: usando variáveis para contar quantos 0 estão
>>> entre 1's consecutivos, separada em casos de dois, três e quatro 1's
>>> consecutivos. Mas assim fica difícil para quem começou a aprender agora.
>>> >
>>> > Abraços
>>> >
>>> > --
>>> > Esta mensagem foi verificada pelo sistema de antivírus e
>>> > acredita-se estar livre de perigo.
>>>
>>> --
>>> Esta mensagem foi verificada pelo sistema de antivírus e
>>>  acredita-se estar livre de perigo.
>>>
>>>
>>> 
>>> =
>>> Instru�ões para entrar na lista, sair da lista e usar a lista em
>>> http://www.mat.puc-rio.br/~obmlistas/obm-l.html
>>> 
>>> =
>>>
>>
>>
>> --
>> Esta mensagem foi verificada pelo sistema de antivírus e
>> acredita-se estar livre de perigo.
>>
>
>
> --
> Esta mensagem foi verificada pelo sistema de antivírus e
> acredita-se estar livre de perigo.
>

-- 
Esta mensagem foi verificada pelo sistema de antiv�rus e
 acredita-se estar livre de perigo.



[obm-l] Re: [obm-l] Re: [obm-l] Questão de Combinatória

2018-03-29 Por tôpico Claudio Buffara
Sim. Acho essa uma solução bem mais elegante.
Mas também é mais sofisticada, e você falou que o aluno é principiante.

De todo jeito, acho que raciocinar recursivamente é uma habilidade que todo
estudante de matemática deveria desenvolver.

[]s,
Claudio.


2018-03-29 14:45 GMT-03:00 Igor Caetano Diniz :

> Olá Claudio
> Pensei numa solução agora que acredito que eu possa explicar e a pessoa
> irá entender:
>
> Para 1 bit, 2 possibilidades
> Para 2 bits, 3
> Para 3 bits, basta separar em casos: Se for 0 _ _, cai no caso anterior.
> Se for 1 _ _ tem que ser  1 0 _ e, então, cai no caso anterior-1.
> Para 4 bits, separe de novo: 0 _ _ _, que cai no problema anterior, ou, 1
> 0 _ _, caindo no anterior -1.
> Ou seja, Para N bits: F(N) = F(N-1) + F(N-2). É um Fibonacci começando de
> F(1) = 2 e F(2) = 3
>
>
> Estaria correto assim?
>
> Abraços
>
> 2018-03-29 14:26 GMT-03:00 Claudio Buffara :
>
>> Sugestão: separe em casos em função do número N de 1’s na sequência.
>>
>> N = 0: 1 sequência
>> N = 1: 8 sequências
>> N = 2: 8*7/2 - 7 = 21
>> (No de sequências sem restrições menos o no de sequências com os dois 1’s
>> adjacentes)
>> N = 4: 2
>> N > 4: 0
>>
>> O caso N = 3 é o mais chatinho pois tem mais subcasos, mas não chega a
>> ser difícil.
>>
>> Depois eu mando.
>>
>> Abs
>>
>>
>>
>>
>>
>> Enviado do meu iPhone
>>
>> Em 29 de mar de 2018, à(s) 13:31, Igor Caetano Diniz <
>> icaetanodi...@gmail.com> escreveu:
>>
>> > Olá pessoal,
>> >
>> > Estou com uma questão de Combinatória e gostaria de uma solução
>> didática para ela pq como eu fiz ficou complexo para um aluno que iniciou
>> combinatória agora.
>> > segue a questão:
>> >
>> > Quantas sequências de 8 bits(com 0's e 1's) não têm dois 1
>> consecutivos?
>> >
>> > Como foi resolvida: usando variáveis para contar quantos 0 estão
>> entre 1's consecutivos, separada em casos de dois, três e quatro 1's
>> consecutivos. Mas assim fica difícil para quem começou a aprender agora.
>> >
>> > Abraços
>> >
>> > --
>> > Esta mensagem foi verificada pelo sistema de antivírus e
>> > acredita-se estar livre de perigo.
>>
>> --
>> Esta mensagem foi verificada pelo sistema de antivírus e
>>  acredita-se estar livre de perigo.
>>
>>
>> =
>> Instru�ões para entrar na lista, sair da lista e usar a lista em
>> http://www.mat.puc-rio.br/~obmlistas/obm-l.html
>> =
>>
>
>
> --
> Esta mensagem foi verificada pelo sistema de antivírus e
> acredita-se estar livre de perigo.
>

-- 
Esta mensagem foi verificada pelo sistema de antiv�rus e
 acredita-se estar livre de perigo.



[obm-l] Re: [obm-l] Questão de Combinatória

2018-03-29 Por tôpico Igor Caetano Diniz
Olá Claudio
Pensei numa solução agora que acredito que eu possa explicar e a pessoa irá
entender:

Para 1 bit, 2 possibilidades
Para 2 bits, 3
Para 3 bits, basta separar em casos: Se for 0 _ _, cai no caso anterior. Se
for 1 _ _ tem que ser  1 0 _ e, então, cai no caso anterior-1.
Para 4 bits, separe de novo: 0 _ _ _, que cai no problema anterior, ou, 1 0
_ _, caindo no anterior -1.
Ou seja, Para N bits: F(N) = F(N-1) + F(N-2). É um Fibonacci começando de
F(1) = 2 e F(2) = 3


Estaria correto assim?

Abraços

2018-03-29 14:26 GMT-03:00 Claudio Buffara :

> Sugestão: separe em casos em função do número N de 1’s na sequência.
>
> N = 0: 1 sequência
> N = 1: 8 sequências
> N = 2: 8*7/2 - 7 = 21
> (No de sequências sem restrições menos o no de sequências com os dois 1’s
> adjacentes)
> N = 4: 2
> N > 4: 0
>
> O caso N = 3 é o mais chatinho pois tem mais subcasos, mas não chega a ser
> difícil.
>
> Depois eu mando.
>
> Abs
>
>
>
>
>
> Enviado do meu iPhone
>
> Em 29 de mar de 2018, à(s) 13:31, Igor Caetano Diniz <
> icaetanodi...@gmail.com> escreveu:
>
> > Olá pessoal,
> >
> > Estou com uma questão de Combinatória e gostaria de uma solução
> didática para ela pq como eu fiz ficou complexo para um aluno que iniciou
> combinatória agora.
> > segue a questão:
> >
> > Quantas sequências de 8 bits(com 0's e 1's) não têm dois 1
> consecutivos?
> >
> > Como foi resolvida: usando variáveis para contar quantos 0 estão entre
> 1's consecutivos, separada em casos de dois, três e quatro 1's
> consecutivos. Mas assim fica difícil para quem começou a aprender agora.
> >
> > Abraços
> >
> > --
> > Esta mensagem foi verificada pelo sistema de antivírus e
> > acredita-se estar livre de perigo.
>
> --
> Esta mensagem foi verificada pelo sistema de antivírus e
>  acredita-se estar livre de perigo.
>
>
> =
> Instru�ões para entrar na lista, sair da lista e usar a lista em
> http://www.mat.puc-rio.br/~obmlistas/obm-l.html
> =
>

-- 
Esta mensagem foi verificada pelo sistema de antiv�rus e
 acredita-se estar livre de perigo.



[obm-l] Re: [obm-l] Re: [obm-l] Re: [obm-l] Re: [obm-l] Re: [obm-l] Questão de Cardinalidade

2018-01-21 Por tôpico Anderson Torres
Em 16 de janeiro de 2018 13:50, Bernardo Freitas Paulo da Costa
 escreveu:
> 2018-01-16 1:10 GMT-02:00 Anderson Torres :
>> Eu na verdade pensei ao contrário:
>>
>> Começamos com o conjunto de todos os subconjuntos de N. Cada conjunto
>> será representado por uma string infinita de zeros e unzes, da
>> seguinte forma: Se o conjunto contiver o natural x, o x-ésimo
>> caractere desta string será 1; caso contrário, será 0.
>>
>> Botando zero-vírgula na frente, obtemos um número real escrito em base
>> 2, contido no intervalo [0,1] (para efeito de completude do argumento,
>> admitiremos strings infinitas de 1zes).
>>
>> Para cada real em [0,1], bastaria escrever na base 2 e criar um
>> conjunto a partir daí, seguindo os passos acima (se o X-esimo dígito é
>> 1, escolhe X, caso contrário, despreza X).
>>
>> Isso prova que existe uma bijeção entre o conjunto das partes de N e o
>> intervalo [0,1].
>
> Acho que tanto a sua demonstração como a do Sávio têm um problema:
>
> 0,0111... = 0.1...
>
> Isso quer dizer que o conjunto {0} e o conjunto {1,2,3,...} são
> enviados no mesmo número real (conhecido como 1/2, ou 0.5 em decimal).
>
> Eu sempre acho muita "forçação de barra" tentar exibir uma bijeção.
> 99% das vezes, é mais esforço do que precisa, sem ganhar muito
> entendimento.  Ou, como neste caso, papa-se uma mosca...  Minha
> sugestão é exibir uma sobrejeção de P(IN) em IR, e depois uma
> sobrejeção de IR em P(IN).  A primeira está garantida, pois basta
> compor a construção do número binário em [0,1] com qualquer sobrejeção
> deste conjunto em R.  Uma sobrejeção simples é mandar 0 e 1 "pra
> qualquer lugar", e depois usar uma bijeção de (0,1) em IR.

Claro que tem a questão das formalizações, mas acho que elas são
trabalho demais para compreensão de menos. Só quis exibir algumas
funções que podem ser o que precisamos.


>
> Deixo para vocês pensarem como fazer para exibir uma sobrejeção de IR
> nas partes de IN.  Dica: IR contém [0,1) e [1,2).

Diretamente? Ainda acho que bijetar toda a reta em um de seus
segmentos uma jogada mais interessante...

>
> Abraços,
> --
> Bernardo Freitas Paulo da Costa
>
> --
> Esta mensagem foi verificada pelo sistema de antivírus e
>  acredita-se estar livre de perigo.
>
>
> =
> Instru�ões para entrar na lista, sair da lista e usar a lista em
> http://www.mat.puc-rio.br/~obmlistas/obm-l.html
> =

-- 
Esta mensagem foi verificada pelo sistema de antiv�rus e
 acredita-se estar livre de perigo.


=
Instru��es para entrar na lista, sair da lista e usar a lista em
http://www.mat.puc-rio.br/~obmlistas/obm-l.html
=


[obm-l] Re: [obm-l] Re: [obm-l] Re: [obm-l] Re: [obm-l] Re: [obm-l] Re: [obm-l] Questão de Cardinalidade

2018-01-16 Por tôpico Bernardo Freitas Paulo da Costa
2018-01-16 14:11 GMT-02:00 Igor Caetano Diniz :
> Fala Bernardo, tudo certo?
> Mas sera que eu conseguiria provar que esses números não seriam uma
> quantidade enumeravel de pontos entre 0 e 1 e, então, como é enumeravel, eu
> consigo pegar uma quantidade enumeravel em P(N) para esses pontos.

Sim, de fato são enumeráveis (é um exercício legal provar isto).  Dá
um pouco mais de trabalho "modificar" as bijeções para corrigir o que
está acontecendo nestes pontos

> Acha que
> seria ruim?

Não digo que seja ruim, só acho que é "trabalho demais" quando você
poderia ir por um caminho mais simples ;-)

Abraços,
-- 
Bernardo Freitas Paulo da Costa

-- 
Esta mensagem foi verificada pelo sistema de antiv�rus e
 acredita-se estar livre de perigo.


=
Instru��es para entrar na lista, sair da lista e usar a lista em
http://www.mat.puc-rio.br/~obmlistas/obm-l.html
=


[obm-l] Re: [obm-l] Re: [obm-l] Re: [obm-l] Re: [obm-l] Re: [obm-l] Questão de Cardinalidade

2018-01-16 Por tôpico Igor Caetano Diniz
Fala Bernardo, tudo certo?
Mas sera que eu conseguiria provar que esses números não seriam uma
quantidade enumeravel de pontos entre 0 e 1 e, então, como é enumeravel, eu
consigo pegar uma quantidade enumeravel em P(N) para esses pontos. Acha que
seria ruim?

Abraço

On Jan 16, 2018 13:59, "Bernardo Freitas Paulo da Costa" <
bernardo...@gmail.com> wrote:

> 2018-01-16 1:10 GMT-02:00 Anderson Torres :
> > Eu na verdade pensei ao contrário:
> >
> > Começamos com o conjunto de todos os subconjuntos de N. Cada conjunto
> > será representado por uma string infinita de zeros e unzes, da
> > seguinte forma: Se o conjunto contiver o natural x, o x-ésimo
> > caractere desta string será 1; caso contrário, será 0.
> >
> > Botando zero-vírgula na frente, obtemos um número real escrito em base
> > 2, contido no intervalo [0,1] (para efeito de completude do argumento,
> > admitiremos strings infinitas de 1zes).
> >
> > Para cada real em [0,1], bastaria escrever na base 2 e criar um
> > conjunto a partir daí, seguindo os passos acima (se o X-esimo dígito é
> > 1, escolhe X, caso contrário, despreza X).
> >
> > Isso prova que existe uma bijeção entre o conjunto das partes de N e o
> > intervalo [0,1].
>
> Acho que tanto a sua demonstração como a do Sávio têm um problema:
>
> 0,0111... = 0.1...
>
> Isso quer dizer que o conjunto {0} e o conjunto {1,2,3,...} são
> enviados no mesmo número real (conhecido como 1/2, ou 0.5 em decimal).
>
> Eu sempre acho muita "forçação de barra" tentar exibir uma bijeção.
> 99% das vezes, é mais esforço do que precisa, sem ganhar muito
> entendimento.  Ou, como neste caso, papa-se uma mosca...  Minha
> sugestão é exibir uma sobrejeção de P(IN) em IR, e depois uma
> sobrejeção de IR em P(IN).  A primeira está garantida, pois basta
> compor a construção do número binário em [0,1] com qualquer sobrejeção
> deste conjunto em R.  Uma sobrejeção simples é mandar 0 e 1 "pra
> qualquer lugar", e depois usar uma bijeção de (0,1) em IR.
>
> Deixo para vocês pensarem como fazer para exibir uma sobrejeção de IR
> nas partes de IN.  Dica: IR contém [0,1) e [1,2).
>
> Abraços,
> --
> Bernardo Freitas Paulo da Costa
>
> --
> Esta mensagem foi verificada pelo sistema de antivírus e
>  acredita-se estar livre de perigo.
>
>
> =
> Instru�ões para entrar na lista, sair da lista e usar a lista em
> http://www.mat.puc-rio.br/~obmlistas/obm-l.html
> =
>

-- 
Esta mensagem foi verificada pelo sistema de antiv�rus e
 acredita-se estar livre de perigo.



[obm-l] Re: [obm-l] Re: [obm-l] Re: [obm-l] Re: [obm-l] Questão de Cardinalidade

2018-01-16 Por tôpico Bernardo Freitas Paulo da Costa
2018-01-16 1:10 GMT-02:00 Anderson Torres :
> Eu na verdade pensei ao contrário:
>
> Começamos com o conjunto de todos os subconjuntos de N. Cada conjunto
> será representado por uma string infinita de zeros e unzes, da
> seguinte forma: Se o conjunto contiver o natural x, o x-ésimo
> caractere desta string será 1; caso contrário, será 0.
>
> Botando zero-vírgula na frente, obtemos um número real escrito em base
> 2, contido no intervalo [0,1] (para efeito de completude do argumento,
> admitiremos strings infinitas de 1zes).
>
> Para cada real em [0,1], bastaria escrever na base 2 e criar um
> conjunto a partir daí, seguindo os passos acima (se o X-esimo dígito é
> 1, escolhe X, caso contrário, despreza X).
>
> Isso prova que existe uma bijeção entre o conjunto das partes de N e o
> intervalo [0,1].

Acho que tanto a sua demonstração como a do Sávio têm um problema:

0,0111... = 0.1...

Isso quer dizer que o conjunto {0} e o conjunto {1,2,3,...} são
enviados no mesmo número real (conhecido como 1/2, ou 0.5 em decimal).

Eu sempre acho muita "forçação de barra" tentar exibir uma bijeção.
99% das vezes, é mais esforço do que precisa, sem ganhar muito
entendimento.  Ou, como neste caso, papa-se uma mosca...  Minha
sugestão é exibir uma sobrejeção de P(IN) em IR, e depois uma
sobrejeção de IR em P(IN).  A primeira está garantida, pois basta
compor a construção do número binário em [0,1] com qualquer sobrejeção
deste conjunto em R.  Uma sobrejeção simples é mandar 0 e 1 "pra
qualquer lugar", e depois usar uma bijeção de (0,1) em IR.

Deixo para vocês pensarem como fazer para exibir uma sobrejeção de IR
nas partes de IN.  Dica: IR contém [0,1) e [1,2).

Abraços,
-- 
Bernardo Freitas Paulo da Costa

-- 
Esta mensagem foi verificada pelo sistema de antiv�rus e
 acredita-se estar livre de perigo.


=
Instru��es para entrar na lista, sair da lista e usar a lista em
http://www.mat.puc-rio.br/~obmlistas/obm-l.html
=


[obm-l] Re: [obm-l] Re: [obm-l] Re: [obm-l] Re: [obm-l] Questão de Cardinalidade

2018-01-16 Por tôpico Igor Caetano Diniz
Uma ideia legal Para provar que (-1,1) tem bijeção com R, seria usar f(x) =
x/(x^2-1) provando que ela eh injetiva e sobrejetiva

On Jan 16, 2018 01:20, "Anderson Torres" 
wrote:

> Eu na verdade pensei ao contrário:
>
> Começamos com o conjunto de todos os subconjuntos de N. Cada conjunto
> será representado por uma string infinita de zeros e unzes, da
> seguinte forma: Se o conjunto contiver o natural x, o x-ésimo
> caractere desta string será 1; caso contrário, será 0.
>
> Botando zero-vírgula na frente, obtemos um número real escrito em base
> 2, contido no intervalo [0,1] (para efeito de completude do argumento,
> admitiremos strings infinitas de 1zes).
>
> Para cada real em [0,1], bastaria escrever na base 2 e criar um
> conjunto a partir daí, seguindo os passos acima (se o X-esimo dígito é
> 1, escolhe X, caso contrário, despreza X).
>
> Isso prova que existe uma bijeção entre o conjunto das partes de N e o
> intervalo [0,1].
>
> Agora, provar que [0,1] tem a mesma cardinalidade que R é mais
> chatinho. Dá para pensar geometricamente:
>
> Primeiro, [0,1] tem a mesma cardinalidade de [-1,+1], basta dobrar e
> tirar 1 (f(x)=2x-1).
>
> Agora, como demonstrar que [-1,+1] bijeta com todos os reais? Bem,
> isso não me parece complicado: se pensarmos na inversão de centro zero
> e raio um, o elemento X<1 vai ser levado em 1/X>1. Assim, todo número
> fora de [-1,+1] é bijetado com um dentro de [-1,+1] - podemos
> convencionar que -1,0,+1 vão neles mesmos.
>
> Para sermos mais precisos, o intervalo [0,1] é bijetado em [1,+inf], e
> o intervalo [-1,0] em [-inf,-1]
>
> Agora vem o toque final: acrescente 1 em cada elemento do intervalo
> [-inf,-1], diminua 1 em cada elemento de [1,+inf] e una os resultados.
> Com isso, obtemos uma bijeção de [-inf,-1] união [1,+inf] com toda a
> reta!
>
> E acabou!
> Em 15 de janeiro de 2018 17:11, Igor Caetano Diniz
>  escreveu:
> > Olá Sávio,
> > Muito obrigado. Tava pensando em algo parecido mas agora voce esclareceu
> > bastante.
> > Abraços
> >
> > On Jan 15, 2018 16:55, "Sávio Ribas"  wrote:
> >>
> >> Boa tarde!
> >> A primeira parte servirá para mostrar que a cardinalidade de IR é igual
> à
> >> cardinalidade de [0,1].
> >> Não é difícil mostrar que a reta tem a mesma cardinalidade que, por
> >> exemplo, o intervalo (-1,1) -- basta tomar a bijeção f: (-1,1) -> IR
> dada
> >> por f(x) = tg(pi*x/2).
> >> O passo seguinte seria mostrar que (-1,1) tem a mesma cardinalidade que
> o
> >> intervalo (fechado) [0,1], e para isso vamos tomar a bijeção g: (0,1) ->
> >> (-1,1) dada por g(x) = 2x-1. Mas note que "faltam o pontos 0 e 1" no
> domínio
> >> de g. Vamos acrescentar esses pontos, tomando um conjunto enumerável A =
> >> {a_1, a_2, a_3,...} contido em (0,1) e fazendo o seguinte: Seja B = {0,
> 1,
> >> a_1, a_2, a_3, ...}. A função h: (0,1) -> [0,1] dada por h(x) = x se x
> não
> >> está em A, h(a_1) = 0, h(a_2) = 1, h(a_n) = a_{n-2} se n>2 é uma bijeção
> >> (verifique).
> >> Assim, a função [ h o g^(-1) o f^(-1) ]: IR -> [0,1] é uma bijeção. Daí,
> >> concluímos que IR e [0,1] possuem a mesma cardinalidade.
> >>
> >> Vamos agora mostrar que as cardinalidades de [0,1] e IN são iguais. Seja
> >> 0,b_1b_2b_3... a representação binária de um número em [0,1] com
> infinitas
> >> casas (por exemplo, 1 será representado por 0,1...). Essa escrita
> >> binária dos elementos de [0,1] gera uma bijeção com as partes de IN da
> >> seguinte forma: k perntence a um subconjunto M dos naturais se e
> somente se
> >> b_k = 1 (por exemplo, o vazio corresponde ao 0 = 0,..., IN
> corresponde
> >> ao 1 = 0,... e {2,3,5,7} corresponde a 0,01101010...). Dessa
> forma,
> >> construímos uma bijeção entre P(IN) e [0,1].
> >>
> >> Concluímos que P(IN) e IR possuem mesma cardinalidade, pois ambos estão
> em
> >> bijeção com [0,1].
> >>
> >> Sávio
> >>
> >>
> >> Em 15 de jan de 2018 13:43, "Igor Caetano Diniz" <
> icaetanodi...@gmail.com>
> >> escreveu:
> >>>
> >>> Olá a todos, estou com uma dúvida para provar uma questão(Sem usar
> >>> hipótese do contínuo)
> >>>
> >>> Prove que a cardinalidade do conjunto das partes dos números naturais é
> >>> igual à cardinalidade dos reais, i.e., |P(N)| = |R|
> >>>
> >>>
> >>> quem puder ajudar, agradeço.
> >>>
> >>> Abraços
> >>>
> >>> --
> >>> Esta mensagem foi verificada pelo sistema de antivírus e
> >>> acredita-se estar livre de perigo.
> >>
> >>
> >> --
> >> Esta mensagem foi verificada pelo sistema de antivírus e
> >> acredita-se estar livre de perigo.
> >
> >
> > --
> > Esta mensagem foi verificada pelo sistema de antivírus e
> > acredita-se estar livre de perigo.
>
> --
> Esta mensagem foi verificada pelo sistema de antivírus e
>  acredita-se estar livre de perigo.
>
>
> =
> Instru�ões para entrar na lista, sair da lista e usar a lista em
> http://www.mat.puc-rio.br/~obmlistas/obm-l.html
> 

[obm-l] Re: [obm-l] Re: [obm-l] Re: [obm-l] Questão de Cardinalidade

2018-01-15 Por tôpico Anderson Torres
Eu na verdade pensei ao contrário:

Começamos com o conjunto de todos os subconjuntos de N. Cada conjunto
será representado por uma string infinita de zeros e unzes, da
seguinte forma: Se o conjunto contiver o natural x, o x-ésimo
caractere desta string será 1; caso contrário, será 0.

Botando zero-vírgula na frente, obtemos um número real escrito em base
2, contido no intervalo [0,1] (para efeito de completude do argumento,
admitiremos strings infinitas de 1zes).

Para cada real em [0,1], bastaria escrever na base 2 e criar um
conjunto a partir daí, seguindo os passos acima (se o X-esimo dígito é
1, escolhe X, caso contrário, despreza X).

Isso prova que existe uma bijeção entre o conjunto das partes de N e o
intervalo [0,1].

Agora, provar que [0,1] tem a mesma cardinalidade que R é mais
chatinho. Dá para pensar geometricamente:

Primeiro, [0,1] tem a mesma cardinalidade de [-1,+1], basta dobrar e
tirar 1 (f(x)=2x-1).

Agora, como demonstrar que [-1,+1] bijeta com todos os reais? Bem,
isso não me parece complicado: se pensarmos na inversão de centro zero
e raio um, o elemento X<1 vai ser levado em 1/X>1. Assim, todo número
fora de [-1,+1] é bijetado com um dentro de [-1,+1] - podemos
convencionar que -1,0,+1 vão neles mesmos.

Para sermos mais precisos, o intervalo [0,1] é bijetado em [1,+inf], e
o intervalo [-1,0] em [-inf,-1]

Agora vem o toque final: acrescente 1 em cada elemento do intervalo
[-inf,-1], diminua 1 em cada elemento de [1,+inf] e una os resultados.
Com isso, obtemos uma bijeção de [-inf,-1] união [1,+inf] com toda a
reta!

E acabou!
Em 15 de janeiro de 2018 17:11, Igor Caetano Diniz
 escreveu:
> Olá Sávio,
> Muito obrigado. Tava pensando em algo parecido mas agora voce esclareceu
> bastante.
> Abraços
>
> On Jan 15, 2018 16:55, "Sávio Ribas"  wrote:
>>
>> Boa tarde!
>> A primeira parte servirá para mostrar que a cardinalidade de IR é igual à
>> cardinalidade de [0,1].
>> Não é difícil mostrar que a reta tem a mesma cardinalidade que, por
>> exemplo, o intervalo (-1,1) -- basta tomar a bijeção f: (-1,1) -> IR dada
>> por f(x) = tg(pi*x/2).
>> O passo seguinte seria mostrar que (-1,1) tem a mesma cardinalidade que o
>> intervalo (fechado) [0,1], e para isso vamos tomar a bijeção g: (0,1) ->
>> (-1,1) dada por g(x) = 2x-1. Mas note que "faltam o pontos 0 e 1" no domínio
>> de g. Vamos acrescentar esses pontos, tomando um conjunto enumerável A =
>> {a_1, a_2, a_3,...} contido em (0,1) e fazendo o seguinte: Seja B = {0, 1,
>> a_1, a_2, a_3, ...}. A função h: (0,1) -> [0,1] dada por h(x) = x se x não
>> está em A, h(a_1) = 0, h(a_2) = 1, h(a_n) = a_{n-2} se n>2 é uma bijeção
>> (verifique).
>> Assim, a função [ h o g^(-1) o f^(-1) ]: IR -> [0,1] é uma bijeção. Daí,
>> concluímos que IR e [0,1] possuem a mesma cardinalidade.
>>
>> Vamos agora mostrar que as cardinalidades de [0,1] e IN são iguais. Seja
>> 0,b_1b_2b_3... a representação binária de um número em [0,1] com infinitas
>> casas (por exemplo, 1 será representado por 0,1...). Essa escrita
>> binária dos elementos de [0,1] gera uma bijeção com as partes de IN da
>> seguinte forma: k perntence a um subconjunto M dos naturais se e somente se
>> b_k = 1 (por exemplo, o vazio corresponde ao 0 = 0,..., IN corresponde
>> ao 1 = 0,... e {2,3,5,7} corresponde a 0,01101010...). Dessa forma,
>> construímos uma bijeção entre P(IN) e [0,1].
>>
>> Concluímos que P(IN) e IR possuem mesma cardinalidade, pois ambos estão em
>> bijeção com [0,1].
>>
>> Sávio
>>
>>
>> Em 15 de jan de 2018 13:43, "Igor Caetano Diniz" 
>> escreveu:
>>>
>>> Olá a todos, estou com uma dúvida para provar uma questão(Sem usar
>>> hipótese do contínuo)
>>>
>>> Prove que a cardinalidade do conjunto das partes dos números naturais é
>>> igual à cardinalidade dos reais, i.e., |P(N)| = |R|
>>>
>>>
>>> quem puder ajudar, agradeço.
>>>
>>> Abraços
>>>
>>> --
>>> Esta mensagem foi verificada pelo sistema de antivírus e
>>> acredita-se estar livre de perigo.
>>
>>
>> --
>> Esta mensagem foi verificada pelo sistema de antivírus e
>> acredita-se estar livre de perigo.
>
>
> --
> Esta mensagem foi verificada pelo sistema de antivírus e
> acredita-se estar livre de perigo.

-- 
Esta mensagem foi verificada pelo sistema de antiv�rus e
 acredita-se estar livre de perigo.


=
Instru��es para entrar na lista, sair da lista e usar a lista em
http://www.mat.puc-rio.br/~obmlistas/obm-l.html
=


[obm-l] Re: [obm-l] Re: [obm-l] Questão de Cardinalidade

2018-01-15 Por tôpico Igor Caetano Diniz
Olá Sávio,
Muito obrigado. Tava pensando em algo parecido mas agora voce esclareceu
bastante.
Abraços

On Jan 15, 2018 16:55, "Sávio Ribas"  wrote:

> Boa tarde!
> A primeira parte servirá para mostrar que a cardinalidade de IR é igual à
> cardinalidade de [0,1].
> Não é difícil mostrar que a reta tem a mesma cardinalidade que, por
> exemplo, o intervalo (-1,1) -- basta tomar a bijeção f: (-1,1) -> IR dada
> por f(x) = tg(pi*x/2).
> O passo seguinte seria mostrar que (-1,1) tem a mesma cardinalidade que o
> intervalo (fechado) [0,1], e para isso vamos tomar a bijeção g: (0,1) ->
> (-1,1) dada por g(x) = 2x-1. Mas note que "faltam o pontos 0 e 1" no
> domínio de g. Vamos acrescentar esses pontos, tomando um conjunto
> enumerável A = {a_1, a_2, a_3,...} contido em (0,1) e fazendo o seguinte:
> Seja B = {0, 1, a_1, a_2, a_3, ...}. A função h: (0,1) -> [0,1] dada por
> h(x) = x se x não está em A, h(a_1) = 0, h(a_2) = 1, h(a_n) = a_{n-2} se
> n>2 é uma bijeção (verifique).
> Assim, a função [ h o g^(-1) o f^(-1) ]: IR -> [0,1] é uma bijeção. Daí,
> concluímos que IR e [0,1] possuem a mesma cardinalidade.
>
> Vamos agora mostrar que as cardinalidades de [0,1] e IN são iguais. Seja
> 0,b_1b_2b_3... a representação binária de um número em [0,1] com infinitas
> casas (por exemplo, 1 será representado por 0,1...). Essa escrita
> binária dos elementos de [0,1] gera uma bijeção com as partes de IN da
> seguinte forma: k perntence a um subconjunto M dos naturais se e somente se
> b_k = 1 (por exemplo, o vazio corresponde ao 0 = 0,..., IN corresponde
> ao 1 = 0,... e {2,3,5,7} corresponde a 0,01101010...). Dessa forma,
> construímos uma bijeção entre P(IN) e [0,1].
>
> Concluímos que P(IN) e IR possuem mesma cardinalidade, pois ambos estão em
> bijeção com [0,1].
>
> Sávio
>
>
> Em 15 de jan de 2018 13:43, "Igor Caetano Diniz" 
> escreveu:
>
>> Olá a todos, estou com uma dúvida para provar uma questão(Sem usar
>> hipótese do contínuo)
>>
>> Prove que a cardinalidade do conjunto das partes dos números naturais é
>> igual à cardinalidade dos reais, i.e., |P(N)| = |R|
>>
>>
>> quem puder ajudar, agradeço.
>>
>> Abraços
>>
>> --
>> Esta mensagem foi verificada pelo sistema de antivírus e
>> acredita-se estar livre de perigo.
>
>
> --
> Esta mensagem foi verificada pelo sistema de antivírus e
> acredita-se estar livre de perigo.

-- 
Esta mensagem foi verificada pelo sistema de antiv�rus e
 acredita-se estar livre de perigo.



[obm-l] Re: [obm-l] Questão de Cardinalidade

2018-01-15 Por tôpico Sávio Ribas
Boa tarde!
A primeira parte servirá para mostrar que a cardinalidade de IR é igual à
cardinalidade de [0,1].
Não é difícil mostrar que a reta tem a mesma cardinalidade que, por
exemplo, o intervalo (-1,1) -- basta tomar a bijeção f: (-1,1) -> IR dada
por f(x) = tg(pi*x/2).
O passo seguinte seria mostrar que (-1,1) tem a mesma cardinalidade que o
intervalo (fechado) [0,1], e para isso vamos tomar a bijeção g: (0,1) ->
(-1,1) dada por g(x) = 2x-1. Mas note que "faltam o pontos 0 e 1" no
domínio de g. Vamos acrescentar esses pontos, tomando um conjunto
enumerável A = {a_1, a_2, a_3,...} contido em (0,1) e fazendo o seguinte:
Seja B = {0, 1, a_1, a_2, a_3, ...}. A função h: (0,1) -> [0,1] dada por
h(x) = x se x não está em A, h(a_1) = 0, h(a_2) = 1, h(a_n) = a_{n-2} se
n>2 é uma bijeção (verifique).
Assim, a função [ h o g^(-1) o f^(-1) ]: IR -> [0,1] é uma bijeção. Daí,
concluímos que IR e [0,1] possuem a mesma cardinalidade.

Vamos agora mostrar que as cardinalidades de [0,1] e IN são iguais. Seja
0,b_1b_2b_3... a representação binária de um número em [0,1] com infinitas
casas (por exemplo, 1 será representado por 0,1...). Essa escrita
binária dos elementos de [0,1] gera uma bijeção com as partes de IN da
seguinte forma: k perntence a um subconjunto M dos naturais se e somente se
b_k = 1 (por exemplo, o vazio corresponde ao 0 = 0,..., IN corresponde
ao 1 = 0,... e {2,3,5,7} corresponde a 0,01101010...). Dessa forma,
construímos uma bijeção entre P(IN) e [0,1].

Concluímos que P(IN) e IR possuem mesma cardinalidade, pois ambos estão em
bijeção com [0,1].

Sávio


Em 15 de jan de 2018 13:43, "Igor Caetano Diniz" 
escreveu:

> Olá a todos, estou com uma dúvida para provar uma questão(Sem usar
> hipótese do contínuo)
>
> Prove que a cardinalidade do conjunto das partes dos números naturais é
> igual à cardinalidade dos reais, i.e., |P(N)| = |R|
>
>
> quem puder ajudar, agradeço.
>
> Abraços
>
> --
> Esta mensagem foi verificada pelo sistema de antivírus e
> acredita-se estar livre de perigo.

-- 
Esta mensagem foi verificada pelo sistema de antiv�rus e
 acredita-se estar livre de perigo.



[obm-l] Re: [obm-l] Re: [obm-l] Questão

2017-08-28 Por tôpico Pedro José
Bom dia!

Daniel,
eu já me sinto gratificado quando consigo resolver algo. Não sou
matemático, sou um pitaqueiro, com alto grau curiosidade e matemática é uma
das minhas curiosidades preferidas.
O que mais me fascina, é que sou totalmente crente em que um modelo
matemático formulado com estrutura, se não tem aplicação prática é porque
não enxergamos onde podemos usá-lo no momento e não por que não tenha
utilização, como alguns leigos (como eu, somente no fato de ser leigo)
 apregoam.
A matemática é a mais veloz das ciências.
O Gabriel falou em ordem, fez uma observação e depois se corrigiu.
O difícil aqui é saber o nível de conhecimento das pessoas, por isso que
não falei em ordem, mas se você não souber o que é ordem, pesquise.
Pesquise também o que seria uma raiz primitiva, caso não conheça o
conceito, pois 10 é raiz primitiva módulo 23.

Saudações,
PJMS

Em 25 de agosto de 2017 16:51, Gabriel Tostes  escreveu:

> Confundi, eh 22 msm. :D
>
> On Aug 25, 2017, at 12:28 AM, Daniel da Silva <
> danielrochadasi...@icloud.com> wrote:
>
> Obrigado Pedro.
>
> Daniel Rocha da Silva
>
> Em 23 de ago de 2017, às 19:31, Pedro José 
> escreveu:
>
> Boa noite!
>
> O difícil é achar o n.
>
> Como o menor inteiro positivo que atende 10^a = 1 mod23 é a=22
>
> E como 10^3 = 11 mod23.
>
> Temos que K + 1 = 3 +22*m com m natural
> então k = 2 + 22*m.
>
> e n/2 = [10^(k+1) -11]/23 ==> n=2*[10^(k+1)-11]/23.
>
> Portanto as soluções serão (2+ 22*m; 2*[10^(3+22*m)-11]/23; com
> m= 0,1, 2, 3, 4
>
> Então há uma infinidade de soluções. você achou a relativa
> a m=0.
>
> ou seja, k= 2 e n = 2*[10^3-11]/23=2*43=86
>
> para m =1; k= 24 e n= 869,575.217.391.304.347.826.086
>
> Salvo engano para n pois fiz na marra.
>
> Saudações,
> PJMS
>
> Em 23 de agosto de 2017 17:19, Daniel da Silva <
> danielrochadasi...@icloud.com> escreveu:
>
>> Boa tarde,
>>
>> Como saber quantos valores inteiros
>> de N e K satisfazem a seguinte equação:
>>
>> 10^(K+1)=11+23N/2
>>
>> Encontrei uma solução (N=86, K=2), mas como saber se
>> é única?
>>
>> Obrigado,
>> Daniel Rocha da Silva
>> --
>> Esta mensagem foi verificada pelo sistema de antivírus e
>> Â acredita-se estar livre de perigo.
>>
>>
>> =
>> Instruções para entrar na lista, sair da lista e usar a lista em
>> http://www.mat.puc-rio.br/~obmlistas/obm-l.html
>> =
>>
>
>
> --
> Esta mensagem foi verificada pelo sistema de antivírus e
> acredita-se estar livre de perigo.
>
>
> --
> Esta mensagem foi verificada pelo sistema de antivírus e
> acredita-se estar livre de perigo.
>
>
> --
> Esta mensagem foi verificada pelo sistema de antivírus e
> acredita-se estar livre de perigo.
>

-- 
Esta mensagem foi verificada pelo sistema de antiv�rus e
 acredita-se estar livre de perigo.



Re: [obm-l] Re: [obm-l] Questão

2017-08-25 Por tôpico Gabriel Tostes
Faltou so uma coisa, a ordem de 10 mod 23 é 11 nao 22. Entao o k= 2+11k 


> On Aug 25, 2017, at 12:28 AM, Daniel da Silva  
> wrote:
> 
> Obrigado Pedro.
> 
> Daniel Rocha da Silva
> 
> Em 23 de ago de 2017, às 19:31, Pedro José  escreveu:
> 
>> Boa noite!
>> 
>> O difícil é achar o n.
>> 
>> Como o menor inteiro positivo que atende 10^a = 1 mod23 é a=22
>> 
>> E como 10^3 = 11 mod23.
>> 
>> Temos que K + 1 = 3 +22*m com m natural
>> então k = 2 + 22*m.
>> 
>> e n/2 = [10^(k+1) -11]/23 ==> n=2*[10^(k+1)-11]/23.
>> 
>> Portanto as soluções serão (2+ 22*m; 2*[10^(3+22*m)-11]/23; com m= 
>> 0,1, 2, 3, 4
>> 
>> Então há uma infinidade de soluções. você achou a relativa a 
>> m=0.
>> 
>> ou seja, k= 2 e n = 2*[10^3-11]/23=2*43=86
>> 
>> para m =1; k= 24 e n= 869,575.217.391.304.347.826.086
>> 
>> Salvo engano para n pois fiz na marra.
>> 
>> Saudações,
>> PJMS
>> 
>> Em 23 de agosto de 2017 17:19, Daniel da Silva 
>>  escreveu:
>>> Boa tarde,
>>> 
>>> Como saber quantos valores inteiros
>>> de N e K satisfazem a seguinte equação:
>>> 
>>> 10^(K+1)=11+23N/2
>>> 
>>> Encontrei uma solução (N=86, K=2), mas como saber se é 
>>> única?
>>> 
>>> Obrigado,
>>> Daniel Rocha da Silva
>>> --
>>> Esta mensagem foi verificada pelo sistema de antivírus e
>>> Â acredita-se estar livre de perigo.
>>> 
>>> 
>>> =
>>> Instruções para entrar na lista, sair da lista e usar a lista em
>>> http://www.mat.puc-rio.br/~obmlistas/obm-l.html
>>> =
>> 
>> 
>> -- 
>> Esta mensagem foi verificada pelo sistema de antivírus e 
>> acredita-se estar livre de perigo.
> 
> -- 
> Esta mensagem foi verificada pelo sistema de antivírus e 
> acredita-se estar livre de perigo.

-- 
Esta mensagem foi verificada pelo sistema de antiv�rus e
 acredita-se estar livre de perigo.



Re: [obm-l] Re: [obm-l] Questão

2017-08-25 Por tôpico Gabriel Tostes
Confundi, eh 22 msm. :D

> On Aug 25, 2017, at 12:28 AM, Daniel da Silva  
> wrote:
> 
> Obrigado Pedro.
> 
> Daniel Rocha da Silva
> 
> Em 23 de ago de 2017, às 19:31, Pedro José  escreveu:
> 
>> Boa noite!
>> 
>> O difícil é achar o n.
>> 
>> Como o menor inteiro positivo que atende 10^a = 1 mod23 é a=22
>> 
>> E como 10^3 = 11 mod23.
>> 
>> Temos que K + 1 = 3 +22*m com m natural
>> então k = 2 + 22*m.
>> 
>> e n/2 = [10^(k+1) -11]/23 ==> n=2*[10^(k+1)-11]/23.
>> 
>> Portanto as soluções serão (2+ 22*m; 2*[10^(3+22*m)-11]/23; com m= 
>> 0,1, 2, 3, 4
>> 
>> Então há uma infinidade de soluções. você achou a relativa a 
>> m=0.
>> 
>> ou seja, k= 2 e n = 2*[10^3-11]/23=2*43=86
>> 
>> para m =1; k= 24 e n= 869,575.217.391.304.347.826.086
>> 
>> Salvo engano para n pois fiz na marra.
>> 
>> Saudações,
>> PJMS
>> 
>> Em 23 de agosto de 2017 17:19, Daniel da Silva 
>>  escreveu:
>>> Boa tarde,
>>> 
>>> Como saber quantos valores inteiros
>>> de N e K satisfazem a seguinte equação:
>>> 
>>> 10^(K+1)=11+23N/2
>>> 
>>> Encontrei uma solução (N=86, K=2), mas como saber se é 
>>> única?
>>> 
>>> Obrigado,
>>> Daniel Rocha da Silva
>>> --
>>> Esta mensagem foi verificada pelo sistema de antivírus e
>>> Â acredita-se estar livre de perigo.
>>> 
>>> 
>>> =
>>> Instruções para entrar na lista, sair da lista e usar a lista em
>>> http://www.mat.puc-rio.br/~obmlistas/obm-l.html
>>> =
>> 
>> 
>> -- 
>> Esta mensagem foi verificada pelo sistema de antivírus e 
>> acredita-se estar livre de perigo.
> 
> -- 
> Esta mensagem foi verificada pelo sistema de antivírus e 
> acredita-se estar livre de perigo.

-- 
Esta mensagem foi verificada pelo sistema de antiv�rus e
 acredita-se estar livre de perigo.



Re: [obm-l] Re: [obm-l] Questão

2017-08-24 Por tôpico Daniel da Silva
Obrigado Pedro.

Daniel Rocha da Silva

> Em 23 de ago de 2017, às 19:31, Pedro José  escreveu:
> 
> Boa noite!
> 
> O difícil é achar o n.
> 
> Como o menor inteiro positivo que atende 10^a = 1 mod23 é a=22
> 
> E como 10^3 = 11 mod23.
> 
> Temos que K + 1 = 3 +22*m com m natural
> então k = 2 + 22*m.
> 
> e n/2 = [10^(k+1) -11]/23 ==> n=2*[10^(k+1)-11]/23.
> 
> Portanto as soluções serão (2+ 22*m; 2*[10^(3+22*m)-11]/23; com m= 0,1, 2, 
> 3, 4
> 
> Então há uma infinidade de soluções. você achou a relativa a m=0.
> 
> ou seja, k= 2 e n = 2*[10^3-11]/23=2*43=86
> 
> para m =1; k= 24 e n= 869,575.217.391.304.347.826.086
> 
> Salvo engano para n pois fiz na marra.
> 
> Saudações,
> PJMS
> 
> Em 23 de agosto de 2017 17:19, Daniel da Silva 
>  escreveu:
>> Boa tarde,
>> 
>> Como saber quantos valores inteiros
>> de N e K satisfazem a seguinte equação:
>> 
>> 10^(K+1)=11+23N/2
>> 
>> Encontrei uma solução (N=86, K=2), mas como saber se é única?
>> 
>> Obrigado,
>> Daniel Rocha da Silva
>> --
>> Esta mensagem foi verificada pelo sistema de antivírus e
>> Â acredita-se estar livre de perigo.
>> 
>> 
>> =
>> Instruções para entrar na lista, sair da lista e usar a lista em
>> http://www.mat.puc-rio.br/~obmlistas/obm-l.html
>> =
> 
> 
> -- 
> Esta mensagem foi verificada pelo sistema de antivírus e 
> acredita-se estar livre de perigo.

-- 
Esta mensagem foi verificada pelo sistema de antiv�rus e
 acredita-se estar livre de perigo.



[obm-l] Re: [obm-l] Questão

2017-08-23 Por tôpico Pedro José
Boa noite!

O difícil é achar o n.

Como o menor inteiro positivo que atende 10^a = 1 mod23 é a=22

E como 10^3 = 11 mod23.

Temos que K + 1 = 3 +22*m com m natural
então k = 2 + 22*m.

e n/2 = [10^(k+1) -11]/23 ==> n=2*[10^(k+1)-11]/23.

Portanto as soluções serão (2+ 22*m; 2*[10^(3+22*m)-11]/23; com m= 0,1, 2,
3, 4

Então há uma infinidade de soluções. você achou a relativa a m=0.

ou seja, k= 2 e n = 2*[10^3-11]/23=2*43=86

para m =1; k= 24 e n= 869,575.217.391.304.347.826.086

Salvo engano para n pois fiz na marra.

Saudações,
PJMS

Em 23 de agosto de 2017 17:19, Daniel da Silva <
danielrochadasi...@icloud.com> escreveu:

> Boa tarde,
>
> Como saber quantos valores inteiros
> de N e K satisfazem a seguinte equação:
>
> 10^(K+1)=11+23N/2
>
> Encontrei uma solução (N=86, K=2), mas como saber se é única?
>
> Obrigado,
> Daniel Rocha da Silva
> --
> Esta mensagem foi verificada pelo sistema de antivírus e
>  acredita-se estar livre de perigo.
>
>
> =
> Instruções para entrar na lista, sair da lista e usar a lista em
> http://www.mat.puc-rio.br/~obmlistas/obm-l.html
> =
>

-- 
Esta mensagem foi verificada pelo sistema de antiv�rus e
 acredita-se estar livre de perigo.



[obm-l] Re: [obm-l]Re: [obm-l] Re: [obm-l] Re: [obm-l] Questão de teoria numérica

2017-08-01 Por tôpico Pedro Cardoso
Realmente. Se isso serve de desculpa eu escrevi isso assim que acordei.
O que eu quis dizer é que não existem múltiplos de 2017 que terminem em 0 e 
que, ao serem divididos por 10, deixam de ser múltiplos de 2017. Para isso 
existir, 2017 teria que ter um número de fatores 2 diferente do número de 
fatores 5, mostrar porque isso não é difícil: supondo que k tem x fatores 2 e y 
fatores 5 se x>y temos que 5k/10 tem x-1 fatores 2 e y fatores 5, e portanto, 
não é divisível por k, para x escreveu:
>
> Obrigado! Era exatamente isso que a questão anterior sugeria, usar o 
> princípio da casa dos pombos.
> Uma coisa que percebi na sua dsmonstração é que o número encontrado 
> terminaria em 0s, mas como nenhum multiplo de 2017 também é multiplo de 10 
> (2017 é primo) então também existe um multiplo de 2017 com apenas 1s!

A conclusão está certa (existe um múltiplo de 2017 só com 1s), mas a
justificativa está errada: 20170 é múltiplo de 2017 e de 10.  Tem a
ver com primalidade, mas não é bem o que você escreveu.

Abraços,
-- 
Bernardo Freitas Paulo da Costa

-- 
Esta mensagem foi verificada pelo sistema de antiv�rus e
 acredita-se estar livre de perigo.


=
Instru��es para entrar na lista, sair da lista e usar a lista em
http://www.mat.puc-rio.br/~obmlistas/obm-l.html
=


-- 
Esta mensagem foi verificada pelo sistema de antiv�rus e
 acredita-se estar livre de perigo.



[obm-l] Re: [obm-l] Re: [obm-l] Re: [obm-l] Questão de teoria numérica

2017-08-01 Por tôpico Bernardo Freitas Paulo da Costa
Em 01/08/2017 08:14, "Pedro Cardoso"  escreveu:
>
> Obrigado! Era exatamente isso que a questão anterior sugeria, usar o 
> princípio da casa dos pombos.
> Uma coisa que percebi na sua dsmonstração é que o número encontrado 
> terminaria em 0s, mas como nenhum multiplo de 2017 também é multiplo de 10 
> (2017 é primo) então também existe um multiplo de 2017 com apenas 1s!

A conclusão está certa (existe um múltiplo de 2017 só com 1s), mas a
justificativa está errada: 20170 é múltiplo de 2017 e de 10.  Tem a
ver com primalidade, mas não é bem o que você escreveu.

Abraços,
-- 
Bernardo Freitas Paulo da Costa

-- 
Esta mensagem foi verificada pelo sistema de antiv�rus e
 acredita-se estar livre de perigo.


=
Instru��es para entrar na lista, sair da lista e usar a lista em
http://www.mat.puc-rio.br/~obmlistas/obm-l.html
=


[obm-l] Re: [obm-l] Re: [obm-l] Questão de teoria numérica

2017-08-01 Por tôpico Pedro Cardoso
Obrigado! Era exatamente isso que a questão anterior sugeria, usar o
princípio da casa dos pombos.
Uma coisa que percebi na sua dsmonstração é que o número encontrado
terminaria em 0s, mas como nenhum multiplo de 2017 também é multiplo de 10
(2017 é primo) então também existe um multiplo de 2017 com apenas 1s!

Obrigado.
Pedro Cardoso

Em 31 de jul de 2017 23:31, "Adilson Francisco da Silva" <
adilson...@gmail.com> escreveu:

> Salve!
>
> Construa uma sequência com 2018 números naturais da seguinte forma:
> 1
> 11
> 111
> 
> .
> .
> .
> 111...1 (2018 dígitos 1).
>
> Pelo princípio da casa dos pombos existe ao menos dois desses números que
> deixam o mesmo resto na divisão por 2017.
>
> Use o fato de que se dois números deixar o mesmo resto na divisão por um
> certo número d, então a diferença entre eles é divisível por d.
>
> Assim pegue os dois termos da sequência que deixa o mesmo resto é faça
>
> 11...111 - 11...1 = 111...100...0
>
> Que é divisível por 2017.
>
> Abraços
>
>
> Em 31 de jul de 2017 10:38 PM, "Pedro Cardoso" 
> escreveu:
>
> Segue uma questão de teoria numérica da Olimpíada SESI de Matemática (AM):
>
> Mostre que existe um múltiplo de 2017 formado apenas pelos dígitos 0 e 1
> (em base 10).
>
>
>
> Na olimpíada, a questão anterior sugere uma maneira de resolver, porém,
> estou interessado em outras demonstrações também.
>
> Se ninguém conseguir achar uma prova, mando a outra questão mais tarde.
>
> Att.
> Pedro Cardoso
>
>
>
> --
> Esta mensagem foi verificada pelo sistema de antivírus e
> acredita-se estar livre de perigo.
>
>
>
> --
> Esta mensagem foi verificada pelo sistema de antivírus e
> acredita-se estar livre de perigo.

-- 
Esta mensagem foi verificada pelo sistema de antiv�rus e
 acredita-se estar livre de perigo.



[obm-l] Re: [obm-l] Questão de teoria numérica

2017-07-31 Por tôpico Adilson Francisco da Silva
Salve!

Construa uma sequência com 2018 números naturais da seguinte forma:
1
11
111

.
.
.
111...1 (2018 dígitos 1).

Pelo princípio da casa dos pombos existe ao menos dois desses números que
deixam o mesmo resto na divisão por 2017.

Use o fato de que se dois números deixar o mesmo resto na divisão por um
certo número d, então a diferença entre eles é divisível por d.

Assim pegue os dois termos da sequência que deixa o mesmo resto é faça

11...111 - 11...1 = 111...100...0

Que é divisível por 2017.

Abraços


Em 31 de jul de 2017 10:38 PM, "Pedro Cardoso" 
escreveu:

Segue uma questão de teoria numérica da Olimpíada SESI de Matemática (AM):

Mostre que existe um múltiplo de 2017 formado apenas pelos dígitos 0 e 1
(em base 10).



Na olimpíada, a questão anterior sugere uma maneira de resolver, porém,
estou interessado em outras demonstrações também.

Se ninguém conseguir achar uma prova, mando a outra questão mais tarde.

Att.
Pedro Cardoso



-- 
Esta mensagem foi verificada pelo sistema de antivírus e
acredita-se estar livre de perigo.

-- 
Esta mensagem foi verificada pelo sistema de antiv�rus e
 acredita-se estar livre de perigo.



[obm-l] Re: [obm-l] Questão de um vestibular do Acre

2016-12-23 Por tôpico Pacini Bores
 

Oi Wanderlei, 

Realmente, acredito que falte o ângulo theta, já que ele pede para usar
sqrt(2)=1,4. 

Na verdade o comprimento da maca, para tocar os extremos nas paredes dos
corredores, tem sua limitação dada por 

(p^(2/3)+q^(2/3)^(3/2) se imaginarmos a largura da maca desprezível. 

Abraços 

Pacini 

Em 21/12/2016 16:50, Vanderlei Nemitz escreveu: 

> Boa tarde! 
> Tentei resolver uma questão de um vestibular do Acre, mas parece que faltam 
> informações, que talvez seja necessário supor. 
> Como acho que não posso anexar um arquivo aqui, deixo um link que acessa a 
> prova. É a questão 32, de geometria plana. 
> 
> http://www.strixeducacao.com.br/vs-arquivos/HtmlEditor/file/PROVAS%20APLICADAS/Uninorte_2016_2_Tipo1.pdf
>  [1] 
> 
> Muito obrigado! 
> 
> Vanderlei 
> -- 
> Esta mensagem foi verificada pelo sistema de antivrus e 
> acredita-se estar livre de perigo.
 

Links:
--
[1]
http://www.strixeducacao.com.br/vs-arquivos/HtmlEditor/file/PROVAS%20APLICADAS/Uninorte_2016_2_Tipo1.pdf
-- 
Esta mensagem foi verificada pelo sistema de antiv�rus e
 acredita-se estar livre de perigo.



[obm-l] Re: [obm-l] Re: [obm-l] Questão de Física

2016-10-16 Por tôpico Luiz Antonio Rodrigues
Olá, Pacini!
Muito obrigado!
Um abraço!
Luiz
On Oct 16, 2016 10:38 AM, "Pacini Bores"  wrote:

>
>
>
> Oi Luiz,
>
> o T para pequenas oscilações , T = 2.pi.sqrt(L/g) e com T´=5T=
> 2.pi.sqrt(L/g´), onde g´= (P-q.E)/m.
>
> Logo teremos : (T^2).g = ((T´)^2).g´ ou seja g=25.g´ou g = 25(P-q.E)/m e
> fazendo as contas, encontramos
>
> E = 240N/C.
>
> Abraços
>
> Pacini
>
> Em 15/10/2016 13:49, Luiz Antonio Rodrigues escreveu:
>
> Olá, pessoal! Peço desculpas por postar uma questão de Física, mas preciso
> de ajuda... Já tentei resolvê-la muitas vezes, sem sucesso. Não conheço um
> bom fórum de Física. Desde já agradeço qualquer ajuda. A questão é a
> seguinte:
>
>
> Um pequeno pêndulo simples é posto a oscilar entre duas superfícies
> metálicas planas, quadradas, muito grandes, paralelas e inicialmente
> neutras, apresentando um período T. O pêndulo simples é constituído por
> uma esfera metálica de massa 3,0x10– 4kg, eletrizada com carga de 12μC,
> e um fio isolante de massa desprezível e de comprimento 100cm. Nesse
> local, a aceleração da gravidade vale 10m/s2. A seguir, um dispositivo
> eletriza as placas metálicas, produzindo um campo elétrico uniforme e
> constante orientado para cima. Como as placas metálicas são muito
> grandes, toda a região de oscilação do pêndulo é abrangida pelo campo
> elétrico uniforme, fazendo com que o pêndulo passe a oscilar com um
> período 5T. Nessas condições, a intensidade do campo elétrico uniforme
> e constante estabelecido entre as placas metálicas vale
>
> (A) 125 N/C (B) 150 N/C (C) 200 N/C (D) 240 N/C
>
>
>
> Um abraço!
>
> Luiz
>
> --
> Esta mensagem foi verificada pelo sistema de antivrus e
> acredita-se estar livre de perigo.
>
>
> --
> Esta mensagem foi verificada pelo sistema de antivírus e
> acredita-se estar livre de perigo.
>

-- 
Esta mensagem foi verificada pelo sistema de antiv�rus e
 acredita-se estar livre de perigo.



[obm-l] Re: [obm-l] Questão de Física

2016-10-16 Por tôpico Pacini Bores
 

Oi Luiz, 

o T para pequenas oscilações , T = 2.pi.sqrt(L/g) e com T´=5T=
2.pi.sqrt(L/g´), onde g´= (P-q.E)/m. 

Logo teremos : (T^2).g = ((T´)^2).g´ ou seja g=25.g´ou g = 25(P-q.E)/m e
fazendo as contas, encontramos 

E = 240N/C. 

Abraços 

Pacini 

Em 15/10/2016 13:49, Luiz Antonio Rodrigues escreveu: 

> Olá, pessoal! Peço desculpas por postar uma questão de Física, mas preciso de 
> ajuda... Já tentei resolvê-la muitas vezes, sem sucesso. Não conheço um bom 
> fórum de Física. Desde já agradeço qualquer ajuda. A questão é a seguinte: 
> 
> Um pequeno pêndulo simples é posto a oscilar entre duas superfícies 
> metálicas planas, quadradas, muito grandes, paralelas e inicialmente 
> neutras, apresentando um período T. O pêndulo simples é constituído por 
> uma esfera metálica de massa 3,0x10- 4kg, eletrizada com carga de 12μC, e um 
> fio isolante de massa desprezível e de comprimento 100cm. Nesse local, a 
> aceleração da gravidade vale 10m/s2. A seguir, um dispositivo eletriza as 
> placas metálicas, produzindo um campo elétrico uniforme e constante 
> orientado para cima. Como as placas metálicas são muito grandes, toda a 
> região de oscilação do pêndulo é abrangida pelo campo elétrico 
> uniforme, fazendo com que o pêndulo passe a oscilar com um período 5T. 
> Nessas condições, a intensidade do campo elétrico uniforme e constante 
> estabelecido entre as placas metálicas vale 
> 
> (A) 125 N/C (B) 150 N/C (C) 200 N/C (D) 240 N/C 
> 
> Um abraço! 
> 
> Luiz 
> -- 
> Esta mensagem foi verificada pelo sistema de antivrus e 
> acredita-se estar livre de perigo.
 
-- 
Esta mensagem foi verificada pelo sistema de antiv�rus e
 acredita-se estar livre de perigo.



[obm-l] Re: [obm-l] Questão Geometria

2016-10-05 Por tôpico vinicius raimundo
Obrigado Douglas

Em quarta-feira, 5 de outubro de 2016, Douglas Oliveira de Lima <
profdouglaso.del...@gmail.com> escreveu:

> Bom vamos lá, não tem nada de bonito nessa resolução.
>
> Seja O o centro do ex-incirculo de ABC tangente ao lado BC, temos que AO é
> bissetriz do ângulo BAC, seja Q a intercessão de AO com BC, e J o pé da
> perpendicular tirada de O ao lado AC, sendo BAQ=x, nós teremos CAQ=ACB=x,
> AQB=OQC=2x. E OC é bissetriz de BCJ, assim BCO=90-x/2, e sendo P a
> intercessao de MO com BC.
>
> 1)Aplicando lei dos senos no triângulo AQC teremos
>
> AQ/AC=senx/sen(2x)
>
> 2)Agora aplicando no triângulo AMO teremos
>
> AM/MO=sen(QOP)/senx
>
> 3)E no triângulo CMO novamente lei dos senos teremos
>
> MC/MO=sen(COP)/cos(x/2)
>
> 4)Como AM=MC, dos itens (2) e (3) segue que
>
> sen(QOP)/sen(COP)=senx/cox(x/2)
>
> 5) Para o triângulo QPO, nós teremos
>
> sen(QOP)=[(QP)sen(2x)]/PO
>
> 6) Para o triângulo CPO, nós teremos
>
> sen(COP)=[(CP).cos(x/2)]/PO
>
> 7)Dos itens (5) e (6) podemos concluir que
>
> sen(QOP)/sen(COP)=[(QP).sen(2x)]/[(CP).cos(x/2)]
>
> 8)E de (4) e (7) nós temos
>
> senx/cos(x/2)=[(QP).sen(2x)]/[(CP).cos(x/2)], ou melhor QP/CP=senx/sen(2x)
>
> 9)Agora de (1) e (8) AQ/AC=QP/CP, donde vem
>
> QAP=CAP e BAP=x+QAP=x+CAP=BPA, ou seja ABP é isosceles e AB=BP.
>
>
> Um abraço  do
> Douglas Oliveira.
>
> Em 1 de outubro de 2016 19:54, vinicius raimundo  > escreveu:
>
>> Será que alguém poria me ajudar na seguinte questão?
>>
>>
>>1.
>>
>>(Belarus) Seja O o centro do círculo ex-inscrito do triângulo ABC oposto
>>ao vértice A. Seja M o ponto médio de AC e seja P a intersec ̧ão das
>>retas MO e BC. Prove que se ∠BAC = 2∠ACB, então  AB = BP.
>>
>>
>> --
>> Esta mensagem foi verificada pelo sistema de antivírus e
>> acredita-se estar livre de perigo.
>
>
>
> --
> Esta mensagem foi verificada pelo sistema de antivírus e
> acredita-se estar livre de perigo.

-- 
Esta mensagem foi verificada pelo sistema de antiv�rus e
 acredita-se estar livre de perigo.



[obm-l] Re: [obm-l] Re: [obm-l] Questão de vetores

2016-09-21 Por tôpico Bruno Lira
Tome N um ponto tal que MN seja paralelo a AB.
Note que o triângulo ABC é semelhante ao triângulo
NMC. Dá semelhança de triângulo temos que:
NM/AB = MC/BC => NM/BC = 5/8 => NM =5AB/8.
e
NC/AC = MC/BC => NC/AC = 5/8 => NC = 5AC/8.

AN = AC -NC = 3AC/8.

Daí:

vetor(AM) = vetor(AN) + vetor(NM)
  = (5/8)vetor(AB) + (3/8)vetor(AC)
C.Q.D

Em 19/09/2016 07:48, Anderson Torres escreveu:
> Em 21 de agosto de 2016 00:59, Henrique N. Lengler
>  escreveu:
>> Olá,
>>
>> Estou estudando vetores pelo livro "Vetores e uma iniciação à Geometria
>> Analítica" de Dorival A. de Mello e Renate Watanabe.
>>
>> Encontrei uma questão simples, mas que me deixou de cabelo em pé. Eu consegui
>> resolver, mas de uma maneira meio feia.
>>
>> A questão é: *.Obs: Todas as duplas de letras são vetores.
>>
>>
>> --
>> Em um triângulo ABC, o ponto M é tal que 3 BM = 5 MC. Escreva o vetor AM em
>> função dos vetores AB e AC.
>> --
>>
>> Alguém poderia me mostrar como resolveu?
>> A parte que me encomoda é a de descobrir qual número multiplicar o lado BC 
>> para
>> resultar em BM.
> A ideia de porcentagens é boa. Só lembrar que você pode considerar A
> como sendo a origem do sistema de coordenadas (afinal o resultado é
> invariante por translações), e concentrar-se apenas em B,.M, e C.
> Pense em M como sendo uma média ponderada entre B e C. Com isso, temos
> 3(M-B)=5(C-M), o que nos dará M=(3B+5C)/(3+5).
>
> Instantâneo!
>
>> Eu fiz de uma maneira tosca, como dá pra fazer em porcentagem por exemplo no
>> acréscimo de 50% eu fazia:
>>
>> x + (x * 50%) = x * y -> "y" será o número que multiplicado por x é o mesmo 
>> que
>> um acréscimo de 50%.
>>
>> No caso do problema é um decréscimo, então montei, sabendo que BC - MC = BM:
>>
>> BC - MC = BC * x(1)
>>
>> e resolvi transformando tudo em MC pelos dados da questão:
>>
>> BC = BM + MC
>>
>> BC = 5/3 MC + MC
>>
>> BC = 8/3 MC   (2)
>>
>> substituindo (2) em (1)
>>
>> 8/3 MC - MC = 8/3 MC * x
>>
>> 5/3 MC = 8/3 MC * x
>>
>> 5/3 MC * 3/8 * 1/MC = x
>>
>> simplificando MC
>>
>> x = 15/24 => 5/8
>>
>> Minha dúvida é, como poderia descobrir esse valor, que multiplica BC para me 
>> dar
>> BM de maneira mais rápida?
>>
>> Agradeço
>>
>> Henrique N. Lengler
>>
>> --
>> Esta mensagem foi verificada pelo sistema de antivírus e
>>   acredita-se estar livre de perigo.
>>
>> =
>> Instruções para entrar na lista, sair da lista e usar a lista em
>> http://www.mat.puc-rio.br/~obmlistas/obm-l.html
>> =


-- 
Esta mensagem foi verificada pelo sistema de antiv�rus e
 acredita-se estar livre de perigo.


=
Instru��es para entrar na lista, sair da lista e usar a lista em
http://www.mat.puc-rio.br/~obmlistas/obm-l.html
=


[obm-l] Re: [obm-l] Questão de vetores

2016-09-19 Por tôpico Anderson Torres
Em 21 de agosto de 2016 00:59, Henrique N. Lengler
 escreveu:
> Olá,
>
> Estou estudando vetores pelo livro "Vetores e uma iniciação à Geometria
> Analítica" de Dorival A. de Mello e Renate Watanabe.
>
> Encontrei uma questão simples, mas que me deixou de cabelo em pé. Eu consegui
> resolver, mas de uma maneira meio feia.
>
> A questão é: *.Obs: Todas as duplas de letras são vetores.
>
>
> --
> Em um triângulo ABC, o ponto M é tal que 3 BM = 5 MC. Escreva o vetor AM em
> função dos vetores AB e AC.
> --
>
> Alguém poderia me mostrar como resolveu?
> A parte que me encomoda é a de descobrir qual número multiplicar o lado BC 
> para
> resultar em BM.

A ideia de porcentagens é boa. Só lembrar que você pode considerar A
como sendo a origem do sistema de coordenadas (afinal o resultado é
invariante por translações), e concentrar-se apenas em B,.M, e C.
Pense em M como sendo uma média ponderada entre B e C. Com isso, temos
3(M-B)=5(C-M), o que nos dará M=(3B+5C)/(3+5).

Instantâneo!

>
> Eu fiz de uma maneira tosca, como dá pra fazer em porcentagem por exemplo no
> acréscimo de 50% eu fazia:
>
> x + (x * 50%) = x * y -> "y" será o número que multiplicado por x é o mesmo 
> que
> um acréscimo de 50%.
>
> No caso do problema é um decréscimo, então montei, sabendo que BC - MC = BM:
>
> BC - MC = BC * x(1)
>
> e resolvi transformando tudo em MC pelos dados da questão:
>
> BC = BM + MC
>
> BC = 5/3 MC + MC
>
> BC = 8/3 MC   (2)
>
> substituindo (2) em (1)
>
> 8/3 MC - MC = 8/3 MC * x
>
> 5/3 MC = 8/3 MC * x
>
> 5/3 MC * 3/8 * 1/MC = x
>
> simplificando MC
>
> x = 15/24 => 5/8
>
> Minha dúvida é, como poderia descobrir esse valor, que multiplica BC para me 
> dar
> BM de maneira mais rápida?
>
> Agradeço
>
> Henrique N. Lengler
>
> --
> Esta mensagem foi verificada pelo sistema de antivírus e
>  acredita-se estar livre de perigo.
>
> =
> Instruções para entrar na lista, sair da lista e usar a lista em
> http://www.mat.puc-rio.br/~obmlistas/obm-l.html
> =

-- 
Esta mensagem foi verificada pelo sistema de antiv�rus e
 acredita-se estar livre de perigo.


=
Instru��es para entrar na lista, sair da lista e usar a lista em
http://www.mat.puc-rio.br/~obmlistas/obm-l.html
=


[obm-l] Re: [obm-l] Questão de teoria dos números

2016-03-27 Por tôpico Anderson Torres
Em 27 de março de 2016 19:20, Pedro Chaves  escreveu:
> Caros Colegas,
>
> Peço-lhes ajuda na questão abaixo.
>
> Sendo x um número inteiro qualquer e y um inteiro positivo,  mostrar que
> existe um inteiro k tal que:
> --- ky é menor ou igual a x   e  (k+1)y é maior do que x.  
>

k <= x/y <= (k+1)

Tipo, é a parte inteira?

>
> Abraços do Pedro Chaves
> __
>
>
> --
> Esta mensagem foi verificada pelo sistema de antivírus e
> acredita-se estar livre de perigo.

-- 
Esta mensagem foi verificada pelo sistema de antiv�rus e
 acredita-se estar livre de perigo.


=
Instru��es para entrar na lista, sair da lista e usar a lista em
http://www.mat.puc-rio.br/~obmlistas/obm-l.html
=


[obm-l] Re: [obm-l] questão de treinamento olimpica.

2015-10-07 Por tôpico Esdras Muniz
Supondo por absurdo que isso ocorra, daí  temos que se a_i=11, então
b_i=11, do contrario, teríamos dois produtos de resto zero por 11. Então
vamos supor sem perda de generalidade que a_11=b_11=11.
daí, se x_i=a_i.b_i, supondo que {x_1,..., x_10} têm todos os restos
positivos possíveis por 11, então x_1.x_2...x_10 é congruente a (1.2...10)
(mod 11). (1.2...10)=(11-1)!.
Mas: x_1.x_2...x_10=(1.2...10)².
O teorema de Wilson garante que:
(11-1)! é congruente a -1 (mod 11). enquanto que (1.2...10)² é congruente a
1 (mod 11). O que é um absurdo.

Em 7 de outubro de 2015 08:59, Mauricio de Araujo <
mauricio.de.ara...@gmail.com> escreveu:

> Sejam a1, a2, ..., a11 e b1, b2, ..., b11 duas permutações dos inteiros 1,
> 2, ..., 11. Considere os números a1.b1, a2.b2, a3.b3, ..., a11.b11. Mostre
> que pelo menos dois destes números deixam o mesmo resto quando divididos
> por 11.
> Sugestão: Redução ao absurdo.
>
> --
> Abraços
>
> oɾnɐɹɐ ǝp oıɔıɹnɐɯ
>
>
> --
> Esta mensagem foi verificada pelo sistema de antivírus e
> acredita-se estar livre de perigo.




-- 
Esdras Muniz Mota
Mestrando em Matemática
Universidade Federal do Ceará

-- 
Esta mensagem foi verificada pelo sistema de antiv�rus e
 acredita-se estar livre de perigo.



[obm-l] Re: [obm-l] Re: [obm-l] questão de treinamento olimpica.

2015-10-07 Por tôpico Israel Meireles Chrisostomo
Mas isso aí não pode ser resolvido pelo princípio da casa dos pombos?


Em 7 de outubro de 2015 10:29, Esdras Muniz 
escreveu:

> Supondo por absurdo que isso ocorra, daí  temos que se a_i=11, então
> b_i=11, do contrario, teríamos dois produtos de resto zero por 11. Então
> vamos supor sem perda de generalidade que a_11=b_11=11.
> daí, se x_i=a_i.b_i, supondo que {x_1,..., x_10} têm todos os restos
> positivos possíveis por 11, então x_1.x_2...x_10 é congruente a (1.2...10)
> (mod 11). (1.2...10)=(11-1)!.
> Mas: x_1.x_2...x_10=(1.2...10)².
> O teorema de Wilson garante que:
> (11-1)! é congruente a -1 (mod 11). enquanto que (1.2...10)² é congruente
> a 1 (mod 11). O que é um absurdo.
>
> Em 7 de outubro de 2015 08:59, Mauricio de Araujo <
> mauricio.de.ara...@gmail.com> escreveu:
>
>> Sejam a1, a2, ..., a11 e b1, b2, ..., b11 duas permutações dos inteiros
>> 1, 2, ..., 11. Considere os números a1.b1, a2.b2, a3.b3, ..., a11.b11.
>> Mostre que pelo menos dois destes números deixam o mesmo resto quando
>> divididos por 11.
>> Sugestão: Redução ao absurdo.
>>
>> --
>> Abraços
>>
>> oɾnɐɹɐ ǝp oıɔıɹnɐɯ
>>
>>
>> --
>> Esta mensagem foi verificada pelo sistema de antivírus e
>> acredita-se estar livre de perigo.
>
>
>
>
> --
> Esdras Muniz Mota
> Mestrando em Matemática
> Universidade Federal do Ceará
>
>
>
> --
> Esta mensagem foi verificada pelo sistema de antivírus e
> acredita-se estar livre de perigo.
>

-- 
Esta mensagem foi verificada pelo sistema de antiv�rus e
 acredita-se estar livre de perigo.



[obm-l] Re: [obm-l] Re: [obm-l] questão de treinamento olimpica.

2015-10-07 Por tôpico Israel Meireles Chrisostomo
Ah tah, agora que eu vi que é o produto

Em 7 de outubro de 2015 13:57, Israel Meireles Chrisostomo <
israelmchrisost...@gmail.com> escreveu:

> Mas isso aí não pode ser resolvido pelo princípio da casa dos pombos?
>
>
> Em 7 de outubro de 2015 10:29, Esdras Muniz 
> escreveu:
>
>> Supondo por absurdo que isso ocorra, daí  temos que se a_i=11, então
>> b_i=11, do contrario, teríamos dois produtos de resto zero por 11. Então
>> vamos supor sem perda de generalidade que a_11=b_11=11.
>> daí, se x_i=a_i.b_i, supondo que {x_1,..., x_10} têm todos os restos
>> positivos possíveis por 11, então x_1.x_2...x_10 é congruente a (1.2...10)
>> (mod 11). (1.2...10)=(11-1)!.
>> Mas: x_1.x_2...x_10=(1.2...10)².
>> O teorema de Wilson garante que:
>> (11-1)! é congruente a -1 (mod 11). enquanto que (1.2...10)² é congruente
>> a 1 (mod 11). O que é um absurdo.
>>
>> Em 7 de outubro de 2015 08:59, Mauricio de Araujo <
>> mauricio.de.ara...@gmail.com> escreveu:
>>
>>> Sejam a1, a2, ..., a11 e b1, b2, ..., b11 duas permutações dos inteiros
>>> 1, 2, ..., 11. Considere os números a1.b1, a2.b2, a3.b3, ..., a11.b11.
>>> Mostre que pelo menos dois destes números deixam o mesmo resto quando
>>> divididos por 11.
>>> Sugestão: Redução ao absurdo.
>>>
>>> --
>>> Abraços
>>>
>>> oɾnɐɹɐ ǝp oıɔıɹnɐɯ
>>>
>>>
>>> --
>>> Esta mensagem foi verificada pelo sistema de antivírus e
>>> acredita-se estar livre de perigo.
>>
>>
>>
>>
>> --
>> Esdras Muniz Mota
>> Mestrando em Matemática
>> Universidade Federal do Ceará
>>
>>
>>
>> --
>> Esta mensagem foi verificada pelo sistema de antivírus e
>> acredita-se estar livre de perigo.
>>
>
>

-- 
Esta mensagem foi verificada pelo sistema de antiv�rus e
 acredita-se estar livre de perigo.



[obm-l] RE: [obm-l] Questão do ano passado

2015-09-28 Por tôpico Domingos Delgado Filho

 
From: domingos...@hotmail.com
To: obm-l@mat.puc-rio.br
Subject: [obm-l] Questão do ano passado
Date: Wed, 9 Sep 2015 18:26:41 -0300




Olá pessoal, boa noite a todos.Eu assisti um desenvolvimento de uma questão da 
obm do ano passado.Não sei precisar o nome da questão, mas é da segunda fase de 
2014, o problema das escadas.O enunciado , ao meu ver, restringi a questão ao 
comentar que tal pessoa não poderia voltar para um degrau intermediário, mas 
quando acompanhei tal resolução, o professor   não comenta tal fato.Por favor, 
peço que verifiquem tal situação.
Obrigado e parabéns pelo trabalho realizado ( a todos do grupo).
Domingos  
--

Esta mensagem foi verificada pelo sistema de antivírus e 

 acredita-se estar livre de perigo.   
-- 
Esta mensagem foi verificada pelo sistema de antivírus e
 acredita-se estar livre de perigo.



[obm-l] Re: [obm-l] Re: [obm-l] Questão Colégio Militar - anulada

2015-07-27 Por tôpico Pedro José
Boa tarde!

Desculpe-me, mas a Kacilda, faz o julgamento com as suas premissas.
Obviamente, se ela pensa eu o relógio está atrasado ela incluirá os 5 min.

Sds,

PJMS

Em 27 de julho de 2015 13:43, Mauricio de Araujo 
mauricio.de.ara...@gmail.com escreveu:


 Em 25 de julho de 2015 22:17, Martins Rama martin...@pop.com.br
 escreveu:

 Kacilda comparece ao trabalho julgando estar 10 minutos atrasada


 ​Acho que esta questão foi cancelada por conta do trecho Kacilda
 comparece ao trabalho julgando estar 10 minutos atrasada​ aqui temos
 incluídos os 5 minutos que ela já acredita que o relógio está atrasado ou
 não??



 --
 Abraços

 oɾnɐɹɐ ǝp oıɔıɹnɐɯ


 --
 Esta mensagem foi verificada pelo sistema de antivírus e
 acredita-se estar livre de perigo.


-- 
Esta mensagem foi verificada pelo sistema de antiv�rus e
 acredita-se estar livre de perigo.



[obm-l] Re: [obm-l] Questão Colégio Militar - anulada

2015-07-27 Por tôpico Mauricio de Araujo
Em 25 de julho de 2015 22:17, Martins Rama martin...@pop.com.br escreveu:

 Kacilda comparece ao trabalho julgando estar 10 minutos atrasada


​Acho que esta questão foi cancelada por conta do trecho Kacilda comparece
ao trabalho julgando estar 10 minutos atrasada​ aqui temos incluídos
os 5 minutos que ela já acredita que o relógio está atrasado ou não??



-- 
Abraços

oɾnɐɹɐ ǝp oıɔıɹnɐɯ

-- 
Esta mensagem foi verificada pelo sistema de antiv�rus e
 acredita-se estar livre de perigo.



[obm-l] Re: [obm-l] Re: [obm-l] Re: [obm-l] Questão Colégio Militar - anulada

2015-07-27 Por tôpico Mauricio de Araujo
Na verdade eu concordo com as soluções só estou atrás de um argumento
que justifique a anulação da questão... a única possibilidade que encontrei
foi alguém desenvolver um raciocínio diferente em cima da inclusão ou não
dos 5 minutos nos 10 minutos de atraso...
[]'s

Em 27 de julho de 2015 14:16, Pedro José petroc...@gmail.com escreveu:

 Boa tarde!

 Desculpe-me, mas a Kacilda, faz o julgamento com as suas premissas.
 Obviamente, se ela pensa eu o relógio está atrasado ela incluirá os 5 min.

 Sds,

 PJMS

 Em 27 de julho de 2015 13:43, Mauricio de Araujo 
 mauricio.de.ara...@gmail.com escreveu:


 Em 25 de julho de 2015 22:17, Martins Rama martin...@pop.com.br
 escreveu:

 Kacilda comparece ao trabalho julgando estar 10 minutos atrasada


 ​Acho que esta questão foi cancelada por conta do trecho Kacilda
 comparece ao trabalho julgando estar 10 minutos atrasada​ aqui temos
 incluídos os 5 minutos que ela já acredita que o relógio está atrasado ou
 não??



 --
 Abraços

 oɾnɐɹɐ ǝp oıɔıɹnɐɯ


 --
 Esta mensagem foi verificada pelo sistema de antivírus e
 acredita-se estar livre de perigo.



 --
 Esta mensagem foi verificada pelo sistema de antivírus e
 acredita-se estar livre de perigo.




-- 
Abraços

oɾnɐɹɐ ǝp oıɔıɹnɐɯ

-- 
Esta mensagem foi verificada pelo sistema de antiv�rus e
 acredita-se estar livre de perigo.



[obm-l] Re: [obm-l] Questão Colégio Militar - anulada

2015-07-26 Por tôpico Hermann
Eu acho que me enganei e a resposta é A

vamos supor que o correto seria chegar as 12h

ora para ela teria que chega 11h 55min (foi aí que me enganei)

se ela acha que está atrasada 10 minutos é porque o relogio dela marca 12h05 
e se ele está adiantado 15 minutos a hora correta é 11 h 50


Logo não vejo razão para terem anulado a questão.

abs
Hermann
  - Original Message - 
  From: Martins Rama 
  To: OBM-L 
  Sent: Saturday, July 25, 2015 10:17 PM
  Subject: [obm-l] Questão Colégio Militar - anulada


  Olá pessoal. Fiquei intrigado com essa questão quando minha filha me 
apresentou hoje. Foi um problema do concurso de admissão ao Colégio Militar de 
Brasília de 2008, 6a. série do Ensino Fundamental.
  A questão à época foi anulada. Alguém saberia argumentar o porquê? Há algum 
erro na lógica apresentada?

  Kacilda pensa que seu relógio está 5 minutos atrasado. Este, porém, está 15 
minutos adiantado. Com base em seu relógio, Kacilda comparece ao trabalho 
julgando estar 10 minutos atrasada. Na realidade, Kacilda chegou:
  a) 10 minutos adiantada.
  b) na hora certa.
  c) 10 minutos atrasada.
  d) 20 minutos adiantada.
  e) 15 minutos atrasada.

  Abraço,
  Martins Rama.

  -- 
  Esta mensagem foi verificada pelo sistema de antiv�rus e 
  acredita-se estar livre de perigo. 
-- 
Esta mensagem foi verificada pelo sistema de antiv�rus e
 acredita-se estar livre de perigo.



[obm-l] Re: [obm-l] Re: [obm-l] Questão Colégio Militar - anulada

2015-07-26 Por tôpico Ralph Teixeira
P.S.: Oops, erro tipografico: troque aquele 10 minutos do finalzinho por
15. O resto estah ok: sao, de fato, 11h50m.

2015-07-26 14:18 GMT-03:00 Ralph Teixeira ralp...@gmail.com:

 Para mim, letra (a).

 O problema eh esquisito porque a lingua portuguesa parece esquisita
 Afinal:

 Suponha que voce tem um encontro na hora x.
 Se voce estah ATRASADO y minutos, voce chegou na hora x+y.
 Se voce estah ADIANTADO y minutos, voce chegou na hora x-y.

 A principio, a linguagem PARECE funcionar ao contrario para relogios.
 Afinal, suponha que a HORA CERTA eh x.
 Se um relogio estah ATRASADO y minutos, ele marca x-y.
 Se um relogio estah ADIANTADO y minutos, ele marca x+y.

 Mas eh soh mudar o ponto de referencia e a analogia de relogios com
 pessoas fica clara. Agora suponha que a HORA MARCADA NO RELOGIO eh x.
 Se o relogio estah ATRASADO y minutos, a hora correta eh x+y.
 Se o relogio estah ADIANTADO y minutos, a hora correta eh x-y.

 Enfim, a vantagem dessa convencao eh que relogios ATRASADOS fazem seus
 donos chegarem ATRASADOS nos eventos. Faz sentido.

 ---///---

 Entao, como o Hermann falou: suponha que ela devia chegar 12h. Ela ACHA
 que sao 12h10m. Como ela PENSA que o relogio estah atrasado 5 minutos, o
 relogio mostra 12h05m...

 Mas isto tudo foi no mundo dela! De fato, o relogio marca 12h05m e estah
 ADIANTADO 10 minutos. Sao, de fato, 11h50m. Concordo, letra (a).

 Quem disse que relogio atrasado nao adianta?

 Ralph.

 2015-07-26 12:51 GMT-03:00 Martins Rama martin...@pop.com.br:

 Hermann, você apresentou justamente a discussão que tive com minha filha.
 Eu e ela chegamos a respostas diferentes: A ou B? e ambos tínhamos
 certeza que estávamos certos.
 Por isso achei a questão intrigante e resolvi postar.

 Talvez, por essa possível dupla interpretação, a banca do concurso achou
 melhor anular.

 Mais alguém do grupo pode contribuir com a análise?

 Abraços,
 Martins Rama.




 --
 Esta mensagem foi verificada pelo sistema de antivírus e
 acredita-se estar livre de perigo.




-- 
Esta mensagem foi verificada pelo sistema de antiv�rus e
 acredita-se estar livre de perigo.



[obm-l] Re: [obm-l] Re: [obm-l] Questão Colégio Militar - anulada

2015-07-26 Por tôpico Ralph Teixeira
Para mim, letra (a).

O problema eh esquisito porque a lingua portuguesa parece esquisita
Afinal:

Suponha que voce tem um encontro na hora x.
Se voce estah ATRASADO y minutos, voce chegou na hora x+y.
Se voce estah ADIANTADO y minutos, voce chegou na hora x-y.

A principio, a linguagem PARECE funcionar ao contrario para relogios.
Afinal, suponha que a HORA CERTA eh x.
Se um relogio estah ATRASADO y minutos, ele marca x-y.
Se um relogio estah ADIANTADO y minutos, ele marca x+y.

Mas eh soh mudar o ponto de referencia e a analogia de relogios com pessoas
fica clara. Agora suponha que a HORA MARCADA NO RELOGIO eh x.
Se o relogio estah ATRASADO y minutos, a hora correta eh x+y.
Se o relogio estah ADIANTADO y minutos, a hora correta eh x-y.

Enfim, a vantagem dessa convencao eh que relogios ATRASADOS fazem seus
donos chegarem ATRASADOS nos eventos. Faz sentido.

---///---

Entao, como o Hermann falou: suponha que ela devia chegar 12h. Ela ACHA que
sao 12h10m. Como ela PENSA que o relogio estah atrasado 5 minutos, o
relogio mostra 12h05m...

Mas isto tudo foi no mundo dela! De fato, o relogio marca 12h05m e estah
ADIANTADO 10 minutos. Sao, de fato, 11h50m. Concordo, letra (a).

Quem disse que relogio atrasado nao adianta?

Ralph.

2015-07-26 12:51 GMT-03:00 Martins Rama martin...@pop.com.br:

 Hermann, você apresentou justamente a discussão que tive com minha filha.
 Eu e ela chegamos a respostas diferentes: A ou B? e ambos tínhamos certeza
 que estávamos certos.
 Por isso achei a questão intrigante e resolvi postar.

 Talvez, por essa possível dupla interpretação, a banca do concurso achou
 melhor anular.

 Mais alguém do grupo pode contribuir com a análise?

 Abraços,
 Martins Rama.




 --
 Esta mensagem foi verificada pelo sistema de antivírus e
 acredita-se estar livre de perigo.


-- 
Esta mensagem foi verificada pelo sistema de antiv�rus e
 acredita-se estar livre de perigo.



[obm-l] Re: [obm-l] Questão Colégio Militar - anulada

2015-07-26 Por tôpico Hermann
Eu acho que a resposta certa seria letra b

vamos supor que o correto seria chegar as 12h

ora para ela teria que chega 12h 5min

se ela acha que está atrasada 10 minutos é porque o relogio dela marca 12h15

e se ele está adiantado 15 minutos a hora correta é 12 h


Logo não vejo razão para terem anulado a questão.

abs
Hermann
  - Original Message -
  From: Martins Rama
  To: OBM-L
  Sent: Saturday, July 25, 2015 10:17 PM
  Subject: [obm-l] Questão Colégio Militar - anulada


  Olá pessoal. Fiquei intrigado com essa questão quando minha filha me 
apresentou hoje. Foi um problema do concurso de admissão ao Colégio Militar de 
Brasília de 2008, 6a. série do Ensino Fundamental.
  A questão à época foi anulada. Alguém saberia argumentar o porquê? Há algum 
erro na lógica apresentada?

  Kacilda pensa que seu relógio está 5 minutos atrasado. Este, porém, está 15 
minutos adiantado. Com base em seu relógio, Kacilda comparece ao trabalho 
julgando estar 10 minutos atrasada. Na realidade, Kacilda chegou:
  a) 10 minutos adiantada.
  b) na hora certa.
  c) 10 minutos atrasada.
  d) 20 minutos adiantada.
  e) 15 minutos atrasada.

  Abraço,
  Martins Rama.

  --
  Esta mensagem foi verificada pelo sistema de antiv�rus e
  acredita-se estar livre de perigo.

---
Este email foi escaneado pelo Avast antivírus.
https://www.avast.com/antivirus

-- 
Esta mensagem foi verificada pelo sistema de antiv�rus e
 acredita-se estar livre de perigo.



[obm-l] Re: [obm-l] Questão Colégio Militar - anulada

2015-07-26 Por tôpico Martins Rama

Hermann, você apresentou justamente a discussão que tive com minha filha.
Eu e ela chegamos a respostas diferentes: A ou B? e ambos tínhamos certeza
que estávamos certos.
Por isso achei a questão intrigante e resolvi postar.

Talvez, por essa possível dupla interpretação, a banca do concurso achou
melhor anular.

Mais alguém do grupo pode contribuir com a análise?

Abraços,
Martins Rama.

--
Esta mensagem foi verificada pelo sistema de antiv�rus e
acredita-se estar livre de perigo.



[obm-l] Re: [obm-l] Re: [obm-l] Questão Colégio Militar - anulada

2015-07-26 Por tôpico Pedro José
Bom dia!

Também não vejo uma razão para a anulação.
Porém se o relógio está 15 min adiantado e Kacilda acha que está 5min
atrasado. A referência Kacilda está 20min adiantada. Se ela julga estar 10
min atrasada na verdade ela está 10 min adiantada. Letra a).

Sds,
PJMS

Em 26 de julho de 2015 12:12, Hermann ilhadepaqu...@bol.com.br escreveu:

  Eu acho que a resposta certa seria letra b

 vamos supor que o correto seria chegar as 12h

 ora para ela teria que chega 12h 5min

 se ela acha que está atrasada 10 minutos é porque o relogio dela marca
 12h15

 e se ele está adiantado 15 minutos a hora correta é 12 h


 Logo não vejo razão para terem anulado a questão.

 abs
 Hermann

 - Original Message -
 *From:* Martins Rama martin...@pop.com.br
 *To:* OBM-L obm-l@mat.puc-rio.br
 *Sent:* Saturday, July 25, 2015 10:17 PM
 *Subject:* [obm-l] Questão Colégio Militar - anulada

 Olá pessoal. Fiquei intrigado com essa questão quando minha filha me
 apresentou hoje. Foi um problema do concurso de admissão ao Colégio Militar
 de Brasília de 2008, 6a. série do Ensino Fundamental.
 A questão à época foi anulada. Alguém saberia argumentar o porquê? Há
 algum erro na lógica apresentada?

 Kacilda pensa que seu relógio está 5 minutos atrasado. Este, porém, está
 15 minutos adiantado. Com base em seu relógio, Kacilda comparece ao
 trabalho julgando estar 10 minutos atrasada. Na realidade, Kacilda chegou:
 a) 10 minutos adiantada.
 b) na hora certa.
 c) 10 minutos atrasada.
 d) 20 minutos adiantada.
 e) 15 minutos atrasada.

 Abraço,
 Martins Rama.

 --
 Esta mensagem foi verificada pelo sistema de antiv�rus e
 acredita-se estar livre de perigo.



 --
   [image: Avast logo] https://www.avast.com/antivirus

 Este email foi escaneado pelo Avast antivírus.
 www.avast.com https://www.avast.com/antivirus


 --
 Esta mensagem foi verificada pelo sistema de antivírus e
 acredita-se estar livre de perigo.


-- 
Esta mensagem foi verificada pelo sistema de antiv�rus e
 acredita-se estar livre de perigo.



[obm-l] Re: [obm-l] Questão simples

2015-06-11 Por tôpico Pedro José
Boa tarde!

Corrigindo,

a resposta do gabarito está correta colocando o fator 10^5 para fora da
expressão, ´
q = 777*( 10^995+ 10^889+...+ 10^11 + 10^5) +77
q = 777*10^5* ( 10^990+ 10^889+...+ 10^6 + 1) +77

a última parcela será 1. Portanto o B está correto

Serão 166,  777000, seguidos da sequência  77 ou 777*B*10^5 + 77, com B
igual ao proposto no gabarito.

Saudações.


Em 11 de junho de 2015 09:58, Pedro José petroc...@gmail.com escreveu:

 Bom dia!

 O final do texto deu erro na formatação. O correto está abaixo:

 como mdc(9,1001) =1 existe 9^-1 (mod1001) onde 9^-1 ≡ 445 (mod1001)
 se 9 não dividisse, bastava multiplicar por 445 dos dois lados e a ≡ 445
 *7*(10^5-1)≡ 700 (mod1001)

 Saudações,
 PJMS

 Em 11 de junho de 2015 09:54, Pedro José petroc...@gmail.com escreveu:

 Seja um número da forma 1000..01 com n algarismos zeros, e multiplicarmos
 por um número na forma aaa.a com n+1 algarismos. Teremos como resultado
 ...a com 2*(n+1) algarismos.

 Portanto, 777 = 1001*777
 logo A = 1001*777 ( 10^995+ 10^889+...+ 10^11 + 10^5) + 7

 o resto será o resto da divisão de 7 por 1001, como 7 = 77*1001
 +700

 podemos escrever A= 1001*(777 * ( 10^995+ 10^889+...+ 10^11 + 10^5) +77)
 +700

 Como 0700 1001 == r = 700

 Já q = 777*( 10^995+ 10^889+...+ 10^11 + 10^5) +77

 Que darão 166 777000 na sequência, seguidos de três algarismos zero e
 dois algarismos 7, ou seja,

 q =77700070...7770077 166 pois (995 - 5)/6+1

 ou de outra maneira colando 777 em evidência q = 777*
 (1010...101000)*10^5 +77

 A resposta deveria ter no final do número B a seguinte sequência de
 algarismos 10 ao invés do algarismo 1 destacado em amarelo. ou então
 usar B da forma exposta e corrigir a potência de 10 em q de 5 para 8.
  q = 777*B*10^8 + 77

 Saudações,
 PJMS

 Para achar o resto dava para usar mod., mas para o quociente creio que
 não.

 A = 7 + 7*10^1 + 7*10^2 +...+ 7*10^999 + 7*10^1000

 Por soma da PG A = 7*(10^1001-1)/9
 9A  ≡ 7* (10^1001-1) (mod1001)
 como 10^6 ≡ 1 (mod 1001)

 temos que 9a ≡ 7*(10^5-1) (mod1001)

 como 9 | (10^5 -1) (| significa divide) pode-se: a ≡ 7*(1) (mod1001)
 == a ≡ 700 (mod1001)

 se 9 não dividisse, como mdc(9,1001) =1 existe 9^-1 (mod1001) onde 9^-1 ≡
 445 (mod1001)
 bastava multiplicar por 445 dos dois lados e a ≡ 445 *7*(10^5-1)≡ 700
 (mod1001)

 Em 9 de junho de 2015 22:03, marcone augusto araújo borges 
 marconeborge...@hotmail.com escreveu:

 Seja A = 777...77(1001 algarismos). Determine o quociente e o resto da
 divisão de A por 1001

 Eu achei o quociente 777000777000777000...00077 e resto 700
 o bloco 777000 reproduzido 111 vezes e mais 77 no final
 Mas o gabarito dá quociente 777.B.10^5 + 77, sendo B = 10101...1(166
  1`s )
 Não entendi  a resposta do gabarito.
 Outra coisa: daria pra achar o resto usando congruência?

 --
 Esta mensagem foi verificada pelo sistema de antivírus e
 acredita-se estar livre de perigo.





-- 
Esta mensagem foi verificada pelo sistema de antiv�rus e
 acredita-se estar livre de perigo.



[obm-l] Re: [obm-l] Questão simples

2015-06-11 Por tôpico Pedro José
Seja um número da forma 1000..01 com n algarismos zeros, e multiplicarmos
por um número na forma aaa.a com n+1 algarismos. Teremos como resultado
...a com 2*(n+1) algarismos.

Portanto, 777 = 1001*777
logo A = 1001*777 ( 10^995+ 10^889+...+ 10^11 + 10^5) + 7

o resto será o resto da divisão de 7 por 1001, como 7 = 77*1001 +700

podemos escrever A= 1001*(777 * ( 10^995+ 10^889+...+ 10^11 + 10^5) +77)
+700

Como 0700 1001 == r = 700

Já q = 777*( 10^995+ 10^889+...+ 10^11 + 10^5) +77

Que darão 166 777000 na sequência, seguidos de três algarismos zero e dois
algarismos 7, ou seja,

q =77700070...7770077 166 pois (995 - 5)/6+1

ou de outra maneira colando 777 em evidência q = 777*
(1010...101000)*10^5 +77

A resposta deveria ter no final do número B a seguinte sequência de
algarismos 10 ao invés do algarismo 1 destacado em amarelo. ou então
usar B da forma exposta e corrigir a potência de 10 em q de 5 para 8.
 q = 777*B*10^8 + 77

Saudações,
PJMS

Para achar o resto dava para usar mod., mas para o quociente creio que não.

A = 7 + 7*10^1 + 7*10^2 +...+ 7*10^999 + 7*10^1000

Por soma da PG A = 7*(10^1001-1)/9
9A  ≡ 7* (10^1001-1) (mod1001)
como 10^6 ≡ 1 (mod 1001)

temos que 9a ≡ 7*(10^5-1) (mod1001)

como 9 | (10^5 -1) (| significa divide) pode-se: a ≡ 7*(1) (mod1001)
== a ≡ 700 (mod1001)

se 9 não dividisse, como mdc(9,1001) =1 existe 9^-1 (mod1001) onde 9^-1 ≡ 445
(mod1001)
bastava multiplicar por 445 dos dois lados e a ≡ 445 *7*(10^5-1)≡ 700
(mod1001)

Em 9 de junho de 2015 22:03, marcone augusto araújo borges 
marconeborge...@hotmail.com escreveu:

 Seja A = 777...77(1001 algarismos). Determine o quociente e o resto da
 divisão de A por 1001

 Eu achei o quociente 777000777000777000...00077 e resto 700
 o bloco 777000 reproduzido 111 vezes e mais 77 no final
 Mas o gabarito dá quociente 777.B.10^5 + 77, sendo B = 10101...1(166
  1`s )
 Não entendi  a resposta do gabarito.
 Outra coisa: daria pra achar o resto usando congruência?

 --
 Esta mensagem foi verificada pelo sistema de antivírus e
 acredita-se estar livre de perigo.


-- 
Esta mensagem foi verificada pelo sistema de antiv�rus e
 acredita-se estar livre de perigo.



[obm-l] Re: [obm-l] Questão simples

2015-06-11 Por tôpico Pedro José
Bom dia!

O final do texto deu erro na formatação. O correto está abaixo:

como mdc(9,1001) =1 existe 9^-1 (mod1001) onde 9^-1 ≡ 445 (mod1001)
se 9 não dividisse, bastava multiplicar por 445 dos dois lados e a ≡ 445
*7*(10^5-1)≡ 700 (mod1001)

Saudações,
PJMS

Em 11 de junho de 2015 09:54, Pedro José petroc...@gmail.com escreveu:

 Seja um número da forma 1000..01 com n algarismos zeros, e multiplicarmos
 por um número na forma aaa.a com n+1 algarismos. Teremos como resultado
 ...a com 2*(n+1) algarismos.

 Portanto, 777 = 1001*777
 logo A = 1001*777 ( 10^995+ 10^889+...+ 10^11 + 10^5) + 7

 o resto será o resto da divisão de 7 por 1001, como 7 = 77*1001
 +700

 podemos escrever A= 1001*(777 * ( 10^995+ 10^889+...+ 10^11 + 10^5) +77)
 +700

 Como 0700 1001 == r = 700

 Já q = 777*( 10^995+ 10^889+...+ 10^11 + 10^5) +77

 Que darão 166 777000 na sequência, seguidos de três algarismos zero e dois
 algarismos 7, ou seja,

 q =77700070...7770077 166 pois (995 - 5)/6+1

 ou de outra maneira colando 777 em evidência q = 777*
 (1010...101000)*10^5 +77

 A resposta deveria ter no final do número B a seguinte sequência de
 algarismos 10 ao invés do algarismo 1 destacado em amarelo. ou então
 usar B da forma exposta e corrigir a potência de 10 em q de 5 para 8.
  q = 777*B*10^8 + 77

 Saudações,
 PJMS

 Para achar o resto dava para usar mod., mas para o quociente creio que não.

 A = 7 + 7*10^1 + 7*10^2 +...+ 7*10^999 + 7*10^1000

 Por soma da PG A = 7*(10^1001-1)/9
 9A  ≡ 7* (10^1001-1) (mod1001)
 como 10^6 ≡ 1 (mod 1001)

 temos que 9a ≡ 7*(10^5-1) (mod1001)

 como 9 | (10^5 -1) (| significa divide) pode-se: a ≡ 7*(1) (mod1001)
 == a ≡ 700 (mod1001)

 se 9 não dividisse, como mdc(9,1001) =1 existe 9^-1 (mod1001) onde 9^-1 ≡ 445
 (mod1001)
 bastava multiplicar por 445 dos dois lados e a ≡ 445 *7*(10^5-1)≡ 700
 (mod1001)

 Em 9 de junho de 2015 22:03, marcone augusto araújo borges 
 marconeborge...@hotmail.com escreveu:

 Seja A = 777...77(1001 algarismos). Determine o quociente e o resto da
 divisão de A por 1001

 Eu achei o quociente 777000777000777000...00077 e resto 700
 o bloco 777000 reproduzido 111 vezes e mais 77 no final
 Mas o gabarito dá quociente 777.B.10^5 + 77, sendo B = 10101...1(166
  1`s )
 Não entendi  a resposta do gabarito.
 Outra coisa: daria pra achar o resto usando congruência?

 --
 Esta mensagem foi verificada pelo sistema de antivírus e
 acredita-se estar livre de perigo.




-- 
Esta mensagem foi verificada pelo sistema de antiv�rus e
 acredita-se estar livre de perigo.



[obm-l] Re: [obm-l] Questão interessante

2015-02-22 Por tôpico saulo nilson
x-r+x+x+r=180
x=60
(y-w)a+y(b)+(y+w)c=acrq3/2
b^2=a^2+c^2-ac
sen(60-r)=h1/b


2015-02-21 13:39 GMT-02:00 marcone augusto araújo borges 
marconeborge...@hotmail.com:

  Espero que alguém goste assim como eu gostei:

 As medidas dos ângulos internos de um triângulo estão em PA e as medidas
 das alturas do mesmo triângulo estão em PA.Prove que o triângulo é
 equilátero.

 --
 Esta mensagem foi verificada pelo sistema de antivírus e
 acredita-se estar livre de perigo.


-- 
Esta mensagem foi verificada pelo sistema de antiv�rus e
 acredita-se estar livre de perigo.



[obm-l] Re: [obm-l] Questão interessante

2015-02-22 Por tôpico saulo nilson
x-r+x+x+r=180
x=60
(y-w)a+y(b)+(y+w)c=acrq3/2
b^2=a^2+c^2-ac
sen(60-r)=h2/b=h3/a
sen(60+r)=h1/b=h3/c
h3/h2=a/b
h3/h1=c/b
h1/h2=a/c
(h3-h2)/h2=(a-b)/b
(h2-h1)/h1=(c-a)/a
w/h2=(a-b)/b
w/h1=(c-a)/a
h1/h2=(a-b)a/(c-a)b=a/c
(c-a)b=(a-b)c
cb-ab=ac-bc
2bc=ac+ab
b^2=a^2+c^2-ac
b^2=4b^2c^2/(b+c)^2 +c^2-2bc^2/(b+c)
b^2/c^2 (b+c)^2=4b^2+b^2+c^2 -2b^2=3b^2+c^2
b^2(b^2+2bc+c^2)=c^4+3b^2c^2
b^4+2b^3c=c^4+2b^2c^2
b=c uma das respostas
logo a=b=c triângulo equilátero

-02-22 15:26 GMT-03:00 saulo nilson saulo.nil...@gmail.com:

 x-r+x+x+r=180
 x=60
 (y-w)a+y(b)+(y+w)c=acrq3/2
 b^2=a^2+c^2-ac
 sen(60-r)=h1/b


 2015-02-21 13:39 GMT-02:00 marcone augusto araújo borges 
 marconeborge...@hotmail.com:

  Espero que alguém goste assim como eu gostei:

 As medidas dos ângulos internos de um triângulo estão em PA e as medidas
 das alturas do mesmo triângulo estão em PA.Prove que o triângulo é
 equilátero.

 --
 Esta mensagem foi verificada pelo sistema de antivírus e
 acredita-se estar livre de perigo.




-- 
Esta mensagem foi verificada pelo sistema de antiv�rus e
 acredita-se estar livre de perigo.



[obm-l] Re: [obm-l] Questão simples

2015-02-09 Por tôpico Bernardo Freitas Paulo da Costa
2015-02-09 0:49 GMT-02:00 marcone augusto araújo borges
marconeborge...@hotmail.com:
 Eu não conheço o project Euler, Bernardo.
 Dei uma olhadinha bem rápida depois da sua citação.
 Essa questão eu formulei porque  um colega mandou uma
 mensagem com uma brincadeira dizendo que ´´às vezes a sorte
 ajuda´´1/4 = 16/64(cancelando os seis).Eu notei que valia para
 outros números como 19/95 e tentei uma explicação.como não
 obtive sucesso eu recorri à lista.
Vou dar a dica básica, que você já pensou no resto: um número ab é
na verdade (10a + b). Agora, você pode cancelar dígitos em 2
posições em cima e 2 embaixo. Cada uma dessas 2*2 = 4
possibilidades dá uma equação com a,b,c,d. Depois, se não me falha a
memória, é na força bruta mesmo...

Abraços,
-- 
Bernardo Freitas Paulo da Costa

-- 
Esta mensagem foi verificada pelo sistema de antiv�rus e
 acredita-se estar livre de perigo.


=
Instru��es para entrar na lista, sair da lista e usar a lista em
http://www.mat.puc-rio.br/~obmlistas/obm-l.html
=


[obm-l] Re: [obm-l] Questão simples

2015-02-08 Por tôpico Bernardo Freitas Paulo da Costa
2015-02-08 21:14 GMT-02:00 Bernardo Freitas Paulo da Costa
bernardo...@gmail.com:
 2015-02-07 14:07 GMT-02:00 marcone augusto araújo borges
 marconeborge...@hotmail.com:
 16/64 = 1/4(´´cancelando´´ 6 com 6´) e 19/95 = 1/5(´´cancelando´´ 9 com 9)
 Quais são os números ab e bc tais que ab/bc = a/c ?
 Essa questão é do Project Euler. Não respondam...

Me apressei, a questão é MUITO POUCO diferente. Marcone: seja mais
preciso com a sua pergunta (e o project Euler já dá uma dica para
você), por exemplo dizendo o que você já tentou fazer nela.

Ou, se é para compartilhar um problema que você achou legal, seja
explícito que este é o caso, pois é muito raro... (não que faltem
problemas legais na lista, mas falta um pouco a iniciativa de pessoas
como o Artur que postavam problemas apenas para que nós
aproveitássemos)

Abraços,
-- 
Bernardo Freitas Paulo da Costa

-- 
Esta mensagem foi verificada pelo sistema de antiv�rus e
 acredita-se estar livre de perigo.


=
Instru��es para entrar na lista, sair da lista e usar a lista em
http://www.mat.puc-rio.br/~obmlistas/obm-l.html
=


[obm-l] Re: [obm-l] Questão simples

2015-02-08 Por tôpico Bernardo Freitas Paulo da Costa
2015-02-07 14:07 GMT-02:00 marcone augusto araújo borges
marconeborge...@hotmail.com:
 16/64 = 1/4(´´cancelando´´ 6 com 6´) e 19/95 = 1/5(´´cancelando´´ 9 com 9)
 Quais são os números ab e bc tais que ab/bc = a/c ?
Essa questão é do Project Euler. Não respondam...

-- 
Bernardo Freitas Paulo da Costa

-- 
Esta mensagem foi verificada pelo sistema de antiv�rus e
 acredita-se estar livre de perigo.


=
Instru��es para entrar na lista, sair da lista e usar a lista em
http://www.mat.puc-rio.br/~obmlistas/obm-l.html
=


[obm-l] Re: [obm-l] Questão de probabilidade

2015-01-18 Por tôpico Hermann
Meu amigo Pedro, onde foi que você encontrou esse exercício?
Na minha ignorância, e sei que sou muito ignorante!, não acredito que isso 
possa ter uma resposta.
Mas se tiver resposta, devo aprender, com ela, muita matemática.
Abraços Hermann
  - Original Message - 
  From: Pedro Chaves 
  To: obm-l@mat.puc-rio.br 
  Sent: Sunday, January 18, 2015 11:25 AM
  Subject: [obm-l] Questão de probabilidade


  Caros Colegas,

  Dados três números reais positivos a, b e c, com a   b   c, qual  é a 
probabilidade de que se tenha a  b + c ?

  Abraços do Pedro Chaves.
  _





  -- 
  Esta mensagem foi verificada pelo sistema de antivírus e 
  acredita-se estar livre de perigo. 
-- 
Esta mensagem foi verificada pelo sistema de antivírus e
 acredita-se estar livre de perigo.



[obm-l] Re: [obm-l] Re: [obm-l] Questão de probabilidade

2015-01-18 Por tôpico Ralph Teixeira
Eh, o Hermann tem razao, nao existe uma distribuicao de probabilidade nos
reais positivos que funcione bem. O problema eh que existem varias
maneiras de escolher um numero real positivo aleatoriamente, nenhuma
delas completamente padrao, e elas dariam respostas diferentes para seu
problema.
---///---
Por exemplo, UMA maneira de adaptar sua pergunta eh a seguinte:

1. Seja dado um N real positivo. Escolha x, y e z aleatoriamente e
independentemente, com distribuicao uniforme, no intervalo [0,N]. Sabendo
que z=y=x, qual a probabilidade de termos zx+y?
2. O que acontece com esta probabilidade quando N-+Inf?

Geometricamente: voce escolhe um ponto (x,y,z) aleatoriamente
(uniformemente) dentro da regiao R:0=x=y=z=N; qual a chance de ele
estar na sub-regiao S:zx+y? O uniformemente indica que probabilidades
sao proporcionais a volumes.

Vejamos: R eh um tetraedro de vertices (0,0,0), (0,0,N), (0,N,N) e (N,N,N).
O plano P:z=x+y passa pelos vertices (0,0,0) e (0,N,N) e corta a aresta que
vai de (0,0,N) a (N,N,N) exatamente ao meio, em (N/2,N/2,N). Entao P corta
R em dois pedacos de mesmo volume (pense em P como base dos tetraedros S e
R-S; a area da base eh a mesma, e as distancias de (0,0,0) e de (N,N,N) a P
sao iguais)! Isto eh: vol(S)=vol(R)/2. Assim, dado um ponto escolhido
uniformemente em R, a probabilidade de ele estar em S eh 1/2.
---///---
Outra maneira:
Tome uma vareta de comprimento L, e escolha nela dois pontos
independentemente, de maneira aleatoria uniforme. Quebrando a vareta nestes
dois pontos, qual a probabilidade de elas serem os lados de um triangulo?

Esta jah apareceu na lista:
http://www.mat.puc-rio.br/~nicolau/olimp/obm-l.200706/msg00182.html. Agora
dah 25%. :P

(Alias, vide tambem
http://www.mail-archive.com/obm-l%40mat.puc-rio.br/msg41301.html)

Abraco, Ralph.

2015-01-18 12:41 GMT-02:00 Hermann ilhadepaqu...@bol.com.br:

  Meu amigo Pedro, onde foi que você encontrou esse exercício?
 Na minha ignorância, e sei que sou muito ignorante!, não acredito que isso
 possa ter uma resposta.
 Mas se tiver resposta, devo aprender, com ela, muita matemática.
 Abraços Hermann

 - Original Message -
 *From:* Pedro Chaves brped...@hotmail.com
 *To:* obm-l@mat.puc-rio.br
 *Sent:* Sunday, January 18, 2015 11:25 AM
 *Subject:* [obm-l] Questão de probabilidade

 Caros Colegas,

 Dados três números reais positivos a, b e c, com a **  b **  c, qual  é
 a probabilidade de que se tenha a  b + c ?

 Abraços do Pedro Chaves.
 _




 --
 Esta mensagem foi verificada pelo sistema de antivírus e
 acredita-se estar livre de perigo.


 --
 Esta mensagem foi verificada pelo sistema de antivírus e
 acredita-se estar livre de perigo.


-- 
Esta mensagem foi verificada pelo sistema de antiv�rus e
 acredita-se estar livre de perigo.



[obm-l] Re: [obm-l] Questão de probabilidade (reformulação)

2015-01-18 Por tôpico Ralph Teixeira
Ainda existe o problema: na hora de escolher uma solucao (x,y,z), qual a
distribuicao de probabilidade a ser utilizada? Mas, agora que voce
restringiu o problema, a interpretacao mais natural leva aa mesma solucao
daquele de dividir uma vareta que eu pus no meu link, de uma olhada lah.

Abraco, Ralph.

2015-01-18 14:45 GMT-02:00 Pedro Chaves brped...@hotmail.com:

 Creio que formulei mal a pergunta.
 Eu queria dizer o seguinte:

 Dada a equação x + y + z = S, onde x, y e z são variáveis reais positivas,
 com x=y=z, e S é uma constante real positiva, qual é a probabilidade de
 que se tenha a  b +c, quando se escolhe aleatoriamente uma solução (a, b,
 c) dessa equação?


 
  From: brped...@hotmail.com
  To: obm-l@mat.puc-rio.br
  Subject: [obm-l] Questão de probabilidade (correção)
  Date: Sun, 18 Jan 2015 16:45:16 +0300
 
  Eu quis dizer: a = b = c . ___ From:
  brped...@hotmail.com To: obm-l@mat.puc-rio.br Subject: Questão de
  probabilidade Date: Sun, 18 Jan 2015 16:25:53 +0300 Caros Colegas,
  Dados três números reais positivos a, b e c, com a b c, qual é a
  probabilidade de que se tenha a  b + c ? Abraços do Pedro Chaves.
  _
 
  --
  Esta mensagem foi verificada pelo sistema de antivírus e
  acredita-se estar livre de perigo.

 --
 Esta mensagem foi verificada pelo sistema de antivírus e
  acredita-se estar livre de perigo.


 =
 Instruções para entrar na lista, sair da lista e usar a lista em
 http://www.mat.puc-rio.br/~obmlistas/obm-l.html
 =


-- 
Esta mensagem foi verificada pelo sistema de antiv�rus e
 acredita-se estar livre de perigo.



[obm-l] Re: [obm-l] Re: [obm-l] Questão da 3ª fase nível 1 da OBM 2013

2014-04-26 Por tôpico saulo nilson
222  4

1  6  12

 9  18  3


b)
8 412

 2   1 3

  105 15






2014-04-23 23:13 GMT-03:00 Willy George Amaral Petrenko 
wgapetre...@gmail.com:

 vc quer uma ajuda ou uma solução?

 Uma ajuda:

 a) Observe que 22 não tem muitos divisores próprios, apenas 1,2 e 11.
 Mostre que se 11 fizer parte do quadrado, então algum outro múltiplo de 11
 além do 22 também estará (ou seja, não existe quadrado onde os únicos
 múltiplos de 11 sejam 11 e 22). Assim se 22 for o maior, 11 não pode estar
 no quadrado.

 Portanto 22 deverá estar ao lado de 1 e 2, logo deve estar no canto:

 22  1  x
 2   x   x
 x   x   x

 Agora eu fiz por tentativa e erro.

 b) Generalize a observação que eu fiz para um primo qualquer (ou seja se p
 está no quadrado, então outro múltiplo de p que não é o 2p também estará).
 Tente achar um quadrado cujo maior número seja o menor que conseguir e use
 a observação para mostrar que não existe nenhum menor.

 Se não conseguir fazer eu posso mandar a solução.


 2014-04-23 18:37 GMT-03:00 Érica G. Pongelupe Giacoia profer...@ig.com.br
 :



 Prezados,
 Gostaria da ajuda de vcs para resolver a seguinte questão que caiu na 3ª
 fase (nível 1) da OBM do ano passado.

 Desde já obrigada a todos.

 Érica G. P. Giacoia

 (OBM 2013) Desejamos preencher um tabuleiro 3x3 com 9 inteiros positivos
 distintos sendo que números a e b que têm um lado em comum devem ser tais
 que a é divisível por b ou b é divisível por a.

 Vejamos uma configuração que satisfaz as condições do problema. Observe
 que o maior número que aparece no tabuleiro é o 25.

 8 2   10

 420   5

 12   1   25

 a) Apresente uma maneira de preencher um tabuleiro de modo que o maior
 número que aparece é  22.

 b) Qual é o menor inteiro positivo que pode ser o maior número que
 aparece no tabuleiro?



 --
 Esta mensagem foi verificada pelo sistema de antivírus e
 acredita-se estar livre de perigo.



 --
 Esta mensagem foi verificada pelo sistema de antivírus e
 acredita-se estar livre de perigo.


-- 
Esta mensagem foi verificada pelo sistema de antiv�rus e
 acredita-se estar livre de perigo.



[obm-l] Re: [obm-l] Questão da 3ª fase nível 1 da OBM 2013

2014-04-23 Por tôpico Willy George Amaral Petrenko
vc quer uma ajuda ou uma solução?

Uma ajuda:

a) Observe que 22 não tem muitos divisores próprios, apenas 1,2 e 11.
Mostre que se 11 fizer parte do quadrado, então algum outro múltiplo de 11
além do 22 também estará (ou seja, não existe quadrado onde os únicos
múltiplos de 11 sejam 11 e 22). Assim se 22 for o maior, 11 não pode estar
no quadrado.

Portanto 22 deverá estar ao lado de 1 e 2, logo deve estar no canto:

22  1  x
2   x   x
x   x   x

Agora eu fiz por tentativa e erro.

b) Generalize a observação que eu fiz para um primo qualquer (ou seja se p
está no quadrado, então outro múltiplo de p que não é o 2p também estará).
Tente achar um quadrado cujo maior número seja o menor que conseguir e use
a observação para mostrar que não existe nenhum menor.

Se não conseguir fazer eu posso mandar a solução.


2014-04-23 18:37 GMT-03:00 Érica G. Pongelupe Giacoia profer...@ig.com.br:



 Prezados,
 Gostaria da ajuda de vcs para resolver a seguinte questão que caiu na 3ª
 fase (nível 1) da OBM do ano passado.

 Desde já obrigada a todos.

 Érica G. P. Giacoia

 (OBM 2013) Desejamos preencher um tabuleiro 3x3 com 9 inteiros positivos
 distintos sendo que números a e b que têm um lado em comum devem ser tais
 que a é divisível por b ou b é divisível por a.

 Vejamos uma configuração que satisfaz as condições do problema. Observe
 que o maior número que aparece no tabuleiro é o 25.

 8 2   10

 420   5

 12   1   25

 a) Apresente uma maneira de preencher um tabuleiro de modo que o maior
 número que aparece é  22.

 b) Qual é o menor inteiro positivo que pode ser o maior número que aparece
 no tabuleiro?



 --
 Esta mensagem foi verificada pelo sistema de antivírus e
 acredita-se estar livre de perigo.


-- 
Esta mensagem foi verificada pelo sistema de antiv�rus e
 acredita-se estar livre de perigo.



[obm-l] Re: [obm-l] Questão do IME

2013-10-29 Por tôpico Mauricio de Araujo
veja a solução em

https://www.dropbox.com/s/3wpkb4ht01oidsz/foto%205.PNG


2013/10/29 marcone augusto araújo borges marconeborge...@hotmail.com

 Sejam x = 370370370...37(89 algarismos) e y = 111...1000...0(30 algarismos
 1 e 30 algarismos 0)
 Calcule (x - y)^1/3

 --
 Esta mensagem foi verificada pelo sistema de antivírus e
 acredita-se estar livre de perigo.




-- 
Abraços

oɾnɐɹɐ ǝp oıɔıɹnɐɯ
*momentos excepcionais pedem ações excepcionais.*
*Os cemitérios estão cheios de pessoas insubstituíveis em seus ofícios.*

-- 
Esta mensagem foi verificada pelo sistema de antiv�rus e
 acredita-se estar livre de perigo.



[obm-l] Re: [obm-l] Re: [obm-l] Questão do IME

2013-10-29 Por tôpico Vanderlei Nemitz
Maurício:

Que livro é esse? O IME retirou a questão na íntegra.


Obrigado!


Em 29 de outubro de 2013 15:56, Mauricio de Araujo 
mauricio.de.ara...@gmail.com escreveu:

 veja a solução em

 https://www.dropbox.com/s/3wpkb4ht01oidsz/foto%205.PNG


 2013/10/29 marcone augusto araújo borges marconeborge...@hotmail.com

 Sejam x = 370370370...37(89 algarismos) e y = 111...1000...0(30
 algarismos 1 e 30 algarismos 0)
 Calcule (x - y)^1/3

 --
 Esta mensagem foi verificada pelo sistema de antivírus e
 acredita-se estar livre de perigo.




 --
 Abraços

 oɾnɐɹɐ ǝp oıɔıɹnɐɯ
 *momentos excepcionais pedem ações excepcionais.*
 *Os cemitérios estão cheios de pessoas insubstituíveis em seus ofícios.*

 --
 Esta mensagem foi verificada pelo sistema de antivírus e
 acredita-se estar livre de perigo.


-- 
Esta mensagem foi verificada pelo sistema de antiv�rus e
 acredita-se estar livre de perigo.



[obm-l] Re: [obm-l] Re: [obm-l] Re: [obm-l] Questão do IME

2013-10-29 Por tôpico Hermann
O autor é Suprun tem hoje a venda por um absurdo de preço na editora Vestseller
  - Original Message - 
  From: Vanderlei Nemitz 
  To: obm-l@mat.puc-rio.br 
  Sent: Tuesday, October 29, 2013 4:13 PM
  Subject: [obm-l] Re: [obm-l] Re: [obm-l] Questão do IME


  Maurício:

  Que livro é esse? O IME retirou a questão na íntegra.




  Obrigado!




  Em 29 de outubro de 2013 15:56, Mauricio de Araujo 
mauricio.de.ara...@gmail.com escreveu:

veja a solução em 


https://www.dropbox.com/s/3wpkb4ht01oidsz/foto%205.PNG




2013/10/29 marcone augusto araújo borges marconeborge...@hotmail.com

  Sejam x = 370370370...37(89 algarismos) e y = 111...1000...0(30 
algarismos 1 e 30 algarismos 0)
   
   
   
   
   
   
   
   
   
  Calcule (x - y)^1/3

  -- 
  Esta mensagem foi verificada pelo sistema de antivírus e 
  acredita-se estar livre de perigo. 





-- 

Abraços

oɾnɐɹɐ ǝp oıɔıɹnɐɯ
momentos excepcionais pedem ações excepcionais.

Os cemitérios estão cheios de pessoas insubstituíveis em seus ofícios.

-- 
Esta mensagem foi verificada pelo sistema de antivírus e 
acredita-se estar livre de perigo. 



  -- 
  Esta mensagem foi verificada pelo sistema de antiv�rus e 
  acredita-se estar livre de perigo. 
-- 
Esta mensagem foi verificada pelo sistema de antiv�rus e
 acredita-se estar livre de perigo.



[obm-l] Re: [obm-l] Re: [obm-l] Re: [obm-l] Re: [obm-l] Questão do IME

2013-10-29 Por tôpico Mauricio de Araujo
O IME tirou 4 questões da prova deste ano do referido livro do Suprun,
Ipsis litteris...

E a VestSeller aproveita para fazer a propaganda do livro pelo qual eles
cobram o olho da cara... pelo menos no marketing eles são bons (o dono é
engenheiro do ITA)...

e o povo do IME vai e ainda ajuda a valorizar o produto (que é bom
mesmo)... podiam ao menos dar uma alterada nos números da questão, sei lá...

De qq forma o livro está à venda (na verdade está em falta) na
Vestseller... veja o link

http://www.vestseller.com.br/detalhamento.asp?produto_id=471






2013/10/29 Hermann ilhadepaqu...@bol.com.br

 **
 O autor é Suprun tem hoje a venda por um absurdo de preço na
 editora Vestseller

 - Original Message -
 *From:* Vanderlei Nemitz vanderma...@gmail.com
 *To:* obm-l@mat.puc-rio.br
 *Sent:* Tuesday, October 29, 2013 4:13 PM
 *Subject:* [obm-l] Re: [obm-l] Re: [obm-l] Questão do IME

  Maurício:

 Que livro é esse? O IME retirou a questão na íntegra.


 Obrigado!


 Em 29 de outubro de 2013 15:56, Mauricio de Araujo 
 mauricio.de.ara...@gmail.com escreveu:

  veja a solução em

 https://www.dropbox.com/s/3wpkb4ht01oidsz/foto%205.PNG


 2013/10/29 marcone augusto araújo borges marconeborge...@hotmail.com

  Sejam x = 370370370...37(89 algarismos) e y = 111...1000...0(30
 algarismos 1 e 30 algarismos 0)
   Calcule (x - y)^1/3

 --
 Esta mensagem foi verificada pelo sistema de antivírus e
 acredita-se estar livre de perigo.




 --
  Abraços

 oɾnɐɹɐ ǝp oıɔıɹnɐɯ
 *momentos excepcionais pedem ações excepcionais.*
 *Os cemitérios estão cheios de pessoas insubstituíveis em seus ofícios.*

 --
 Esta mensagem foi verificada pelo sistema de antivírus e
 acredita-se estar livre de perigo.



 --
 Esta mensagem foi verificada pelo sistema de antiv�rus e
 acredita-se estar livre de perigo.


 --
 Esta mensagem foi verificada pelo sistema de antivírus e
 acredita-se estar livre de perigo.




-- 
Abraços

oɾnɐɹɐ ǝp oıɔıɹnɐɯ
*momentos excepcionais pedem ações excepcionais.*
*Os cemitérios estão cheios de pessoas insubstituíveis em seus ofícios.*

-- 
Esta mensagem foi verificada pelo sistema de antiv�rus e
 acredita-se estar livre de perigo.



[obm-l] Re: [obm-l] Re: [obm-l] Re: [obm-l] Re: [obm-l] Re: [obm-l] Questão do IME

2013-10-29 Por tôpico Leonardo Borges Avelino
O IME sempre teve costume de usar questões de livros famosos, como Lidski,
Caronnet e outros. Realmente é muito difícil ter acesso a estes livros (em
papel), pois são caros. Muitos tem como achar na internet em PDF, mas eu
sempre gostei de ter os livros.

Recomendo a seguinte engine de busca de livros para compra:
http://www.fetchbook.info/

Sempre fui muito feliz comprando da abebooks e da alibris, amazon e outros.
Olhando por este livro que indicaram (
http://www.fetchbook.info/compare.do?userId=1search=9785396001015destinationKey=BRAcurrencyKey=BRLSubmit=Update)
deu pra achar por 70 reais (livro +frete)

abs

-- 
Esta mensagem foi verificada pelo sistema de antiv�rus e
 acredita-se estar livre de perigo.



[obm-l] Re: [obm-l] Questão simples(equações polinomiais?)

2013-08-26 Por tôpico Ralph Teixeira
Eu tenho outra solução também na marra, mas de outro tipo: se você me dá
uma questão qualquer deste tipo com polinômios não exageradamente
horrorosos, o que eu tento fazer é dividir um polinômio pelo outro:

P(x)=x^8-7x^4+1=Q(x)(x^2+x-1)+R(x)

Algumas contas depois, temos R(x). Botando x=a=raiz de x^2+x-1, vem que
P(a)=R(a), que seria a resposta.

No seu caso, fazendo isso eu descobriria (surpreso!) que x^8-7x^4+1 pode
ser fatorado como (x^2+x-1)(x^6-x^5+2x^4-3x^3-2x^2-x-1), então R(a)=0.

--//--

Outra ideia mais mágica é partir para a fatoração mesmo completando
quadrados de maneira esperta:

x^8-7x^4+1=(x^8+2x^4+1-9x^4)=(x^4+1)^2-(3x^2)^2=(x^4-3x^2+1)(x^4+3x^2+1)

e de novo

x^4-3x^2+1=(x^4-2x^2+1)-x^2=(x^2-1)^2-x^2=(x^2+x-1)(x^2-x-1)

Então, puxa!, um é divisível pelo outro, a resposta é 0... Mas estou
roubando: só suspeitei que isso ia dar certo depois que eu vi a sua
resposta... :) :)

Abraço, Ralph.
On Aug 26, 2013 11:44 AM, marcone augusto araújo borges 
marconeborge...@hotmail.com wrote:

 Seja x um número que satisfaz a equação x^2 + x - 1 = 0,determine
 o valor da expressão x^8 - 7x^4 + 1

 Eu fiz ´´na marra´´.
 x^2  = 1 - x (1)
 Calculei x^4 e x^8(elevando ao quadrado)
 usei também x^3 = x -x^2 de (1)
 Encontrei zero como resposta
 Dever existir um modo mais interessante.
 Agradeço por um esclarecimento.



 --
 Esta mensagem foi verificada pelo sistema de antivírus e
 acredita-se estar livre de perigo.


-- 
Esta mensagem foi verificada pelo sistema de antivírus e
 acredita-se estar livre de perigo.



  1   2   3   4   5   >